Chapter 58 Iggy Practice Questions, Chapter 18 Iggy Practice Questions, Chapter 55 Iggy Practice Questions, Chapter 45 Iggy Practice Questions, Chapter 24 Iggy Practice Questions -Integumentary, Chapter 54 Iggy Practice Questions, Chapter 14 Iggy Pra...

¡Supera tus tareas y exámenes ahora con Quizwiz!

A nurse plans care for an older adult who is admitted to the hospital for pneumonia. The client has no known drug allergies and no significant health history. Which action should the nurse include in this client's plan of care? a. Initiate Airborne Precautions. b. Offer fluids every hour or two. c. Place an indwelling urinary catheter. d. Palpate the client's thyroid gland.

A normal age-related endocrine change is decreased antidiuretic hormone (ADH) production. This results in a more diluted urine output, which can lead to dehydration. If no contraindications are known, the nurse should offer (or delegate) the client something to drink at least every 2 hours. A client with simple pneumonia would not require Airborne Precautions. Indwelling urinary catheterization is not necessary for this client and would increase the client's risk for infection. The nurse should plan a toileting schedule and assist the client to the bathroom if needed. Palpating the client's thyroid gland is a part of a comprehensive examination but is not specifically related to this client.

A nurse assesses a client who has a history of migraines. Which clinical manifestation should the nurse identify as an early sign of a migraine with aura? a. Vertigo b. Lethargy c. Visual disturbances d. Numbness of the tongue

ANS: C Early warning of impending migraine with aura usually consists of visual changes, flashing lights, or diplopia. The other manifestations are not associated with an impending migraine with aura.

A client is scheduled to have a fundoplication. What statement by the client indicates a need to review preoperative teaching? a. "After the operation I can eat anything I want." b. "I will have to eat smaller, more frequent meals." c. "I will take stool softeners for several weeks." d. "This surgery may not totally control my symptoms."

ANS: A Nutritional and lifestyle changes need to continue after surgery as the procedure does not offer a lifetime cure. The other statements show good understanding.

A nurse cares for a client who has a family history of diabetes mellitus. The client states, "My father has type 1 diabetes mellitus. Will I develop this disease as well?" How should the nurse respond? a. "Your risk of diabetes is higher than the general population, but it may not occur." b. "No genetic risk is associated with the development of type 1 diabetes mellitus." c. "The risk for becoming a diabetic is 50% because of how it is inherited." d. "Female children do not inherit diabetes mellitus, but male children will."

ANS: A Risk for type 1 diabetes is determined by inheritance of genes coding for HLA-DR and HLA-DQ tissue types. Clients who have one parent with type 1 diabetes are at increased risk for its development. Diabetes (type 1) seems to require interaction between inherited risk and environmental factors, so not everyone with these genes develops diabetes. The other statements are not accurate.

After teaching a client with diabetes mellitus to inject insulin, the nurse assesses the client's understanding. Which statement made by the client indicates a need for additional teaching? a. "The lower abdomen is the best location because it is closest to the pancreas." b. "I can reach my thigh the best, so I will use the different areas of my thighs." c. "By rotating the sites in one area, my chance of having a reaction is decreased." d. "Changing injection sites from the thigh to the arm will change absorption rates."

ANS: A The abdominal site has the fastest rate of absorption because of blood vessels in the area, not because of its proximity to the pancreas. The other statements are accurate assessments of insulin administration.

A client is in the emergency department with an esophageal trauma. The nurse palpates subcutaneous emphysema in the mediastinal area and up into the lower part of the client's neck. What action by the nurse takes priority? a. Assess the client's oxygenation. b. Facilitate a STAT chest x-ray. c. Prepare for immediate surgery. d. Start two large-bore IVs.

ANS: A The priorities of care are airway, breathing, and circulation. The priority option is to assess oxygenation. This occurs before diagnostic or therapeutic procedures. The client needs two large-bore IVs as a trauma client, but oxygenation comes first.

A nurse assesses a client who has diabetes mellitus and notes the client is awake and alert, but shaky, diaphoretic, and weak. Five minutes after administering a half-cup of orange juice, the client's clinical manifestations have not changed. Which action should the nurse take next? a. Administer another half-cup of orange juice. b. Administer a half-ampule of dextrose 50% intravenously. c. Administer 10 units of regular insulin subcutaneously. d. Administer 1 mg of glucagon intramuscularly.

ANS: A This client is experiencing mild hypoglycemia. For mild hypoglycemic manifestations, the nurse should administer oral glucose in the form of orange juice. If the symptoms do not resolve immediately, the treatment should be repeated. The client does not need intravenous dextrose, insulin, or glucagon.

While assessing a client with Graves' disease, the nurse notes that the client's temperature has risen 1° F. Which action should the nurse take first? a. Turn the lights down and shut the client's door. b. Call for an immediate electrocardiogram (ECG). c. Calculate the client's apical-radial pulse deficit. d. Administer a dose of acetaminophen (Tylenol).

ANS: A A temperature increase of 1° F may indicate the development of thyroid storm, and the provider needs to be notified. But before notifying the provider, the nurse should take measures to reduce environmental stimuli that increase the risk of cardiac complications. The nurse can then call for an ECG. The apical-radial pulse deficit would not be necessary, and Tylenol is not needed because the temperature increase is due to thyroid activity.

To promote comfort after a colonoscopy, in what position does the nurse place the client? a. Left lateral b. Prone c. Right lateral d. Supine

ANS: A After colonoscopy, clients have less discomfort and quicker passage of flatus when placed in the left lateral position.

A client has a large oral tumor. What assessment by the nurse takes priority? a. Airway b. Breathing c. Circulation d. Nutrition

ANS: A Airway always takes priority. Airway must be assessed first and any problems resolved if present.

A nurse cares for a client with a deficiency of aldosterone. Which assessment finding should the nurse correlate with this deficiency? a. Increased urine output b. Vasoconstriction c. Blood glucose of 98 mg/dL d. Serum sodium of 144 mEq/L

ANS: A Aldosterone, the major mineralocorticoid, maintains extracellular fluid volume. It promotes sodium and water reabsorption and potassium excretion in the kidney tubules. A client with an aldosterone deficiency will have increased urine output. Vasoconstriction is not related. These sodium and glucose levels are normal; in aldosterone deficiency, the client would have hyponatremia and hyperkalemia.

The nurse is working with a client who has rheumatoid arthritis (RA). The nurse has identified the priority problem of poor body image for the client. What finding by the nurse indicates goals for this client problem are being met? a. Attends meetings of a book club b. Has a positive outlook on life c. Takes medication as directed d. Uses assistive devices to protect joints

ANS: A All of the activities are appropriate for a client with RA. Clients who have a poor body image are often reluctant to appear in public, so attending public book club meetings indicates that goals for this client problem are being met.

A nurse working with clients who experience alopecia knows that which is the best method of helping clients manage the psychosocial impact of this problem? a. Assisting the client to pre-plan for this event b. Reassuring the client that alopecia is temporary c. Teaching the client ways to protect the scalp d. Telling the client that there are worse side effects

ANS: A Alopecia does not occur for all clients who have cancer, but when it does, it can be devastating. The best action by the nurse is to teach the client about the possibility and to give the client multiple choices for preparing for this event. Not all clients will have the same reaction, but some possible actions the client can take are buying a wig ahead of time, buying attractive hats and scarves, and having a hairdresser modify a wig to look like the client's own hair. Teaching about scalp protection is important but does not address the psychosocial impact. Reassuring the client that hair loss is temporary and telling him or her that there are worse side effects are both patronizing and do not give the client tools to manage this condition.

A client had an embolic stroke and is having an echocardiogram. When the client asks why the provider ordered "a test on my heart," how should the nurse respond? a. "Most of these types of blood clots come from the heart." b. "Some of the blood clots may have gone to your heart too." c. "We need to see if your heart is strong enough for therapy." d. "Your heart may have been damaged in the stroke too."

ANS: A An embolic stroke is caused when blood clots travel from one area of the body to the brain. The most common source of the clots is the heart. The other statements are inaccurate.

A clinic nurse is teaching a client prior to surgery. The client does not seem to comprehend the teaching, forgets a lot of what is said, and asks the same questions again and again. What action by the nurse is best? a. Assess the client for anxiety. b. Break the information into smaller bits. c. Give the client written information. d. Review the information again.

ANS: A Anxiety can interfere with learning and cooperation. The nurse should assess the client for anxiety. The other actions are appropriate too, and can be included in the teaching plan, but effective teaching cannot occur if the client is highly anxious.

The nurse working with oncology clients understands that which age-related change increases the older client's susceptibility to infection during chemotherapy? a. Decreased immune function b. Diminished nutritional stores c. Existing cognitive deficits d. Poor physical reserves

ANS: A As people age, there is an age-related decrease in immune function, causing the older adult to be more susceptible to infection than other clients. Not all older adults have diminished nutritional stores, cognitive dysfunction, or poor physical reserves.

A client is having a temporary tracheostomy placed during surgery for oral cancer. What action by the nurse is best to relieve anxiety? a. Agree on a postoperative communication method. b. Explain that staff will answer the call light promptly. c. Give the client a Magic Slate to write on postoperatively. d. Reassure the client that you will take care of all of his or her needs.

ANS: A Before surgery that interrupts the client's ability to communicate, the nurse, client, and family (if possible) agree upon a method of communication in the postoperative period. The client may or may not prefer a slate and may not be able to communicate in writing. Reassuring the client and telling him or her you will take care of all of his or her needs does not help the client be an active participant in care. Ensuring that the staff will answer the call light promptly will not guarantee this will occur.

The nurse caring for oncology clients knows that which form of metastasis is the most common? a. Bloodborne b. Direct invasion c. Lymphatic spread d. Via bone marrow

ANS: A Bloodborne metastasis is the most common way for cancer to metastasize. Direct invasion and lymphatic spread are other methods. Bone marrow is not a medium in which cancer spreads, although cancer can occur in the bone marrow.

A client has a brain abscess and is receiving phenytoin (Dilantin). The spouse questions the use of the drug, saying the client does not have a seizure disorder. What response by the nurse is best? a. "Increased pressure from the abscess can cause seizures." b. "Preventing febrile seizures with an abscess is important." c. "Seizures always occur in clients with brain abscesses." d. "This drug is used to sedate the client with an abscess."

ANS: A Brain abscesses can lead to seizures as a complication. The nurse should explain this to the spouse. Phenytoin is not used to prevent febrile seizures. Seizures are possible but do not always occur in clients with brain abscesses. This drug is not used for sedation.

A client is prescribed cetuximab (Erbitux) for oral cancer and asks the nurse how it works. What response by the nurse is best? a. "It blocks epidermal growth factor." b. "It cuts off the tumor's blood supply." c. "It prevents tumor extension." d. "It targets rapidly dividing cells."

ANS: A Cetuximab (Erbitux) targets and blocks the epidermal growth factor, which contributes to the growth of oral cancers. The other explanations are not correct.

A nurse in the oncology clinic is providing preoperative education to a client just diagnosed with cancer. The client has been scheduled for surgery in 3 days. What action by the nurse is best? a. Call the client at home the next day to review teaching. b. Give the client information about a cancer support group. c. Provide all the preoperative instructions in writing. d. Reassure the client that surgery will be over soon.

ANS: A Clients are often overwhelmed at a sudden diagnosis of cancer and may be more overwhelmed at the idea of a major operation so soon. This stress significantly impacts the client's ability to understand, retain, and recall information. The nurse should call the client at home the next day to review the teaching and to answer questions. The client may or may not be ready to investigate a support group, but this does not help with teaching. Giving information in writing is important (if the client can read it), but in itself will not be enough. Telling the client that surgery will be over soon is giving false reassurance and does nothing for teaching.

A client has trigeminal neuralgia and has begun skipping meals and not brushing his teeth, and his family believes he has become depressed. What action by the nurse is best? a. Ask the client to explain his feelings related to this disorder. b. Explain how dental hygiene is related to overall health. c. Refer the client to a medical social worker for assessment. d. Tell the client that he will become malnourished in time.

ANS: A Clients with trigeminal neuralgia are often afraid of causing pain, so they may limit eating, talking, dental hygiene, and socializing. The nurse first assesses the client for feelings related to having the disorder to determine if a psychosocial link is involved. The other options may be needed depending on the outcome of the initial assessment.

A client with a stroke has damage to Broca's area. What intervention to promote communication is best for this client? a. Assess whether or not the client can write. b. Communicate using "yes-or-no" questions. c. Reinforce speech therapy exercises. d. Remind the client not to use neologisms.

ANS: A Damage to Broca's area often leads to expressive aphasia, wherein the client can understand what is said but cannot express thoughts verbally. In some instances the client can write. The nurse should assess to see if that ability is intact. "Yes-or-no" questions are not good for this type of client because he or she will often answer automatically but incorrectly. Reinforcing speech therapy exercises is good for all clients with communication difficulties. Neologisms are made-up "words" often used by clients with sensory aphasia.

An older client has had an instance of drug toxicity and asks why this happens, since the client has been on this medication for years at the same dose. What response by the nurse is best? a. "Changes in your liver cause drugs to be metabolized differently." b. "Perhaps you don't need as high a dose of the drug as before." c. "Stomach muscles atrophy with age and you digest more slowly." d. "Your body probably can't tolerate as much medication anymore."

ANS: A Decreased liver enzyme activity depresses drug metabolism, which leads to accumulation of drugs—possibly to toxic levels. The other options do not accurately explain this age-related change.

A client takes celecoxib (Celebrex) for chronic osteoarthritis in multiple joints. After a knee replacement, the health care provider has prescribed morphine sulfate for postoperative pain relief. The client also requests the celecoxib in addition to the morphine. What action by the nurse is best? a. Consult with the health care provider about administering both drugs to the client. b. Inform the client that the celecoxib will be started when he or she goes home. c. Teach the client that, since morphine is stronger, celecoxib is not needed. d. Tell the client he or she should not take both drugs at the same time.

ANS: A Despite getting an opioid analgesic for postoperative pain, the nurse should be aware that the client may be on other medications for arthritis in other joints. The nonsteroidal anti-inflammatory drug celecoxib will also help with the postoperative pain. The nurse should consult the provider about continuing the celecoxib while the client is in the hospital. The other responses are not warranted, as the client should be restarted on this medication postoperatively.

The nurse on an inpatient rheumatology unit receives a hand-off report on a client with an acute exacerbation of systemic lupus erythematosus (SLE). Which reported laboratory value requires the nurse to assess the client further? a. Creatinine: 3.9 mg/dL b. Platelet count: 210,000/mm3 c. Red blood cell count: 5.2/mm3 d. White blood cell count: 4400/mm3

ANS: A Lupus nephritis is the leading cause of death in clients with SLE. The creatinine level is very high and the nurse needs to perform further assessments related to this finding. The other laboratory values are normal.

A client is started on etanercept (Enbrel). What teaching by the nurse is most appropriate? a. Giving subcutaneous injections b. Having a chest x-ray once a year c. Taking the medication with food d. Using heat on the injection site

ANS: A Etanercept is given as a subcutaneous injection twice a week. The nurse should teach the client how to self-administer the medication. The other options are not appropriate for etanercept.

A nurse works on an oncology unit and delegates personal hygiene to an unlicensed assistive personnel (UAP). What action by the UAP requires intervention from the nurse? a. Allowing a very tired client to skip oral hygiene and sleep b. Assisting clients with washing the perianal area every 12 hours c. Helping the client use a soft-bristled toothbrush for oral care d. Reminding the client to rinse the mouth with water or saline

ANS: A Even though clients may be tired, they still need to participate in hygiene to help prevent infection. The other options are all appropriate.

A nurse promotes the prevention of lower back pain by teaching clients at a community center. Which instruction should the nurse include in this education? a. "Participate in an exercise program to strengthen muscles." b. "Purchase a mattress that allows you to adjust the firmness." c. "Wear flat instead of high-heeled shoes to work each day." d. "Keep your weight within 20% of your ideal body weight."

ANS: A Exercise can strengthen back muscles, reducing the incidence of low back pain. The other options will not prevent low back pain.

A client comes to the family medicine clinic and reports joint pain and stiffness. The nurse is asked to assess the client for Heberden's nodules. What assessment technique is correct? a. Inspect the client's distal finger joints. b. Palpate the client's abdomen for tenderness. c. Palpate the client's upper body lymph nodes. d. Perform range of motion on the client's wrists.

ANS: A Herberden's nodules are seen in osteoarthritis and are bony nodules at the distal interphalangeal joints. To assess for this finding, the nurse inspects the client's distal fingertips. These nodules are not found in the abdomen, lymph nodes, or wrists.

A nurse cares for a client who has obstructive jaundice. The client asks, "Why is my skin so itchy?" How should the nurse respond? a. "Bile salts accumulate in the skin and cause the itching." b. "Toxins released from an inflamed gallbladder lead to itching." c. "Itching is caused by the release of calcium into the skin." d. "Itching is caused by a hypersensitivity reaction."

ANS: A In obstructive jaundice, the normal flow of bile into the duodenum is blocked, allowing excess bile salts to accumulate on the skin. This leads to itching, or pruritus. The other statements are not accurate.

A client with myasthenia gravis (MG) asks the nurse to explain the disease. What response by the nurse is best? a. "MG is an autoimmune problem in which nerves do not cause muscles to contract." b. "MG is an inherited destruction of peripheral nerve endings and junctions." c. "MG consists of trauma-induced paralysis of specific cranial nerves." d. "MG is a viral infection of the dorsal root of sensory nerve fibers."

ANS: A MG is an autoimmune disorder in which nerve fibers are damaged and their impulses do not lead to muscle contraction. MG is not an inherited or viral disorder and does not paralyze specific cranial nerves.

The nurse reads a client's chart and sees that the health care provider assessed mucosal erythroplasia. What should the nurse understand that this means for the client? a. Early sign of oral cancer b. Fungal mouth infection c. Inflammation of the gums d. Obvious oral tumor

ANS: A Mucosal erythroplasia is the earliest sign of oral cancer. It is not a fungal infection, inflammation of the gums, or an obvious tumor.

A client in the orthopedic clinic has a self-reported history of osteoarthritis. The client reports a low-grade fever that started when the weather changed and several joints started "acting up," especially both hips and knees. What action by the nurse is best? a. Assess the client for the presence of subcutaneous nodules or Baker's cysts. b. Inspect the client's feet and hands for podagra and tophi on fingers and toes. c. Prepare to teach the client about an acetaminophen (Tylenol) regimen. d. Reassure the client that the problems will fade as the weather changes again.

ANS: A Osteoarthritis is not a systemic disease, nor does it present bilaterally. These are manifestations of rheumatoid arthritis. The nurse should assess for other manifestations of this disorder, including subcutaneous nodules and Baker's cysts. Podagra and tophi are seen in gout. Acetaminophen is not used for rheumatoid arthritis. Telling the client that the symptoms will fade with weather changes is not accurate.

A client scheduled for a percutaneous transhepatic cholangiography (PTC) denies allergies to medication. What action by the nurse is best? a. Ask the client about shellfish allergies. b. Document this information on the chart. c. Ensure that the client has a ride home. d. Instruct the client on bowel preparation.

ANS: A PTC uses iodinated dye, so the client should be asked about seafood allergies, specifically to shellfish. Documentation should occur, but this is not the priority. The client will need a ride home afterward if the procedure is done on an outpatient basis. There is no bowel preparation for PTC.

A nurse assesses a client who has two skin lesions on his chest. Each lesion is the size of a nickel, flat, and darker in color than the client's skin. How should the nurse document these lesions? a. Two 2-cm hyperpigmented patches b. Two 1-inch erythematous plaques c. Two 2-mm pigmented papules d. Two 1-inch moles

ANS: A Patches are larger flat areas of the skin. The information provided does not indicate a mole or the presence of erythema.

A client is in the clinic for a follow-up visit after a moderate traumatic brain injury. The client's spouse is very frustrated, stating that the client's personality has changed and the situation is intolerable. What action by the nurse is best? a. Explain that personality changes are common following brain injuries. b. Ask the client why he or she is acting out and behaving differently. c. Refer the client and spouse to a head injury support group. d. Tell the spouse this is expected and he or she will have to learn to cope.

ANS: A Personality and behavior often change permanently after head injury. The nurse should explain this to the spouse. Asking the client about his or her behavior isn't useful because the client probably cannot help it. A referral might be a good idea, but the nurse needs to do something in addition to just referring the couple. Telling the spouse to learn to cope belittles the spouse's concerns and feelings.

A nurse assesses a client who is recovering from anterior cervical diskectomy and fusion. Which complication should alert the nurse to urgently communicate with the health care provider? a. Auscultated stridor b. Weak pedal pulses c. Difficulty swallowing d. Inability to shrug shoulders

ANS: A Postoperative swelling can narrow the trachea, cause a partial airway obstruction, and manifest as stridor. The client may also have trouble swallowing, but maintaining an airway takes priority. Weak pedal pulses and an inability to shrug the shoulders are not complications of this surgery.

A nurse assesses a client who is recovering from a paracentesis 1 hour ago. Which assessment finding requires action by the nurse? a. Urine output via indwelling urinary catheter is 20 mL/hr b. Blood pressure increases from 110/58 to 120/62 mm Hg c. Respiratory rate decreases from 18 to 14 breaths/min d. A decrease in the client's weight by 6 kg

ANS: A Rapid removal of ascetic fluid causes decreased abdominal pressure, which can contribute to hypovolemia. This can be manifested by a decrease in urine output to below 30 mL/hr. A slight increase in systolic blood pressure is insignificant. A decrease in respiratory rate indicates that breathing has been made easier by the procedure. The nurse would expect the client's weight to drop as fluid is removed. Six kilograms is less than 3 pounds and is expected.

A nurse teaches a client who has been prescribed a 24-hour urine collection to measure excreted hormones. The client asks, "Why do I need to collect urine for 24 hours instead of providing a random specimen?" How should the nurse respond? a. "This test will assess for a hormone secreted on a circadian rhythm." b. "The hormone is diluted in urine; therefore, we need a large volume." c. "We are assessing when the hormone is secreted in large amounts." d. "To collect the correct hormone, you need to urinate multiple times."

ANS: A Some hormones are secreted in a pulsatile, or circadian, cycle. When testing for these substances, a collection that occurs over 24 hours will most accurately reflect hormone secretion. Dilution of hormones in urine, secretion of hormone amounts, and ability to collect the correct hormone are not reasons to complete a 24-hour urine test.

A nurse assesses a client who is prescribed a medication that stimulates beta1 receptors. Which assessment finding should alert the nurse to urgently contact the health care provider? a. Heart rate of 50 beats/min b. Respiratory rate of 18 breaths/min c. Oxygenation saturation of 92% d. Blood pressure of 144/69 mm Hg

ANS: A Stimulation of beta1 receptor sites in the heart has positive chronotropic and inotropic actions. The nurse expects an increase in heart rate and increased cardiac output. The client with a heart rate of 50 beats/min would be cause for concern because this would indicate that the client was not responding to the medication. The other vital signs are within normal limits and do not indicate a negative response to the medication.

A student is caring for clients in the preoperative area. The nurse contacts the surgeon about a client whose heart rate is 120 beats/min. After consulting with the surgeon, the nurse administers a beta blocker to the client. The student asks why this was needed. What response by the nurse is best? a. "A rapid heart rate requires more effort by the heart." b. "Anesthesia has bad effects if the client is tachycardic." c. "The client may have an undiagnosed heart condition." d. "When the heart rate goes up, the blood pressure does too."

ANS: A Tachycardia increases the workload of the heart and requires more oxygen delivery to the myocardial tissues. This added strain is not needed on top of the physical and emotional stress of surgery. The other statements are not accurate.

A client is scheduled for a below-the-knee amputation. The circulating nurse ensures the proper side is marked prior to the start of surgery. What action by the nurse is most appropriate? a. Facilitate marking the site with the client and surgeon. b. Have the client mark the operative site. c. Mark the operative site with a waterproof marker. d. Tell the surgeon it is time to mark the surgical site.

ANS: A The Joint Commission now recommends that both the client and the surgeon mark the operative site together in order to prevent wrong-site surgery. The nurse should facilitate this process.

The perioperative nurse manager and the postoperative unit manager are concerned about the increasing number of surgical infections in their hospital. What action by the managers is best? a. Audit charts to see if the Surgical Care Improvement Project (SCIP) outcomes were met. b. Encourage staff on both units to provide peer pressure to adhere to hand hygiene policy. c. Hold educational meetings with the nursing and surgical staff on infection prevention. d. Monitor staff on both units for consistent adherence to established hand hygiene practices.

ANS: A The SCIP project contains core measures that are mandatory for all surgical clients and focuses on preventing infection, serious cardiac events, and venous thromboembolism. The managers should start by reviewing charts to see if the guidelines of this project were implemented. The other actions may be necessary too, but first the managers need to assess the situation.

A nurse cares for a client with end-stage pancreatic cancer. The client asks, "Why is this happening to me?" How should the nurse respond? a. "I don't know. I wish I had an answer for you, but I don't." b. "It's important to keep a positive attitude for your family right now." c. "Scientists have not determined why cancer develops in certain people." d. "I think that this is a trial so you can become a better person because of it."

ANS: A The client is not asking the nurse to actually explain why the cancer has occurred. The client may be expressing his or her feelings of confusion, frustration, distress, and grief related to this diagnosis. Reminding the client to keep a positive attitude for his or her family does not address the client's emotions or current concerns. The nurse should validate that there is no easy or straightforward answer as to why the client has cancer. Telling a client that cancer is a trial is untrue and may diminish the client-nurse relationship.

A nurse cares for a client with hepatopulmonary syndrome who is experiencing dyspnea with oxygen saturations at 92%. The client states, "I do not want to wear the oxygen because it causes my nose to bleed. Get out of my room and leave me alone!" Which action should the nurse take? a. Instruct the client to sit in as upright a position as possible. b. Add humidity to the oxygen and encourage the client to wear it. c. Document the client's refusal, and call the health care provider. d. Contact the provider to request an extra dose of the client's diuretic.

ANS: A The client with hepatopulmonary syndrome is often dyspneic. Because the oxygen saturation is not significantly low, the nurse should first allow the client to sit upright to see if that helps. If the client remains dyspneic, or if the oxygen saturation drops further, the nurse should investigate adding humidity to the oxygen and seeing whether the client will tolerate that. The other two options may be beneficial, but they are not the best choices. If the client is comfortable, his or her agitation will decrease; this will improve respiratory status.

A nurse assesses a client diagnosed with adrenal hypofunction. Which client statement should the nurse correlate with this diagnosis? a. "I have a terrible craving for potato chips." b. "I cannot seem to drink enough water." c. "I no longer have an appetite for anything." d. "I get hungry even after eating a meal."

ANS: A The nurse correlates a client's salt craving with adrenal hypofunction. Excessive thirst is related to diabetes insipidus or diabetes mellitus. Clients who have hypothyroidism often have a decrease in appetite. Excessive hunger is associated with diabetes mellitus.

A client has been diagnosed with rheumatoid arthritis. The client has experienced increased fatigue and worsening physical status and is finding it difficult to maintain the role of elder in his cultural community. The elder is expected to attend social events and make community decisions. Stress seems to exacerbate the condition. What action by the nurse is best? a. Assess the client's culture more thoroughly. b. Discuss options for performing duties. c. See if the client will call a community meeting. d. Suggest the client give up the role of elder.

ANS: A The nurse needs a more thorough understanding of the client's culture, including the meaning of illness and the ramifications of the elder not being able to perform traditional duties. This must be done prior to offering any possible solutions. If the nurse does not understand the consequences of what is suggested, the client may simply be unwilling to listen or participate in problem solving. The other options may be reasonable depending on the outcome of a better cultural understanding.

A nurse is concerned that a preoperative client has a great deal of anxiety about the upcoming procedure. What action by the nurse is best? a. Ask the client to describe current feelings. b. Determine if the client wants a chaplain. c. Reassure the client this surgery is common. d. Tell the client there is no need to be anxious.

ANS: A The nurse needs to conduct further assessment of the client's anxiety. Asking open-ended questions about current feelings is an appropriate way to begin. The client may want a chaplain, but the nurse needs to do more for the client. Reassurance can be good, but false hope is not, and simply reassuring the client may not be helpful. Telling the client not to be anxious belittles the client's feelings.

A client is on the phone when the nurse brings a preoperative antibiotic before scheduled surgery. The circulating nurse has requested the antibiotic be started. The client wants the nurse to wait before starting it. What response by the nurse is most appropriate? a. Explain the rationale for giving the medicine now. b. Leave the room and come back in 15 minutes. c. Provide holistic client care and come back later. d. Tell the client you must start the medication now.

ANS: A The preoperative antibiotic must be given within 60 minutes of the surgical start time to ensure the proper amount is in the tissues when the incision is made. The nurse should explain the rationale to the client for this timing. The other options do not take this timing into consideration and do not give the client the information needed to be cooperative.

A nurse cares for a client who is prescribed lactulose (Heptalac). The client states, "I do not want to take this medication because it causes diarrhea." How should the nurse respond? a. "Diarrhea is expected; that's how your body gets rid of ammonia." b. "You may take Kaopectate liquid daily for loose stools." c. "Do not take any more of the medication until your stools firm up." d. "We will need to send a stool specimen to the laboratory."

ANS: A The purpose of administering lactulose to this client is to help ammonia leave the circulatory system through the colon. Lactulose draws water into the bowel with its high osmotic gradient, thereby producing a laxative effect and subsequently evacuating ammonia from the bowel. The client must understand that this is an expected and therapeutic effect for him or her to remain compliant. The nurse should not suggest administering anything that would decrease the excretion of ammonia or holding the medication. There is no need to send a stool specimen to the laboratory because diarrhea is the therapeutic response to this medication.

A nurse assesses a client who is recovering from an open Whipple procedure. Which action should the nurse perform first? a. Assess the client's endotracheal tube with 40% FiO2. b. Insert an indwelling Foley catheter to gravity drainage. c. Place the client's nasogastric tube to low intermittent suction. d. Start lactated Ringer's solution through an intravenous catheter.

ANS: A Using the ABCs, airway and oxygenation status should always be assessed first, so checking the endotracheal tube is the first action. Next, the nurse should start the IV line (circulation). After that, the Foley catheter can be inserted and the nasogastric tube can be set.

A nurse is caring for a client with systemic sclerosis. The client's facial skin is very taut, limiting the client's ability to open the mouth. After consulting with a registered dietitian for appropriate nutrition, what other consultation should the nurse facilitate? a. Dentist b. Massage therapist c. Occupational therapy d. Physical therapy

ANS: A With limited ability to open the mouth, dental hygiene may be lacking. The nurse should encourage the client to see a dentist. The other referrals are not related to the mouth.

A nurse delegates care for a client who has open skin lesions. Which statements should the nurse include when delegating this client's hygiene care to an unlicensed assistive personnel (UAP)? (Select all that apply.) a."Wash your hands before touching the client." b."Wear gloves when bathing the client." c."Assess skin for breakdown during the bath." d."Apply lotion to lesions while the skin is wet." e."Use a damp cloth to scrub the lesions."

ANS: A, B All health care providers should follow Standard Precautions when caring for clients who have any open skin areas. This includes hand hygiene and wearing gloves when in contact with the lesions. The UAP is not qualified to assess the client's skin. The other statements are not appropriate for the care of open skin lesions.

A nurse teaches a client to perform total skin self-examinations on a monthly basis. Which statements should the nurse include in this client's teaching? (Select all that apply.) a. "Look for asymmetry of shape and irregular borders." b. "Assess for color variation within each lesion." c. "Examine the distribution of lesions over a section of the body." d. "Monitor for edema or swelling of tissues." e. "Focus your assessment on skin areas that itch."

ANS: A, B Clients should be taught to examine each lesion following the ABCDE features associated with skin cancer: asymmetry of shape, border irregularity, color variation within one lesion, diameter greater than 6 mm, and evolving or changing in any feature.

A nurse cares for a client who presents with tachycardia and prostration related to biliary colic. Which actions should the nurse take? (Select all that apply.) a. Contact the provider immediately. b. Lower the head of the bed. c. Decrease intravenous fluids. d. Ask the client to bear down. e. Administer prescribed opioids.

ANS: A, B Clients who are experiencing biliary colic may present with tachycardia, pallor, diaphoresis, prostration, or other signs of shock. The nurse should stay with the client, lower the client's head, and contact the provider or Rapid Response Team for immediate assistance. Treatment for shock usually includes intravenous fluids; therefore, decreasing fluids would be an incorrect intervention. The client's tachycardia is a result of shock, not pain. Performing the vagal maneuver or administering opioids could knock out the client's compensation mechanism.

A client has meningitis following brain surgery. What comfort measures may the nurse delegate to the unlicensed assistive personnel (UAP)? (Select all that apply.) a. Applying a cool washcloth to the head b. Assisting the client to a position of comfort c. Keeping voices soft and soothing d. Maintaining low lighting in the room e. Providing antipyretics for fever

ANS: A, B, C, D The client with meningitis often has high fever, pain, and some degree of confusion. Cool washcloths to the forehead are comforting and help with pain. Allowing the client to assume a position of comfort also helps manage pain. Keeping voices low and lights dimmed also helps convey caring in a nonthreatening manner. The nurse provides antipyretics for fever.

The nursing student observing in the perioperative area notes the unique functions of the circulating nurse, which include which roles? (Select all that apply.) a. Ensuring the client's safety b. Accounting for all sharps c. Documenting all care given d. Maintaining the sterile field e. Monitoring traffic in the room

ANS: A, E The circulating nurse has several functions, including maintaining client safety and privacy, monitoring traffic in and out of the operating room, assessing fluid losses, reporting findings to the surgeon and anesthesia provider, anticipating needs of the team, and communicating to the family. The circulating nurse and scrub person work together to ensure accurate counts of sharps, sponges, and instruments. The circulating nurse also documents care, but in the perioperative area, the preoperative or holding room nurse would also document care received there. Maintaining the sterile field is a joint responsibility among all members of the surgical team.

After teaching a client with type 2 diabetes mellitus, the nurse assesses the client's understanding. Which statement made by the client indicates a need for additional teaching? a. "I need to have an annual appointment even if my glucose levels are in good control." b. "Since my diabetes is controlled with diet and exercise, I must be seen only if I am sick." c. "I can still develop complications even though I do not have to take insulin at this time." d. "If I have surgery or get very ill, I may have to receive insulin injections for a short time."

ANS: B Clients with diabetes need to be seen at least annually to monitor for long-term complications, including visual changes, microalbuminuria, and lipid analysis. The client may develop complications and may need insulin in the future.

After teaching a client who has psoriasis, a nurse assesses the client's understanding. Which statement indicates the client needs additional teaching? a."At the next family reunion, I'm going to ask my relatives if they have psoriasis." b."I have to make sure I keep my lesions covered, so I do not spread this to others." c."I expect that these patches will get smaller when I lie out in the sun." d."I should continue to use the cortisone ointment as the patches shrink and dry out."

ANS: B Psoriasis is not a contagious disorder. The client does not have to worry about spreading the condition to others. It is a condition that has hereditary links, the patches will decrease in size with ultraviolet light exposure, and cortisone ointment should be applied directly to lesions to suppress cell division.

A nurse assesses a client with diabetes mellitus. Which clinical manifestation should alert the nurse to decreased kidney function in this client? a. Urine specific gravity of 1.033 b. Presence of protein in the urine c. Elevated capillary blood glucose level d. Presence of ketone bodies in the urine

ANS: B Renal dysfunction often occurs in the client with diabetes. Proteinuria is a result of renal dysfunction. Specific gravity is elevated with dehydration. Elevated capillary blood glucose levels and ketones in the urine are consistent with diabetes mellitus but are not specific to renal function.

A nurse develops a dietary plan for a client with diabetes mellitus and new-onset microalbuminuria. Which component of the client's diet should the nurse decrease? a. Carbohydrates b. Proteins c. Fats d. Total calories

ANS: B Restriction of dietary protein to 0.8 g/kg of body weight per day is recommended for clients with microalbuminuria to delay progression to renal failure. The client's diet does not need to be decreased in carbohydrates, fats, or total calories.

A nurse teaches a client about self-monitoring of blood glucose levels. Which statement should the nurse include in this client's teaching to prevent bloodborne infections? a. "Wash your hands after completing each test." b. "Do not share your monitoring equipment." c. "Blot excess blood from the strip with a cotton ball." d. "Use gloves when monitoring your blood glucose."

ANS: B Small particles of blood can adhere to the monitoring device, and infection can be transported from one user to another. Hepatitis B in particular can survive in a dried state for about a week. The client should be taught to avoid sharing any equipment, including the lancet holder. The client should be taught to wash his or her hands before testing. The client would not need to blot excess blood away from the strip or wear gloves.

A nurse reviews laboratory results for a client with diabetes mellitus who is prescribed an intensified insulin regimen: • Fasting blood glucose: 75 mg/dL • Postprandial blood glucose: 200 mg/dL • Hemoglobin A1c level: 5.5% How should the nurse interpret these laboratory findings? a. Increased risk for developing ketoacidosis b. Good control of blood glucose c. Increased risk for developing hyperglycemia d. Signs of insulin resistance

ANS: B The client is maintaining blood glucose levels within the defined ranges for goals in an intensified regimen. Because the client's glycemic control is good, he or she is not at higher risk for ketoacidosis or hyperglycemia and is not showing signs of insulin resistance.

After teaching a client who is at risk for the formation of pressure ulcers, a nurse assesses the client's understanding. Which dietary choice by the client indicates a good understanding of the teaching? a.Low-fat diet with whole grains and cereals and vitamin supplements b.High-protein diet with vitamins and mineral supplements c.Vegetarian diet with nutritional supplements and fish oil capsules d.Low-fat, low-cholesterol, high-fiber, low-carbohydrate diet

ANS: B The preferred diet is high in protein to assist in wound healing and prevention of new wounds. Fat is also needed to ensure formation of cell membranes, so any of the options with low fat would not be good choices. A vegetarian diet would not provide fat and high levels of protein

A nurse cares for a client who is prescribed pioglitazone (Actos). After 6 months of therapy, the client reports that his urine has become darker since starting the medication. Which action should the nurse take? a. Assess for pain or burning with urination. b. Review the client's liver function study results. c. Instruct the client to increase water intake. d. Test a sample of urine for occult blood.

ANS: B Thiazolidinediones (including pioglitazone) can affect liver function; liver function should be assessed at the start of therapy and at regular intervals while the client continues to take these drugs. Dark urine is one indicator of liver impairment because bilirubin is increased in the blood and is excreted in the urine. The nurse should check the client's most recent liver function studies. The nurse does not need to assess for pain or burning with urination and does not need to check the urine for occult blood. The client does not need to be told to increase water intake.

A client has a nasogastric (NG) tube after a Nissen fundoplication. The nurse answers the call light and finds the client vomiting bright red blood with the NG tube lying on the floor. What action should the nurse take first? a. Notify the surgeon. b. Put on a pair of gloves. c. Reinsert the NG tube. d. Take a set of vital signs.

ANS: B To avoid exposure to blood and body fluids, the nurse first puts on a pair of gloves. Taking vital signs and notifying the surgeon are also appropriate, but the nurse must protect himself or herself first. The surgeon will reinsert the NG tube either at the bedside or in surgery if the client needs to go back to the operating room.

A client waiting for surgery is very anxious. What intervention can the nurse delegate to the unlicensed assistive personnel (UAP)? a. Assess the client's anxiety. b. Give the client a back rub. c. Remind the client to turn. d. Teach about postoperative care.

ANS: B A back rub reduces anxiety and can be delegated to the UAP. Once teaching has been done, the UAP can remind the client to turn, but this is not related to relieving anxiety. Assessing anxiety and teaching are not within the scope of practice for the UAP.

A client has a platelet count of 9800/mm3. What action by the nurse is most appropriate? a. Assess the client for calf pain, warmth, and redness. b. Instruct the client to call for help to get out of bed. c. Obtain cultures as per the facility's standing policy. d. Place the client on protective isolation precautions.

ANS: B A client with a platelet count this low is at high risk for serious bleeding episodes. To prevent injury, the client should be instructed to call for help prior to getting out of bed. Calf pain, warmth, and redness might indicate a deep vein thrombosis, not associated with low platelets. Cultures and isolation relate to low white cell counts.

A nose obtains a clients's health history at a community health clinic. Which statement alerts the nurse to prove health teaching to this client? a. "I drink two glasses of red wine each week." b. "I take a lot of Tylenol for my arthritis pain." c. "I have a cousin who died of liver cancer." d. "I got a hepatitis vaccine before traveling."

ANS: B Acetaminophen (Tylenol) can cause liver damage if taken in large amounts. Clients should be taught not to exceed 4000 mg/day of acetaminophen. The nurse should teach the client about this limitation and should explore other drug options with the client to manage his or her arthritis pain. Two glasses of wine each week, a cousin with liver cancer, and the hepatitis vaccine do not place the client at risk for a liver disorder, and therefore do not require any health teaching.

A nurse teaches an older adult with a decreased production of estrogen. Which statement should the nurse include in this client's teaching to decrease injury? a. "Drink at least 2 liters of fluids each day." b. "Walk around the neighborhood for daily exercise." c. "Bathe your perineal area twice a day." d. "You should check your blood glucose before meals."

ANS: B An older adult client with decreased production of estrogen is at risk for decreased bone density and fractures. The nurse should encourage the client to participate in weight-bearing exercises such as walking. Drinking fluids and performing perineal care will decrease vaginal drying but not decrease injury. Older adults often have a decreased glucose tolerance, but this is not related to a decrease in estrogen.

A nurse assesses a client who is recovering from a hemorrhoidectomy that was done the day before. The nurse notes that the client has lower abdominal distention accompanied by dullness to percussion over the distended area. Which action should the nurse take? a. Assess the client's heart rate and blood pressure. b. Determine when the client last voided. c. Ask if the client is experiencing flatus. d. Auscultate all quadrants of the client's abdomen

ANS: B Assessment findings indicate that the client may have an over-full bladder. In the immediate postoperative period, the client may experience difficulty voiding due to urinary retention. The nurse should assess when the client last voided. The client's vital signs may be checked after the nurse determines the client's last void. Asking about flatus and auscultating bowel sounds are not related to a hemorrhoidectomy.

The student nurse studying the gastrointestinal system understands that chyme refers to what? a. Hormones that reduce gastric acidity b. Liquefied food ready for digestion c. Nutrients after being absorbed d. Secretions that help digest food

ANS: B Before being digested, food must be broken down into a liquid form. This liquid is called chyme. Secretin is the hormone that inhibits acid production and decreases gastric motility. Absorption is carried out as the nutrients produced by digestion move from the lumen of the GI tract into the body's circulatory system for uptake by individual cells. The secretions that help digest food include hydrochloric acid, bile, and digestive enzymes.

A nurse is teaching a client who experiences migraine headaches and is prescribed a beta blocker. Which statement should the nurse include in this client's teaching? a. "Take this drug only when you have prodromal symptoms indicating the onset of a migraine headache." b. "Take this drug as ordered, even when feeling well, to prevent vascular changes associated with migraine headaches." c. "This drug will relieve the pain during the aura phase soon after a headache has started." d. "This medication will have no effect on your heart rate or blood pressure because you are taking it for migraines."

ANS: B Beta blockers are prescribed as prophylactic treatment to prevent the vascular changes that initiate migraine headaches. Heart rate and blood pressure will also be affected, and the client should monitor these side effects. The other responses do not discuss appropriate uses of the medication.

A nurse cares for a client who has excessive catecholamine release. Which assessment finding should the nurse correlate with this condition? a. Decreased blood pressure b. Increased pulse c. Decreased respiratory rate d. Increased urine output

ANS: B Catecholamines are responsible for the fight-or-flight stress response. Activation of the sympathetic nervous system can be correlated with tachycardia. Catecholamines do not decrease blood pressure or respiratory rate, nor do they increase urine output.

A client who has been taking antibiotics reports severe, watery diarrhea. About which test does the nurse teach the client? a. Colonoscopy b. Enzyme-linked immunosorbent assay (ELISA) toxin A+B c. Ova and parasites d. Stool culture

ANS: B Clients taking antibiotics are at risk for Clostridium difficile infection. The most common test for this disorder is a stool sample for ELISA toxin A+B. Colonoscopy, ova and parasites, and stool culture are not warranted at this time.

A nurse assesses the health history of a client who is prescribed ziconotide (Prialt) for chronic back pain. Which assessment question should the nurse ask? a. "Are you taking a nonsteroidal anti-inflammatory drug?" b. "Do you have a mental health disorder?" c. "Are you able to swallow medications?" d. "Do you smoke cigarettes or any illegal drugs?"

ANS: B Clients who have a mental health or behavioral health problem should not take ziconotide. The other questions do not identify a contraindication for this medication.

A nurse assesses clients for potential endocrine disorders. Which client is at greatest risk for hyperparathyroidism? a. A 29-year-old female with pregnancy-induced hypertension b. A 41-year-old male receiving dialysis for end-stage kidney disease c. A 66-year-old female with moderate heart failure d. A 72-year-old male who is prescribed home oxygen therapy

ANS: B Clients who have chronic kidney disease do not completely activate vitamin D and poorly absorb calcium from the GI tract. They are chronically hypocalcemic, and this triggers overstimulation of the parathyroid glands. Pregnancy-induced hypertension, moderate heart failure, and home oxygen therapy do not place a client at higher risk for hyperparathyroidism.

A nurse teaches a client who is recovering from a spinal fusion. Which statement should the nurse include in this client's postoperative instructions? a. "Only lift items that are 10 pounds or less." b. "Wear your brace whenever you are out of bed." c. "You must remain in bed for 3 weeks after surgery." d. "You are prescribed medications to prevent rejection."

ANS: B Clients who undergo spinal fusion are fitted with a brace that they must wear throughout the healing process (usually 3 to 6 months) whenever they are out of bed. The client should not lift anything. The client does not need to remain in bed. Medications for rejection prevention are not necessary for this procedure.

After teaching a client with irritable bowel syndrome (IBS), a nurse assesses the client's understanding. Which menu selection indicates that the client correctly understands the dietary teaching? a. Ham sandwich on white bread, cup of applesauce, glass of diet cola b. Broiled chicken with brown rice, steamed broccoli, glass of apple juice c. Grilled cheese sandwich, small banana, cup of hot tea with lemon d. Baked tilapia, fresh green beans, cup of coffee with low-fat milk

ANS: B Clients with IBS are advised to eat a high-fiber diet (30 to 40 g/day), with 8 to 10 cups of liquid daily. Chicken with brown rice, broccoli, and apple juice has the highest fiber content. They should avoid alcohol, caffeine, and other gastric irritants.

A client had an upper gastrointestinal hemorrhage and now has a nasogastric (NG) tube. What comfort measure may the nurse delegate to the unlicensed assistive personnel (UAP)? a. Lavaging the tube with ice water b. Performing frequent oral care c. Re-positioning the tube every 4 hours d. Taking and recording vital signs

ANS: B Clients with NG tubes need frequent oral care both for comfort and to prevent infection. Lavaging the tube is done by the nurse. Re-positioning the tube, if needed, is also done by the nurse. The UAP can take vital signs, but this is not a comfort measure.

While assessing a client, a nurse detects a bluish tinge to the client's palms, soles, and mucous membranes. Which action should the nurse take next? a. Ask the client about current medications he or she is taking. b. Use pulse oximetry to assess the client's oxygen saturation. c. Auscultate the client's lung fields for adventitious sounds. d. Palpate the client's bilateral radial and pedal pulses.

ANS: B Cyanosis can be present when impaired gas exchange occurs. In a client with dark skin, cyanosis can be seen because the palms, soles, and mucous membranes have a bluish tinge. The nurse should assess for systemic oxygenation before continuing with other assessments.

A client with cancer has anorexia and mucositis, and is losing weight. The client's family members continually bring favorite foods to the client and are distressed when the client won't eat them. What action by the nurse is best? a. Explain the pathophysiologic reasons behind the client not eating. b. Help the family show other ways to demonstrate love and caring. c. Suggest foods and liquids the client might be willing to try to eat. d. Tell the family the client isn't able to eat now no matter what they bring.

ANS: B Families often become distressed when their loved ones won't eat. Providing food is a universal sign of caring, and to some people the refusal to eat signifies worsening of the condition. The best option for the nurse is to help the family find other ways to demonstrate caring and love, because with treatment-related anorexia and mucositis, the client is not likely to eat anything right now. Explaining the rationale for the problem is a good idea but does not suggest to the family anything that they can do for the client. Simply telling the family the client is not able to eat does not give them useful information and is dismissive of their concerns.

A nurse plans care for a client with hyperparathyroidism. Which intervention should the nurse include in this client's plan of care? a. Ask the client to ambulate in the hallway twice a day. b. Use a lift sheet to assist the client with position changes. c. Provide the client with a soft-bristled toothbrush for oral care. d. Instruct the unlicensed assistive personnel to strain the client's urine for stones.

ANS: B Hyperparathyroidism causes increased resorption of calcium from the bones, increasing the risk for pathologic fractures. Using a lift sheet when moving or positioning the client, instead of pulling on the client, reduces the risk of bone injury. Hyperparathyroidism can cause kidney stones, but not every client will need to have urine strained. The priority is preventing injury. Ambulating in the hall and using a soft toothbrush are not specific interventions for this client.

A client with rheumatoid arthritis (RA) has an acutely swollen, red, and painful joint. What nonpharmacologic treatment does the nurse apply? a. Heating pad b. Ice packs c. Splints d. Wax dip

ANS: B Ice is best for acute inflammation. Heat often helps with joint stiffness. Splinting helps preserve joint function. A wax dip is used to provide warmth to the joint which is more appropriate for chronic pain and stiffness.

A nurse is caring for four clients who might be brain dead. Which client would best meet the criteria to allow assessment of brain death? a. Client with a core temperature of 95° F (35° C) for 2 days b. Client in a coma for 2 weeks from a motor vehicle crash c. Client who is found unresponsive in a remote area of a field by a hunter d. Client with a systolic blood pressure of 92 mm Hg since admission

ANS: B In order to determine brain death, clients must meet four criteria: 1) coma from a known cause, 2) normal or near-normal core temperature, 3) normal systolic blood pressure, and 4) at least one neurologic examination. The client who was in the car crash meets two of these criteria. The clients with the lower temperature and lower blood pressure have only one of these criteria. There is no data to support assessment of brain death in the client found by the hunter.

An inpatient nurse brings an informed consent form to a client for an operation scheduled for tomorrow. The client asks about possible complications from the operation. What response by the nurse is best? a. Answer the questions and document that teaching was done. b. Do not have the client sign the consent and call the surgeon. c. Have the client sign the consent, then call the surgeon. d. Remind the client of what teaching the surgeon has done.

ANS: B In order to give informed consent, the client needs sufficient information. Questions about potential complications should be answered by the surgeon. The nurse should notify the surgeon to come back and answer the client's questions before the client signs the consent form. The other actions are not appropriate.

A nurse assesses a client with hyperthyroidism who is prescribed lithium carbonate. Which assessment finding should alert the nurse to a side effect of this therapy? a. Blurred and double vision b. Increased thirst and urination c. Profuse nausea and diarrhea d. Decreased attention and insomnia

ANS: B Lithium antagonizes antidiuretic hormone and can cause symptoms of diabetes insipidus. This manifests with increased thirst and urination. Lithium has no effect on vision, gastric upset, or level of consciousness.

While assessing a client's lower extremities, a nurse notices that one leg is pale and cooler to the touch. Which assessment should the nurse perform next? a. Ask about a family history of skin disorders. b. Palpate the client's pedal pulses bilaterally. c. Check for the presence of Homans' sign. d. Assess the client's skin for adequate skin turgor

ANS: B Localized, decreased skin temperature and pallor indicate interference with vascular flow to the region. The nurse should assess bilateral pedal pulses to screen for vascular sufficiency. Without adequate blood flow, the client's limb could be threatened. Asking about a family history of skin problems would not take priority over assessing blood flow. Homans' sign is a screening tool for deep vein thrombosis and is often inaccurate. Skin turgor gives information about hydration status. This assessment may be needed but certainly does not take priority over assessing for blood flow.

An emergency department nurse cares for a client who experienced a spinal cord injury 1 hour ago. Which prescribed medication should the nurse prepare to administer? a. Intrathecal baclofen (Lioresal) b. Methylprednisolone (Medrol) c. Atropine sulfate d. Epinephrine (Adrenalin)

ANS: B Methylprednisolone (Medrol) should be given within 8 hours of the injury. Clients who receive this therapy usually show improvement in motor and sensory function. The other medications are inappropriate for this client.

After receiving the hand-off report, which client should the oncology nurse see first? a. Client who is afebrile with a heart rate of 108 beats/min b. Older client on chemotherapy with mental status changes c. Client who is neutropenic and in protective isolation d. Client scheduled for radiation therapy today

ANS: B Older clients often do not exhibit classic signs of infection, and often mental status changes are the first observation. Clients on chemotherapy who become neutropenic also often do not exhibit classic signs of infection. The nurse should assess the older client first. The other clients can be seen afterward.

A client has received a dose of ondansetron (Zofran) for nausea. What action by the nurse is most important? a. Assess the client for a headache. b. Assist the client in getting out of bed. c. Instruct the client to reduce salt intake. d. Weigh the client daily before the client eats.

ANS: B Ondansetron side effects include postural hypotension, vertigo, and bradycardia, all of which increase the client's risk for injury. The nurse should assist the client when getting out of bed. Headache and fluid retention are not side effects of this drug.

A nurse assesses a client and identifies that the client has pallor conjunctivae. Which focused assessment should the nurse complete next? a. Partial thromboplastin time b. Hemoglobin and hematocrit c. Liver enzymes d. Basic metabolic panel

ANS: B Pallor conjunctivae signifies anemia. The nurse should assess the client's hemoglobin and hematocrit to confirm anemia. The other laboratory results do not relate to this client's potential anemia.

A client has a pyloric obstruction and reports sudden muscle weakness. What action by the nurse takes priority? a. Document the findings in the chart. b. Request an electrocardiogram (ECG). c. Facilitate a serum potassium test. d. Place the client on bedrest.

ANS: B Pyloric stenosis can lead to hypokalemia, which is manifested by muscle weakness. The nurse first obtains an ECG because potassium imbalances can lead to cardiac dysrhythmias. A potassium level is also warranted, as is placing the client on bedrest for safety. Documentation should be thorough, but none of these actions takes priority over the ECG.

A client in the oncology clinic reports her family is frustrated at her ongoing fatigue 4 months after radiation therapy for breast cancer. What response by the nurse is most appropriate? a. "Are you getting adequate rest and sleep each day?" b. "It is normal to be fatigued even for years afterward." c. "This is not normal and I'll let the provider know." d. "Try adding more vitamins B and C to your diet."

ANS: B Regardless of the cause, radiation-induced fatigue can be debilitating and may last for months or years after treatment has ended. Rest and adequate nutrition can affect fatigue, but it is most important that the client understands this is normal.

The nurse knows that a client with prolonged prothrombin time (PT) values (not related to medication) probably has dysfunction in which organ? a. Kidneys b. Liver c. Spleen d. Stomach

ANS: B Severe acute or chronic liver damage leads to a prolonged PT secondary to impaired synthesis of clotting proteins. The other organs are not related to this issue.

A nurse has conducted a community screening event for oral cancer. What client is the highest priority for referral to a dentist? a. Client who has poor oral hygiene practices b. Client who smokes and drinks daily c. Client who tans for an upcoming vacation d. Client who occasionally uses illicit drugs

ANS: B Smoking and alcohol exposure create a high risk for this client. Poor oral hygiene is not related to the etiology of cancer but may cause a tumor to go unnoticed. Tanning is a risk factor, but short-term exposure does not have the same risk as daily exposure to tobacco and alcohol. Illicit drugs are not related to oral cancers.

A client has a great deal of pain when coughing and deep breathing after abdominal surgery despite having pain medication. What action by the nurse is best? a. Call the provider to request more analgesia. b. Demonstrate how to splint the incision. c. Have the client take shallower breaths. d. Tell the client a little pain is expected.

ANS: B Splinting an incision provides extra support during coughing and activity and helps decrease pain. If the client is otherwise comfortable, no more analgesia is required. Shallow breathing can lead to atelectasis and pneumonia. The client should know some pain is normal and expected after surgery, but that answer alone does not provide any interventions to help the client.

A student nurse is preparing morning medications for a client who had a stroke. The student plans to hold the docusate sodium (Colace) because the client had a large stool earlier. What action by the supervising nurse is best? a. Have the student ask the client if it is desired or not. b. Inform the student that the docusate should be given. c. Tell the student to document the rationale. d. Tell the student to give it unless the client refuses.

ANS: B Stool softeners should be given to clients with neurologic disorders in order to prevent an elevation in intracranial pressure that accompanies the Valsalva maneuver when constipated. The supervising nurse should instruct the student to administer the docusate. The other options are not appropriate. The medication could be held for diarrhea.

A nurse plans care for a client with acute pancreatitis. Which intervention should the nurse include in this client's plan of care to reduce discomfort? a. Administer morphine sulfate intravenously every 4 hours as needed. b. Maintain nothing by mouth (NPO) and administer intravenous fluids. c. Provide small, frequent feedings with no concentrated sweets. d. Place the client in semi-Fowler's position with the head of bed elevated.

ANS: B The client should be kept NPO to reduce GI activity and reduce pancreatic enzyme production. IV fluids should be used to prevent dehydration. The client may need a nasogastric tube. Pain medications should be given around the clock and more frequently than every 4 to 6 hours. A fetal position with legs drawn up to the chest will promote comfort.

A nurse is teaching a client with multiple sclerosis who is prescribed cyclophosphamide (Cytoxan) and methylprednisolone (Medrol). Which statement should the nurse include in this client's discharge teaching? a. "Take warm baths to promote muscle relaxation." b. "Avoid crowds and people with colds." c. "Relying on a walker will weaken your gait." d. "Take prescribed medications when symptoms occur."

ANS: B The client should be taught to avoid people with any type of upper respiratory illness because these medications are immunosuppressive. Warm baths will exacerbate the client's symptoms. Assistive devices may be required for safe ambulation. Medication should be taken at all times and should not be stopped.

A nurse is caring for four clients. After receiving the hand-off report, which client should the nurse see first? a. Client having a radial neck dissection tomorrow who is asking questions b. Client who had a tracheostomy 4 hours ago and needs frequent suctioning c. Client who is 1 day postoperative for an oral tumor resection who is reporting pain d. Client waiting for discharge instructions after a small tumor resection

ANS: B The client who needs frequent suctioning should be seen first to ensure that his or her airway is patent. The client waiting for pain medication should be seen next. The nurse may need to call the surgeon to see the client who is asking questions. The client waiting for discharge instructions can be seen last.

A nurse is giving a client instructions for showering with special antimicrobial soap the night before surgery. What instruction is most appropriate? a. "After you wash the surgical site, shave that area with your own razor." b. "Be sure to wash the area where you will have surgery very thoroughly." c. "Use a washcloth to wash the surgical site; do not take a full shower or bath." d. "Wash the surgical site first, then shampoo and wash the rest of your body."

ANS: B The entire proposed surgical site needs to be washed thoroughly and completely with the antimicrobial soap. Shaving, if absolutely necessary, should be done in the operative suite immediately before the operation begins, using sterile equipment. The client needs a full shower or bath (shower preferred). The client should wash the surgical site last; dirty water from shampooing will run over the cleansed site if the site is washed first.

A nurse witnesses a client begin to experience a tonic-clonic seizure and loss of consciousness. Which action should the nurse take? a. Start fluids via a large-bore catheter. b. Turn the client's head to the side. c. Administer IV push diazepam. d. Prepare to intubate the client.

ANS: B The nurse should turn the client's head to the side to prevent aspiration and allow drainage of secretions. Anticonvulsants are administered on a routine basis if a seizure is sustained. If the seizure is sustained (status epilepticus), the client must be intubated and should be administered oxygen, 0.9% sodium chloride, and IV push lorazepam or diazepam.

A client is recovering from an esophagogastroduodenoscopy (EGD) and requests something to drink. What action by the nurse is best? a. Allow the client cool liquids only. b. Assess the client's gag reflex. c. Remind the client to remain NPO. d. Tell the client to wait 4 hours.

ANS: B The local anesthetic used during this procedure will depress the client's gag reflex. After the procedure, the nurse should ensure that the gag reflex is intact before offering food or fluids. The client does not need to be restricted to cool beverages only and is not required to wait 4 hours before oral intake is allowed. Telling the client to remain NPO does not inform the client of when he or she can have fluids, nor does it reflect the client's readiness for them

A client has a recurrence of gastric cancer and is in the gastrointestinal clinic crying. What response by the nurse is most appropriate? a. "Do you have family or friends for support?" b. "I'd like to know what you are feeling now." c. "Well, we knew this would probably happen." d. "Would you like me to refer you to hospice?"

ANS: B The nurse assesses the client's emotional state with open-ended questions and statements and shows a willingness to listen to the client's concerns. Asking about support people is very limited in nature, and "yes-or-no" questions are not therapeutic. Stating that this was expected dismisses the client's concerns. The client may or may not be ready to hear about hospice, and this is another limited, yes-or-no question.

A nurse cares for a client who is prescribed patient-controlled analgesia (PCA) after a cholecystectomy. The client states, "When I wake up I am in pain." Which action should the nurse take? a. Administer intravenous morphine while the client sleeps. b. Encourage the client to use the PCA pump upon awakening. c. Contact the provider and request a different analgesic. d. Ask a family member to initiate the PCA pump for the client

ANS: B The nurse should encourage the client to use the PCA pump prior to napping and upon awakening. Administering additional intravenous morphine while the client sleeps places the client at risk for respiratory depression. The nurse should also evaluate dosages received compared with dosages requested and contact the provider if the dose or frequency is not adequate. Only the client should push the pain button on a PCA pump.

A client is scheduled to have a hip replacement. Preoperatively, the client is found to be mildly anemic and the surgeon states the client may need a blood transfusion during or after the surgery. What action by the preoperative nurse is most important? a. Administer preoperative medications as prescribed. b. Ensure that a consent for transfusion is on the chart. c. Explain to the client how anemia affects healing. d. Teach the client about foods high in protein and iron.

ANS: B The preoperative nurse should ensure that all valid consents are on the chart, including one for blood transfusions if this may be needed. Administering preoperative medications is important for all preoperative clients and is not specific to this client. Teaching in the preoperative area should focus on immediate concerns.

ANS: B The entire proposed surgical site needs to be washed thoroughly and completely with the antimicrobial soap. Shaving, if absolutely necessary, should be done in the operative suite immediately before the operation begins, using sterile equipment. The client needs a full shower or bath (shower preferred). The client should wash the surgical site last; dirty water from shampooing will run over the cleansed site if the site is washed first.

ANS: B The priority client problem related to a surgical drain is the potential for infection. An insertion site that is free of redness, warmth, and drainage indicates that goals for this client problem are being met. The other assessments are normal, but not related to the drain.

After teaching a client who has been diagnosed with hepatitis A, the nurse assesses the client's understanding. Which statement by the client indicates a correct understanding of the teaching? a. "Some medications have been known to cause hepatitis A." b. "I may have been exposed when we ate shrimp last weekend." c. "I was infected with hepatitis A through a recent blood transfusion." d. "My infection with Epstein-Barr virus can co-infect me with hepatitis A."

ANS: B The route of acquisition of hepatitis A infection is through close personal contact or ingestion of contaminated water or shellfish. Hepatitis A is not transmitted through medications, blood transfusions, or Epstein-Barr virus. Toxic and drug-induced hepatitis is caused from exposure to hepatotoxins, but this is not a form of hepatitis A. Hepatitis B can be spread through blood transfusions. Epstein-Barr virus causes a secondary infection that is not associated with hepatitis A.

A client tells the oncology nurse about an upcoming vacation to the beach to celebrate completing radiation treatments for cancer. What response by the nurse is most appropriate? a. "Avoid getting salt water on the radiation site." b. "Do not expose the radiation area to direct sunlight." c. "Have a wonderful time and enjoy your vacation!" d. "Remember you should not drink alcohol for a year."

ANS: B The skin overlying the radiation site is extremely sensitive to sunlight after radiation therapy has been completed. The nurse should inform the client to avoid sun exposure to this area. This advice continues for 1 year after treatment has been completed. The other statements are not appropriate.

A nurse is giving a preoperative client a dose of ranitidine (Zantac). The client asks why the nurse is giving this drug when the client has no history of ulcers. What response by the nurse is best? a. "All preoperative clients get this medication." b. "It helps prevent ulcers from the stress of the surgery." c. "Since you don't have ulcers, I will have to ask." d. "The physician prescribed this medication for you."

ANS: B Ulcer prophylaxis is common for clients undergoing long procedures or for whom high stress is likely. The nurse is not being truthful by saying all clients get this medication. If the nurse does not know the information, it is appropriate to find out, but this is a common medication for which the nurse should know the rationale prior to administering it. Simply stating that the physician prescribed the medication does not give the client any useful information.

A student nurse is providing care to an older client with stomatitis and dysphagia. What action by the student nurse requires the registered nurse to intervene? a. Assisting the client to perform oral care every 2 hours b. Preparing to administer a viscous lidocaine gargle c. Reminding the client not to swallow nystatin (Mycostatin) d. Teaching the client to use a soft-bristled toothbrush

ANS: B Viscous lidocaine gargles or mouthwashes are sometimes prescribed for clients with stomatitis and pain. However, the numbing effect can lead to choking or mouth burns from hot food. This client already has difficulty swallowing, so this medication is not appropriate. Therefore, the nurse should intervene when the student prepares to administer this preparation. The other options are correct actions.

The nurse working in the rheumatology clinic assesses clients with rheumatoid arthritis (RA) for late manifestations. Which signs/symptoms are considered late manifestations of RA? (Select all that apply.) a. Anorexia b. Felty's syndrome c. Joint deformity d. Low-grade fever e. Weight loss

ANS: B, C, E Late manifestations of RA include Felty's syndrome, joint deformity, weight loss, organ involvement, osteoporosis, extreme fatigue, and anemia, among others. Anorexia and low-grade fever are both seen early in the course of the disease.

A nurse teaches a client with type 2 diabetes mellitus who is prescribed glipizide (Glucotrol). Which statement should the nurse include in this client's teaching? a. "Change positions slowly when you get out of bed." b. "Avoid taking nonsteroidal anti-inflammatory drugs (NSAIDs)." c. "If you miss a dose of this drug, you can double the next dose." d. "Discontinue the medication if you develop a urinary infection."

ANS: B NSAIDs potentiate the hypoglycemic effects of sulfonylurea agents. Glipizide is a sulfonylurea. The other statements are not applicable to glipizide.

A nurse assesses a client who has liver disease. Which laboratory findings should the nurse recognize as potentially causing complications of this disorder? (Select all that apply.) a. Elevated aspartate transaminase b. Elevated international normalized ratio (INR) c. Decreased serum globulin levels d. Decreased serum alkaline phosphatase e. Elevated serum ammonia f. Elevated prothrombin time (PT)

ANS: B, E, F Elevated INR and PT are indications of clotting disturbances and alert the nurse to the increased possibility of hemorrhage. Elevated ammonia levels increase the client's confusion. The other values are abnormal and associated with liver disease but do not necessarily place the client at increased risk for complications.

When transferring a client into a chair, a nurse notices that the pressure-relieving mattress overlay has deep imprints of the client's buttocks, heels, and scapulae. Which action should the nurse take next? a.Turn the mattress overlay to the opposite side. b.Do nothing because this is an expected occurrence. c.Apply a different pressure-relieving device. d.Reinforce the overlay with extra cushions.

ANS: C "Bottoming out," as evidenced by deep imprints in the mattress overlay, indicates that this device is not appropriate for this client, and a different device or strategy should be implemented to prevent pressure ulcer formation.

A nurse assesses a young female client who is prescribed isotretinoin (Accutane). Which question should the nurse ask prior to starting this therapy? a."Do you spend a great deal of time in the sun?" b."Have you or any family members ever had skin cancer?" c."Which method of contraception are you using?" d."Do you drink alcoholic beverages?"

ANS: C Isotretinoin has many side effects. It is a known teratogen and can cause severe birth defects. A pregnancy test is required before therapy is initiated, and strict birth control measures must be used during therapy. Sun exposure, alcohol ingestion, and family history of cancer are contraindications for isotretinoin.

A client is 1 day postoperative after having Zenker's diverticula removed. The client has a nasogastric (NG) tube to suction, and for the last 4 hours there has been no drainage. There are no specific care orders for the NG tube in place. What action by the nurse is most appropriate? a. Document the findings as normal. b. Irrigate the NG tube with sterile saline. c. Notify the surgeon about this finding. d. Remove and reinsert the NG tube.

ANS: C NG tubes placed during surgery should not be irrigated or moved unless prescribed by the surgeon. The nurse should notify the surgeon about this finding. Documentation is important, but this finding is not normal.

A client has gastroesophageal reflux disease (GERD). The provider prescribes a proton pump inhibitor. About what medication should the nurse anticipate teaching the client? a. Famotidine (Pepcid) b. Magnesium hydroxide (Maalox) c. Omeprazole (Prilosec) d. Ranitidine (Zantac)

ANS: C Omeprazole is a proton pump inhibitor used in the treatment of GERD. Famotidine and ranitidine are histamine blockers. Maalox is an antacid.

A client has a nasogastric (NG) tube. What action by the nursing student requires the registered nurse to intervene? a.Checking tube placement every 4 to 8 hours b.Monitoring and documenting drainage from the NG tube c.Pinning the tube to the gown so the client cannot turn the head d.Providing oral care every 4 to 8 hours

ANS: C The client should be able to turn his or her head to prevent pulling the tube out with movement. The other actions are appropriate.

After teaching a client who is newly diagnosed with type 2 diabetes mellitus, the nurse assesses the client's understanding. Which statement made by the client indicates a need for additional teaching? a. "I should increase my intake of vegetables with higher amounts of dietary fiber." b. "My intake of saturated fats should be no more than 10% of my total calorie intake." c. "I should decrease my intake of protein and eliminate carbohydrates from my diet." d. "My intake of water is not restricted by my treatment plan or medication regimen."

ANS: C The client should not completely eliminate carbohydrates from the diet, and should reduce protein if microalbuminuria is present. The client should increase dietary intake of complex carbohydrates, including vegetables, and decrease intake of fat. Water does not need to be restricted unless kidney failure is present.

A nurse assesses a client with diabetes mellitus 3 hours after a surgical procedure and notes the client's breath has a "fruity" odor. Which action should the nurse take? a. Encourage the client to use an incentive spirometer. b. Increase the client's intravenous fluid flow rate. c. Consult the provider to test for ketoacidosis. d. Perform meticulous pulmonary hygiene care.

ANS: C The stress of surgery increases the action of counterregulatory hormones and suppresses the action of insulin, predisposing the client to ketoacidosis and metabolic acidosis. One manifestation of ketoacidosis is a "fruity" odor to the breath. Documentation should occur after all assessments have been completed. Using an incentive spirometer, increasing IV fluids, and performing pulmonary hygiene will not address this client's problem.

A nurse prepares a client for a colonoscopy scheduled for tomorrow. The client states, "My doctor told me that the fecal occult blood test was negative for colon cancer. I don't think I need the colonoscopy and would like to cancel it." How should the nurse respond? a. "Your doctor should not have given you that information prior to the colonoscopy." b. "The colonoscopy is required due to the high percentage of false negatives with the blood test." c. "A negative fecal occult blood test does not rule out the possibility of colon cancer." d. "I will contact your doctor so that you can discuss your concerns about the procedure."

ANS: C A negative result from a fecal occult blood test does not completely rule out the possibility of colon cancer. To determine whether the client has colon cancer, a colonoscopy should be performed so the entire colon can be visualized and a tissue sample taken for biopsy. The client may want to speak with the provider, but the nurse should address the client's concerns prior to contacting the provider.

A preoperative nurse is reviewing morning laboratory values on four clients waiting for surgery. Which result warrants immediate communication with the surgical team? a. Creatinine: 1.2 mg/dL b. Hemoglobin: 14.8 mg/dL c. Potassium: 2.9 mEq/L d. Sodium: 134 mEq/L

ANS: C A potassium of 2.9 mEq/L is critically low and can affect cardiac and respiratory status. The nurse should communicate this laboratory value immediately. The creatinine is at the high end of normal, the hemoglobin is normal, and the sodium is only slightly low (normal low being 136 mEq/L), so these values do not need to be reported immediately.

A client with a bleeding gastric ulcer is having a nuclear medicine scan. What action by the nurse is most appropriate? a. Assess the client for iodine or shellfish allergies. b. Educate the client on the side effects of sedation. c. Inform the client a second scan may be needed. d. Teach the client about bowel preparation for the scan.

ANS: C A second scan may be performed in 1 to 2 days to see if interventions have worked. The nuclear medicine scan does not use iodine-containing contrast dye or sedation. There is no required bowel preparation.

A client with Guillain-Barré syndrome is admitted to the hospital. The nurse plans caregiving priority to interventions that address which priority client problem? a. Anxiety b. Low fluid volume c. Inadequate airway d. Potential for skin breakdown

ANS: C Airway takes priority. Anxiety is probably present, but a physical diagnosis takes priority over a psychosocial one. The client has no reason to have low fluid volume unless he or she has been unable to drink for some time. If present, airway problems take priority over a circulation problem. An actual problem takes precedence over a risk for a problem.

A client in the preoperative holding room has received sedation and now needs to urinate. What action by the nurse is best? a. Allow the client to walk to the bathroom. b. Delegate assisting the client to the nurse's aide. c. Give the client a bedpan or urinal to use. d. Insert a urinary catheter now instead of waiting.

ANS: C Although possibly uncomfortable or embarrassing for the client, the client should not be allowed out of bed after receiving sedation. The nurse should get the client a bedpan or urinal. The client may or may not need a urinary catheter.

A nurse is monitoring a client after moderate sedation. The nurse documents the client's Ramsay Sedation Scale (RSS) score at 3. What action by the nurse is best? a. Assess the client's gag reflex. b. Begin providing discharge instructions. c. Document findings and continue to monitor. d. Increase oxygen and notify the provider.

ANS: C An RSS score of 3 means the client is able to respond quickly, but only to commands. The client has not had enough time to fully arouse. The nurse should document the findings and continue to monitor per agency policy. If the client had an oral endoscopy or was intubated, checking the gag reflex would be appropriate prior to permitting eating or drinking. The client is not yet awake enough for teaching. There is no need to increase oxygen and notify the provider.

A nurse assesses a client with Alzheimer's disease who is recently admitted to the hospital. Which psychosocial assessment should the nurse complete? a. Assess religious and spiritual needs while in the hospital. b. Identify the client's ability to perform self-care activities. c. Evaluate the client's reaction to a change of environment. d. Ask the client about relationships with family members.

ANS: C As Alzheimer's disease progresses, the client experiences changes in emotional and behavioral affect. The nurse should be alert to the client's reaction to a change in environment, such as being hospitalized, because the client may exhibit an exaggerated response, such as aggression, to the event. The other assessments should be completed but are not as important as assessing the client's reaction to environmental change.

A client has a continuous passive motion (CPM) device after a total knee replacement. What action does the nurse delegate to the unlicensed assistive personnel (UAP) after the affected leg is placed in the machine while the client is in bed? a. Assess the distal circulation in 30 minutes. b. Change the settings based on range of motion. c. Raise the lower siderail on the affected side. d. Remind the client to do quad-setting exercises.

ANS: C Because the client's leg is strapped into the CPM, if it falls off the bed due to movement, the client's leg (and new joint) can be injured. The nurse should instruct the UAP to raise the siderail to prevent this from occurring. Assessment is a nursing responsibility. Only the surgeon, physical therapist, or specially trained technician adjusts the CPM settings. Quad-setting exercises are not related to the CPM machine.

A nurse prepares to assess the emotional state of a client with end-stage pancreatic cancer. Which action should the nurse take first? a. Bring the client to a quiet room for privacy. b. Pull up a chair and sit next to the client's bed. c. Determine whether the client feels like talking about his or her feelings. d. Review the health care provider's notes about the prognosis for the client.

ANS: C Before conducting an assessment about the client's feelings, the nurse should determine whether he or she is willing and able to talk about them. If the client is open to the conversation and his or her room is not appropriate, an alternative meeting space may be located. The nurse should be present for the client during this time, and pulling up a chair and sitting with the client indicates that presence. Because the nurse is assessing the client's response to a terminal diagnosis, it is not necessary to have detailed information about the projected prognosis; the nurse knows that the client is facing an end-of-life illness.

The nursing instructor explains the difference between normal cells and benign tumor cells. What information does the instructor provide about these cells? a. Benign tumors grow through invasion of other tissue. b. Benign tumors have lost their cellular regulation from contact inhibition. c. Growing in the wrong place or time is typical of benign tumors. d. The loss of characteristics of the parent cells is called anaplasia.

ANS: C Benign tumors are basically normal cells growing in the wrong place or at the wrong time. Benign cells grow through hyperplasia, not invasion. Benign tumor cells retain contact inhibition. Anaplasia is a characteristic of cancer cells.

A nurse delegates care for a client with early-stage Alzheimer's disease to an unlicensed assistive personnel (UAP). Which statement should the nurse include when delegating this client's care? a. "If she is confused, play along and pretend that everything is okay." b. "Remove the clock from her room so that she doesn't get confused." c. "Reorient the client to the day, time, and environment with each contact." d. "Use validation therapy to recognize and acknowledge the client's concerns."

ANS: C Clients who have early-stage Alzheimer's disease should be reoriented frequently to person, place, and time. The UAP should reorient the client and not encourage the client's delusions. The room should have a clock and white board with the current date written on it. Validation therapy is used with late-stage Alzheimer's disease.

A client is admitted with Guillain-Barré syndrome (GBS). What assessment takes priority? a. Bladder control b. Cognitive perception c. Respiratory system d. Sensory functions

ANS: C Clients with GBS have muscle weakness, possibly to the point of paralysis. If respiratory muscles are paralyzed, the client may need mechanical ventilation, so the respiratory system is the priority. The nurse will complete urinary, cognitive, and sensory assessments as part of a thorough evaluation.

A client has newly diagnosed systemic lupus erythematosus (SLE). What instruction by the nurse is most important? a. "Be sure you get enough sleep at night." b. "Eat plenty of high-protein, high-iron foods." c. "Notify your provider at once if you get a fever." d. "Weigh yourself every day on the same scale."

ANS: C Fever is the classic sign of a lupus flare and should be reported immediately. Rest and nutrition are important but do not take priority over teaching the client what to do if he or she develops an elevated temperature. Daily weights may or may not be important depending on renal involvement.

A nurse assesses a client with multiple sclerosis after administering prescribed fingolimod (Gilenya). For which adverse effect should the nurse monitor? a. Peripheral edema b. Black tarry stools c. Bradycardia d. Nausea and vomiting

ANS: C Fingolimod (Gilenya) is an antineoplastic agent that can cause bradycardia, especially within the first 6 hours after administration. Peripheral edema, black and tarry stools, and nausea and vomiting are not adverse effects of fingolimod.

A client is having an esophagogastroduodenoscopy (EGD) and has been given midazolam hydrochloride (Versed). The client's respiratory rate is 8 breaths/min. What action by the nurse is best? a. Administer naloxone (Narcan). b. Call the Rapid Response Team. c. Provide physical stimulation. d. Ventilate with a bag-valve-mask.

ANS: C For an EGD, clients are given mild sedation but should still be able to follow commands. For shallow or slow respirations after the sedation is given, the nurse's first action is to provide a physical stimulation such as a sternal rub and directions to breathe deeply. Naloxone is not the antidote for Versed. The Rapid Response Team is not needed at this point. The client does not need manual ventilation.

A nurse cares for a client who has hypothyroidism as a result of Hashimoto's thyroiditis. The client asks, "How long will I need to take this thyroid medication?" How should the nurse respond? a. "You will need to take the thyroid medication until the goiter is completely gone." b. "Thyroiditis is cured with antibiotics. Then you won't need thyroid medication." c. "You'll need thyroid pills for life because your thyroid won't start working again." d. "When blood tests indicate normal thyroid function, you can stop the medication."

ANS: C Hashimoto's thyroiditis results in a permanent loss of thyroid function. The client will need lifelong thyroid replacement therapy. The client will not be able to stop taking the medication.

A nurse plans care for a client with lower back pain from a work-related injury. Which intervention should the nurse include in this client's plan of care? a. Encourage the client to stretch the back by reaching toward the toes. b. Massage the affected area with ice twice a day. c. Apply a heating pad for 20 minutes at least four times daily. d. Advise the client to avoid warm baths or showers.

ANS: C Heat increases blood flow to the affected area and promotes healing of injured nerves. Stretching and ice will not promote healing, and there is no need to avoid warm baths or showers.

A nurse plans care for a client with hypothyroidism. Which priority problem should the nurse plan to address first for this client? a. Heat intolerance b. Body image problems c. Depression and withdrawal d. Obesity and water retention

ANS: C Hypothyroidism causes many problems in psychosocial functioning. Depression is the most common reason for seeking medical attention. Memory and attention span may be impaired. The client's family may have great difficulty accepting and dealing with these changes. The client is often unmotivated to participate in self-care. Lapses in memory and attention require the nurse to ensure that the client's environment is safe. Heat intolerance is seen in hyperthyroidism. Body image problems and weight issues do not take priority over mental status and safety.

A client experiences impaired swallowing after a stroke and has worked with speech-language pathology on eating. What nursing assessment best indicates that a priority goal for this problem has been met? a. Chooses preferred items from the menu b. Eats 75% to 100% of all meals and snacks c. Has clear lung sounds on auscultation d. Gains 2 pounds after 1 week

ANS: C Impaired swallowing can lead to aspiration, so the priority goal for this problem is no aspiration. Clear lung sounds is the best indicator that aspiration has not occurred. Choosing menu items is not related to this problem. Eating meals does not indicate the client is not still aspirating. A weight gain indicates improved nutrition but still does not show a lack of aspiration.

A nurse assesses a client who has cholecystitis. Which clinical manifestation indicates that the condition is chronic rather than acute? a. Temperature of 100.1° F (37.8° C) b. Positive Murphy's sign c. Light-colored stools d. Upper abdominal pain after eating

ANS: C Jaundice, clay-colored stools, and dark urine are more commonly seen with chronic cholecystitis. The other symptoms are seen equally with both chronic and acute cholecystitis.

A nurse assesses clients at a community health center. Which client is at highest risk for pancreatic cancer? a. A 32-year-old with hypothyroidism b. A 44-year-old with cholelithiasis c. A 50-year-old who has the BRCA2 gene mutation d. A 68-year-old who is of African-American ethnicity

ANS: C Mutations in both the BRCA2 and p16 genes increase the risk for developing pancreatic cancer in a small number of cases. The other factors do not appear to be linked to increased risk.

A nurse assesses a client who is recovering from a subtotal thyroidectomy. On the second postoperative day the client states, "I feel numbness and tingling around my mouth." What action should the nurse take? a. Offer mouth care. b. Loosen the dressing. c. Assess for Chvostek's sign. d. Ask the client orientation questions

ANS: C Numbness and tingling around the mouth or in the fingers and toes are manifestations of hypocalcemia, which could progress to cause tetany and seizure activity. The nurse should assess the client further by testing for Chvostek's sign and Trousseau's sign. Then the nurse should notify the provider. Mouth care, loosening the dressing, and orientation questions do not provide important information to prevent complications of low calcium levels.

A nurse is working with a community group promoting healthy aging. What recommendation is best to help prevent osteoarthritis (OA)? a. Avoid contact sports. b. Get plenty of calcium. c. Lose weight if needed. d. Engage in weight-bearing exercise.

ANS: C Obesity can lead to OA, and if the client is overweight, losing weight can help prevent OA or reduce symptoms once it occurs. Arthritis can be caused by contact sports, but this is less common than obesity. Calcium and weight-bearing exercise are both important for osteoporosis.

A nurse assesses clients at a community center. Which client is at greatest risk for lower back pain? a. A 24-year-old female who is 25 weeks pregnant b. A 36-year-old male who uses ergonomic techniques c. A 45-year-old male with osteoarthritis d. A 53-year-old female who uses a walker

ANS: C Osteoarthritis causes changes to support structures, increasing the client's risk for low back pain. The other clients are not at high risk.

The nurse working in the orthopedic clinic knows that a client with which factor has an absolute contraindication for having a total joint replacement? a. Needs multiple dental fillings b. Over age 85 c. Severe osteoporosis d. Urinary tract infection

ANS: C Osteoporosis is a contraindication to joint replacement because the bones have a high risk of shattering as the new prosthesis is implanted. The client who needs fillings should have them done prior to the surgery. Age greater than 85 is not an absolute contraindication. A urinary tract infection can be treated prior to surgery.

After teaching a client who expressed concern about a rash located beneath her breast, a nurse assesses the client's understanding. Which statement indicates the client has a good understanding of this condition? a. "This rash is probably due to fluid overload." b. "I need to wash this daily with antibacterial soap." c. "I can use powder to keep this area dry." d. "I will schedule a mammogram as soon as I can."

ANS: C Rashes limited to skinfold areas (e.g., on the axillae, beneath the breasts, in the groin) may reflect problems related to excessive moisture. The client needs to keep the area dry; one option is to use powder. Good hygiene is important, but the rash does not need an antibacterial soap. Fluid overload and breast cancer are not related to rashes in skinfolds.

A client who collapsed during dinner in a restaurant arrives in the emergency department. The client is going to surgery to repair an abdominal aortic aneurysm. What medication does the nurse prepare to administer as a priority for this client? a. Hydroxyzine (Atarax) b. Lorazepam (Ativan) c. Metoclopramide (Reglan) d. Morphine sulfate

ANS: C Reglan increases gastric emptying, an important issue for this client who was eating just prior to the operation. The other drugs are appropriate for any surgical client.

A client is receiving rituximab (Rituxan) and asks how it works. What response by the nurse is best? a. "It causes rapid lysis of the cancer cell membranes." b. "It destroys the enzymes needed to create cancer cells." c. "It prevents the start of cell division in the cancer cells." d. "It sensitizes certain cancer cells to chemotherapy."

ANS: C Rituxan prevents the initiation of cancer cell division. The other statements are not accurate.

A client has undergone a percutaneous stereotactic rhizotomy. What instruction by the nurse is most important on discharge from the ambulatory surgical center? a. "Avoid having teeth pulled for 1 year." b. "Brush your teeth with a soft toothbrush." c. "Do not use harsh chemicals on your face." d. "Inform your dentist of this procedure."

ANS: C The affected side is left without sensation after this procedure. The client should avoid putting harsh chemicals on the face because he or she will not feel burning or stinging on that side. This will help avoid injury. The other instructions are not necessary.

The circulating nurse is plugging in a piece of equipment and notes that the cord is frayed. What action by the nurse is best? a. Call maintenance for repair. b. Check the machine before using. c. Get another piece of equipment. d. Notify the charge nurse.

ANS: C The circulating nurse is responsible for client safety. If an electrical cord is frayed, the risk of fire or sparking increases. The nurse should obtain a replacement. The nurse should also tag the original equipment for repair as per agency policy. Checking the equipment is not important as the nurse should not even attempt to use it. Calling maintenance or requesting maintenance per facility protocol is important, but first ensure client safety by having a properly working piece of equipment for the procedure about to take place. The charge nurse probably does need to know of the need for equipment repair, but ensuring client safety is the priority.

A nurse assesses a client with a spinal cord injury at level T5. The client's blood pressure is 184/95 mm Hg, and the client presents with a flushed face and blurred vision. Which action should the nurse take first? a. Initiate oxygen via a nasal cannula. b. Place the client in a supine position. c. Palpate the bladder for distention. d. Administer a prescribed beta blocker

ANS: C The client is manifesting symptoms of autonomic dysreflexia. Common causes include bladder distention, tight clothing, increased room temperature, and fecal impaction. If persistent, the client could experience neurologic injury. Precipitating conditions should be eliminated and the physician notified. The other actions would not be appropriate.

A nurse is caring for four clients in the neurologic/neurosurgical intensive care unit. Which client should the nurse assess first? a. Client who has been diagnosed with meningitis with a fever of 101° F (38.3° C) b. Client who had a transient ischemic attack and is waiting for teaching on clopidogrel (Plavix) c. Client receiving tissue plasminogen activator (t-PA) who has a change in respiratory pattern and rate d. Client who is waiting for subarachnoid bolt insertion with the consent form already signed

ANS: C The client receiving t-PA has a change in neurologic status while receiving this fibrinolytic therapy. The nurse assesses this client first as he or she may have an intracerebral bleed. The client with meningitis has expected manifestations. The client waiting for discharge teaching is a lower priority. The client waiting for surgery can be assessed quickly after the nurse sees the client who is receiving t-PA, or the nurse could delegate checking on this client to another nurse.

A nurse teaches a client who is at risk for colon cancer. Which dietary recommendation should the nurse teach this client? a. "Eat low-fiber and low-residual foods." b. "White rice and bread are easier to digest." c. "Add vegetables such as broccoli and cauliflower to your new diet." d. "Foods high in animal fat help to protect the intestinal mucosa."

ANS: C The client should be taught to modify his or her diet to decrease animal fat and refined carbohydrates. The client should also increase high-fiber foods and Brassica vegetables, including broccoli and cauliflower, which help to protect the intestinal mucosa from colon cancer.

After teaching a client who is prescribed pancreatic enzyme replacement therapy, the nurse assesses the client's understanding. Which statement made by the client indicates a need for additional teaching? a. "The capsules can be opened and the powder sprinkled on applesauce if needed." b. "I will wipe my lips carefully after I drink the enzyme preparation." c. "The best time to take the enzymes is immediately after I have a meal or a snack." d. "I will not mix the enzyme powder with food or liquids that contain protein."

ANS: C The enzymes should be taken immediately before eating meals or snacks. If the client cannot swallow the capsules whole, they can be opened up and the powder sprinkled on applesauce, mashed fruit, or rice cereal. The client should wipe his or her lips carefully after drinking the enzyme preparation because the liquid could damage the skin. Protein items will be dissolved by the enzymes if they are mixed together.

A nurse cares for a client with a spinal cord injury. With which interdisciplinary team member should the nurse consult to assist the client with activities of daily living? a. Social worker b. Physical therapist c. Occupational therapist d. Case manager

ANS: C The occupational therapist instructs the client in the correct use of all adaptive equipment. In collaboration with the therapist, the nurse instructs family members or the caregiver about transfer skills, feeding, bathing, dressing, positioning, and skin care. The other team members are consulted to assist the client with unrelated issues.

A nurse works on the postoperative floor and has four clients who are being discharged tomorrow. Which one has the greatest need for the nurse to consult other members of the health care team for post-discharge care? a. Married young adult who is the primary caregiver for children b. Middle-aged client who is post knee replacement, needs physical therapy c. Older adult who lives at home despite some memory loss d. Young client who lives alone, has family and friends nearby

ANS: C The older adult has the most potentially complex discharge needs. With memory loss, the client may not be able to follow the prescribed home regimen. The client's physical abilities may be limited by chronic illness. This client has several safety needs that should be assessed. The other clients all have evidence of a support system and no known potential for serious safety issues.

A client has a traumatic brain injury and a positive halo sign. The client is in the intensive care unit, sedated and on a ventilator, and is in critical but stable condition. What collaborative problem takes priority at this time? a. Inability to communicate b. Nutritional deficit c. Risk for acquiring an infection d. Risk for skin breakdown

ANS: C The positive halo sign indicates a leak of cerebrospinal fluid. This places the client at high risk of acquiring an infection. Communication and nutrition are not priorities compared with preventing a brain infection. The client has a definite risk for a skin breakdown, but it is not the immediate danger a brain infection would be.

A nurse assessing a client with colorectal cancer auscultates high-pitched bowel sounds and notes the presence of visible peristaltic waves. Which action should the nurse take? a. Ask if the client is experiencing pain in the right shoulder. b. Perform a rectal examination and assess for polyps. c. Contact the provider and recommend computed tomography. d. Administer a laxative to increase bowel movement activity.

ANS: C The presence of visible peristaltic waves, accompanied by high-pitched or tingling bowel sounds, is indicative of partial obstruction caused by the tumor. The nurse should contact the provider with these results and recommend a computed tomography scan for further diagnostic testing. This assessment finding is not associated with right shoulder pain; peritonitis and cholecystitis are associated with referred pain to the right shoulder. The registered nurse is not qualified to complete a rectal examination for polyps, and laxatives would not help this client.

The circulating nurse is in the operating room and sees the surgeon don gown and gloves using appropriate sterile procedure. The surgeon then folds the hands together and places them down below the hips. What action by the nurse is most appropriate? a. Ask the surgeon to change the sterile gown. b. Do nothing; this is acceptable sterile procedure. c. Inform the surgeon that the sterile field has been broken. d. Obtain a new pair of sterile gloves for the surgeon to put on.

ANS: C The surgical gown is considered sterile from the chest to the level of the surgical field. By placing the hands down by the hips, the surgeon has broken sterile field. The circulating nurse informs the surgeon of this breach. Changing only the gloves or only the gown does not "restore" the sterile sections of the gown. Doing nothing is unacceptable.

A nurse is discharging a client after a total hip replacement. What statement by the client indicates good potential for self-management? a. "I can bend down to pick something up." b. "I no longer need to do my exercises." c. "I will not sit with my legs crossed." d. "I won't wash my incision to keep it dry."

ANS: C There are many precautions clients need to take after hip replacement surgery, including not bending more than 90 degrees at the hips, continuing prescribed exercises, not crossing the legs, and washing the incision daily and patting it dry.

A client with peptic ulcer disease is in the emergency department and reports the pain has gotten much worse over the last several days. The client's blood pressure when lying down was 122/80 mm Hg and when standing was 98/52 mm Hg. What action by the nurse is most appropriate? a. Administer ibuprofen (Motrin). b. Call the Rapid Response Team. c. Start a large-bore IV with normal saline. d. Tell the client to remain lying down.

ANS: C This client has orthostatic changes to the blood pressure, indicating fluid volume loss. The nurse should start a large-bore IV with isotonic solution. Ibuprofen will exacerbate the ulcer. The Rapid Response Team is not needed at this point. The client should be put on safety precautions, which includes staying in bed, but this is not the priority.

A nurse has taught a client about dietary changes that can reduce the chances of developing cancer. What statement by the client indicates the nurse needs to provide additional teaching? a. "Foods high in vitamin A and vitamin C are important." b. "I'll have to cut down on the amount of bacon I eat." c. "I'm so glad I don't have to give up my juicy steaks." d. "Vegetables, fruit, and high-fiber grains are important."

ANS: C To decrease the risk of developing cancer, one should cut down on the consumption of red meats and animal fat. The other statements are correct.

After a total knee replacement, a client is on the postoperative nursing unit with a continuous femoral nerve blockade. On assessment, the nurse notes the client's pulses are 2+/4+ bilaterally; the skin is pale pink, warm, and dry; and the client is unable to dorsiflex or plantarflex the affected foot. What action does the nurse perform next? a. Document the findings and monitor as prescribed. b. Increase the frequency of monitoring the client. c. Notify the surgeon or anesthesia provider immediately. d. Palpate the client's bladder or perform a bladder scan.

ANS: C With the femoral nerve block, the client should still be able to dorsiflex and plantarflex the affected foot. Since this client has an abnormal finding, the nurse should notify either the surgeon or the anesthesia provider immediately. Documentation is the last priority. Increasing the frequency of assessment may be a good idea, but first the nurse must notify the appropriate person. Palpating the bladder is not related.

A nurse assesses a client who is recovering from the implantation of a vagal nerve stimulation device. For which clinical manifestations should the nurse assess as common complications of this procedure? (Select all that apply.) a. Bleeding b. Infection c. Hoarseness d. Dysphagia e. Seizures

ANS: C, D Complications of surgery to implant a vagal nerve stimulation device include hoarseness (most common), dyspnea, neck pain, and dysphagia. The device is tunneled under the skin with an electrode connected to the vagus nerve to control simple or complex partial seizures. Bleeding is not a common complication of this procedure, and infection would not occur during the recovery period.

A nurse working with a client who has possible gastritis assesses the client's gastrointestinal system. Which findings indicate a chronic condition as opposed to acute gastritis? (Select all that apply.) a. Anorexia b. Dyspepsia c. Intolerance of fatty foods d. Pernicious anemia e. Nausea and vomiting

ANS: C, D Intolerance of fatty or spicy foods and pernicious anemia are signs of chronic gastritis. Anorexia and nausea/vomiting can be seen in both conditions. Dyspepsia is seen in acute gastritis.

A nurse reviews the medication list of a client recovering from a computed tomography (CT) scan with IV contrast to rule out small bowel obstruction. Which medication should alert the nurse to contact the provider and withhold the prescribed dose? a. Pioglitazone (Actos) b. Glimepiride (Amaryl) c. Glipizide (Glucotrol) d. Metformin (Glucophage)

ANS: D Glucophage should not be administered when the kidneys are attempting to excrete IV contrast from the body. This combination would place the client at high risk for kidney failure. The nurse should hold the metformin dose and contact the provider. The other medications are safe to administer after receiving IV contrast.

A nurse assesses a client with diabetes mellitus who self-administers subcutaneous insulin. The nurse notes a spongy, swelling area at the site the client uses most frequently for insulin injection. Which action should the nurse take? a. Apply ice to the site to reduce inflammation. b. Consult the provider for a new administration route. c. Assess the client for other signs of cellulitis. d. Instruct the client to rotate sites for insulin injection.

ANS: D The client's tissue has been damaged from continuous use of the same site. The client should be educated to rotate sites. The damaged tissue is not caused by cellulitis or any type infection, and applying ice may cause more damage to the tissue. Insulin can only be administered subcutaneously and intravenously. It would not be appropriate or practical to change the administration route.

A nurse cares for a client with amyotrophic lateral sclerosis (ALS). The client states, "I do not want to be placed on a mechanical ventilator." How should the nurse respond? a. "You should discuss this with your family and health care provider." b. "Why are you afraid of being placed on a breathing machine?" c. "Using the incentive spirometer each hour will delay the need for a ventilator." d. "What would you like to be done if you begin to have difficulty breathing?"

ANS: D ALS is an adult-onset upper and lower motor neuron disease characterized by progressive weakness, muscle wasting, and spasticity, eventually leading to paralysis. Once muscles of breathing are involved, the client must indicate in the advance directive what is to be done when breathing is no longer possible without intervention. The other statements do not address the client's needs.

A nurse is assessing a female client who is taking progestins. What assessment finding requires the nurse to notify the provider immediately? a. Irregular menses b. Edema in the lower extremities c. Ongoing breast tenderness d. Red, warm, swollen calf

ANS: D All clients receiving progestin therapy are at risk for thromboembolism. A red, warm, swollen calf is a manifestation of deep vein thrombosis and should be reported to the provider. Irregular menses, edema in the lower extremities, and breast tenderness are common side effects of the therapy.

A nurse works with clients who have alopecia from chemotherapy. What action by the nurse takes priority? a. Helping clients adjust to their appearance b. Reassuring clients that this change is temporary c. Referring clients to a reputable wig shop d. Teaching measures to prevent scalp injury

ANS: D All of the actions are appropriate for clients with alopecia. However, the priority is client safety, so the nurse should first teach ways to prevent scalp injury.

A client is scheduled for a total gastrectomy for gastric cancer. What preoperative laboratory result should the nurse report to the surgeon immediately? a. Albumin: 2.1 g/dL b. Hematocrit: 28% c. Hemoglobin: 8.1 mg/dL d. International normalized ratio (INR): 4.2

ANS: D An INR as high as 4.2 poses a serious risk of bleeding during the operation and should be reported. The albumin is low and is an expected finding. The hematocrit and hemoglobin are also low, but this is expected in gastric cancer.

A student nurse asks the nursing instructor what "apoptosis" means. What response by the instructor is best? a. Growth by cells enlarging b. Having the normal number of chromosomes c. Inhibition of cell growth d. Programmed cell death

ANS: D Apoptosis is programmed cell death. With this characteristic, organs and tissues function with cells that are at their peak of performance. Growth by cells enlarging is hyperplasia. Having the normal number of chromosomes is euploidy. Inhibition of cell growth is contact inhibition.

A nurse is discharging a client to a short-term rehabilitation center after a joint replacement. Which action by the nurse is most important? a. Administering pain medication before transport b. Answering any last-minute questions by the client c. Ensuring the family has directions to the facility d. Providing a verbal hand-off report to the facility

ANS: D As required by The Joint Commission and other accrediting agencies, a hand-off report must be given to the new provider to prevent error. The other options are valid responses but do not take priority.

After teaching the wife of a client who has Parkinson disease, the nurse assesses the wife's understanding. Which statement by the client's wife indicates she correctly understands changes associated with this disease? a. "His masklike face makes it difficult to communicate, so I will use a white board." b. "He should not socialize outside of the house due to uncontrollable drooling." c. "This disease is associated with anxiety causing increased perspiration." d. "He may have trouble chewing, so I will offer bite-sized portions."

ANS: D Because chewing and swallowing can be problematic, small frequent meals and a supplement are better for meeting the client's nutritional needs. A masklike face and drooling are common in clients with Parkinson disease. The client should be encouraged to continue to socialize and communicate as normally as possible. The wife should understand that the client's masklike face can be misinterpreted and additional time may be needed for the client to communicate with her or others. Excessive perspiration is also common in clients with Parkinson disease and is associated with the autonomic nervous system's response.

A nurse assesses a client who has multiple areas of ecchymosis on both arms. Which question should the nurse ask first? a. "Are you using lotion on your skin?" b. "Do you have a family history of this?" c. "Do your arms itch?" d. "What medications are you taking?"

ANS: D Certain drugs such as aspirin, warfarin, and corticosteroids can lead to easy or excessive bruising, which can result in ecchymosis. The other options would not provide information about bruising.

The nurse has taught a client with cancer ways to prevent infection. What statement by the client indicates that more teaching is needed? a. "I should take my temperature daily and when I don't feel well." b. "I will wash my toothbrush in the dishwasher once a week." c. "I won't let anyone share any of my personal items or dishes." d. "It's alright for me to keep my pets and change the litter box."

ANS: D Clients should wash their hands after touching their pets and should not empty or scoop the cat litter box. The other statements are appropriate for self-management.

A client with rheumatoid arthritis (RA) is on the postoperative nursing unit after having elective surgery. The client reports that one arm feels like "pins and needles" and that the neck is very painful since returning from surgery. What action by the nurse is best? a. Assist the client to change positions. b. Document the findings in the client's chart. c. Encourage range of motion of the neck. d. Notify the provider immediately.

ANS: D Clients with RA can have cervical joint involvement. This can lead to an emergent situation in which the phrenic nerve is compressed, causing respiratory insufficiency. The client can also suffer a permanent spinal cord injury. The nurse needs to notify the provider immediately. Changing positions and doing range of motion may actually worsen the situation. The nurse should document findings after notifying the provider.

After teaching a client who has a history of cholelithiasis, the nurse assesses the client's understanding. Which menu selection made by the client indicates the client clearly understands the dietary teaching? a. Lasagna, tossed salad with Italian dressing, and low-fat milk b. Grilled cheese sandwich, tomato soup, and coffee with cream c. Cream of potato soup, Caesar salad with chicken, and a diet cola d. Roasted chicken breast, baked potato with chives, and orange juice

ANS: D Clients with cholelithiasis should avoid foods high in fat and cholesterol, such as whole milk, butter, and fried foods. Lasagna, low-fat milk, grilled cheese, cream, and cream of potato soup all have high levels of fat. The meal with the least amount of fat is the chicken breast dinner.

A female client hospitalized for an unrelated problem has a large pearly-white lesion on her lip, to which she continues to apply lipstick that she will not remove for inspection. The client refuses to discuss the lesion with the nurse or health care provider. What action by the nurse is best? a. Ask the client why her appearance is so important. b. Ignore the lesion since the client will not discuss it. c. Inform the client that early-stage cancer is curable. d. Work with the client to establish a trusting relationship.

ANS: D Clients with oral cancers often have body image difficulties due to the location of the tumor or the results of surgical treatment. This client appears to be using denial to cope with this problem. The nurse should work to establish a helping-trusting relationship in hopes that the client will be amenable to future discussions about the lesion. Asking "why" questions often puts people on the defensive and should be avoided. Ignoring the lesion is not being an advocate for the client. Education is important, but right now the client is in denial, so this information will not seem relevant to her.

The nurse learns that the pathophysiology of Guillain-Barré syndrome includes segmental demyelination. The nurse should understand that this causes what? a. Delayed afferent nerve impulses b. Paralysis of affected muscles c. Paresthesia in upper extremities d. Slowed nerve impulse transmission

ANS: D Demyelination leads to slowed nerve impulse transmission. The other options are not correct.

A nurse plans care for a client with Parkinson disease. Which intervention should the nurse include in this client's plan of care? a. Ambulate the client in the hallway twice a day. b. Ensure a fluid intake of at least 3 liters per day. c. Teach the client pursed-lip breathing techniques. d. Keep the head of the bed at 30 degrees or greater

ANS: D Elevation of the head of the bed will help prevent aspiration. The other options will not prevent aspiration, which is the greatest respiratory complication of Parkinson disease, nor do these interventions address any of the complications of Parkinson disease. Ambulation in the hallway is usually implemented to prevent venous thrombosis. Increased fluid intake flushes out toxins from the client's blood. Pursed-lip breathing increases exhalation of carbon dioxide.

The nurse is caring for a client using a continuous passive motion (CPM) machine and has delegated some tasks to the unlicensed assistive personnel (UAP). What action by the UAP warrants intervention by the nurse? a. Checking to see if the machine is working b. Keeping controls in a secure place on the bed c. Placing padding in the machine per request d. Storing the CPM machine under the bed after removal

ANS: D For infection control (and to avoid tripping on it), the CPM machine is never placed on the floor. The other actions are appropriate.

A nurse is examining a client reporting right upper quadrant (RUQ) abdominal pain. What technique should the nurse use to assess this client's abdomen? a. Auscultate after palpating. b. Avoid any palpation. c. Palpate the RUQ first. d. Palpate the RUQ last.

ANS: D If pain is present in a certain area of the abdomen, that area should be palpated last to keep the client from tensing up, which could possibly affect the rest of the examination. Auscultation of the abdomen occurs prior to palpation.

A nurse teaches a client with a lower motor neuron lesion who wants to achieve bladder control. Which statement should the nurse include in this client's teaching? a. "Stroke the inner aspect of your thigh to initiate voiding." b. "Use a clean technique for intermittent catheterization." c. "Implement digital anal stimulation when your bladder is full." d. "Tighten your abdominal muscles to stimulate urine flow."

ANS: D In clients with lower motor neuron problems such as spinal cord injury, performing a Valsalva maneuver or tightening the abdominal muscles are interventions that can initiate voiding. Stroking the inner aspect of the thigh may initiate voiding in a client who has an upper motor neuron problem. Intermittent catheterization and digital anal stimulation do not initiate voiding or bladder control.

A client in the family practice clinic has restless leg syndrome. Routine laboratory work reveals white blood cells 8000/mm3, magnesium 0.8 mEq/L, and sodium 138 mEq/L. What action by the nurse is best? a. Advise the client to restrict fluids. b. Assess the client for signs of infection. c. Have the client add table salt to food. d. Instruct the client on a magnesium supplement.

ANS: D Iron and magnesium deficiencies can sometimes exacerbate or increase symptoms of restless leg syndrome. The client's magnesium level is low, and the client should be advised to add a magnesium supplement. The other actions are not needed.

A nurse cares for a client who is hemorrhaging from bleeding esophageal varices and has an esophagogastric tube. Which action should the nurse take first? a. Sedate the client to prevent tube dislodgement. b. Maintain balloon pressure at 15 and 20 mm Hg. c. Irrigate the gastric lumen with normal saline. d. Assess the client for airway patency.

ANS: D Maintaining airway patency is the primary nursing intervention for this client. The nurse suctions oral secretions to prevent aspiration and occlusion of the airway. The client usually is intubated and mechanically ventilated during this treatment. The client should be sedated, balloon pressure should be maintained between 15 and 20 mm Hg, and the lumen can be irrigated with saline or tap water. However, these are not a higher priority than airway patency.

A nurse prepares a client for prescribed magnetic resonance imaging (MRI). Which action should the nurse implement prior to the test? a. Implement nothing by mouth (NPO) status for 8 hours. b. Withhold all daily medications until after the examination. c. Administer morphine sulfate to prevent claustrophobia during the test. d. Place the client in a gown that has cloth ties instead of metal snaps.

ANS: D Metal objects are a hazard because of the magnetic field used in the MRI procedure. Morphine sulfate is not administered to prevent claustrophobia; lorazepam (Ativan) or diazepam (Valium) may be used instead. The client does not need to be NPO, and daily medications do not need to be withheld prior to MRI.

A nurse cares for a client who presents with an acute exacerbation of multiple sclerosis (MS). Which prescribed medication should the nurse prepare to administer? a. Baclofen (Lioresal) b. Interferon beta-1b (Betaseron) c. Dantrolene sodium (Dantrium) d. Methylprednisolone (Medrol)

ANS: D Methylprednisolone is the drug of choice for acute exacerbations of the disease. The other drugs are not used to treat acute exacerbations of MS. Interferon beta-1b is used to treat and control MS, decrease specific symptoms, and slow the progression of the disease. Baclofen and dantrolene sodium are prescribed to lessen muscle spasticity associated with MS.

A nurse assesses a client who is prescribed an infusion of vasopressin (Pitressin) for bleeding esophageal varices. Which clinical manifestation should alert the nurse to a serious adverse effect? a. Nausea and vomiting b. Frontal headache c. Vertigo and syncope d. Mid-sternal chest pain

ANS: D Mid-sternal chest pain is indicative of acute angina or myocardial infarction, which can be precipitated by vasopressin. Nausea and vomiting, headache, and vertigo and syncope are not side effects of vasopressin.

An older client has returned to the surgical unit after a total hip replacement. The client is confused and restless. What intervention by the nurse is most important to prevent injury? a. Administer mild sedation. b. Keep all four siderails up. c. Restrain the client's hands. d. Use an abduction pillow.

ANS: D Older clients often have trouble metabolizing anesthetics and pain medication, leading to confusion or restlessness postoperatively. To prevent the hip from dislocating, the nurse should use an abduction pillow since the client cannot follow directions at this time. Sedation may worsen the client's mental status and should be avoided. Using all four siderails may be considered a restraint. Hand restraints are not necessary in this situation.

A nurse is caring for a client after joint replacement surgery. What action by the nurse is most important to prevent wound infection? a. Assess the client's white blood cell count. b. Culture any drainage from the wound. c. Monitor the client's temperature every 4 hours. d. Use aseptic technique for dressing changes.

ANS: D Preventing surgical wound infection is a primary responsibility of the nurse, who must use aseptic technique to change dressings or empty drains. The other actions do not prevent infection but can lead to early detection of an infection that is already present.

A nurse assesses clients on the medical-surgical unit. Which client should the nurse identify as at high risk for pancreatic cancer? a. A 26-year-old with a body mass index of 21 b. A 33-year-old who frequently eats sushi c. A 48-year-old who often drinks wine d. A 66-year-old who smokes cigarettes

ANS: D Risk factors for pancreatic cancer include obesity, older age, high intake of red meat, and cigarette smoking. Sushi and wine intake are not risk factors for pancreatic cancer.

A preoperative nurse is assessing a client prior to surgery. Which information would be most important for the nurse to relay to the surgical team? a. Allergy to bee and wasp stings b. History of lactose intolerance c. No previous experience with surgery d. Use of multiple herbs and supplements

ANS: D Some herbs and supplements can interact with medications, so this information needs to be reported as the priority. An allergy to bee and wasp stings should not affect the client during surgery. Lactose intolerance should also not affect the client during surgery but will need to be noted before a postoperative diet is ordered. Lack of experience with surgery may increase anxiety and may require higher teaching needs, but is not the priority over client safety.

A nurse assesses a client who is recovering from a total thyroidectomy and notes the development of stridor. Which action should the nurse take first? a. Reassure the client that the voice change is temporary. b. Document the finding and assess the client hourly. c. Place the client in high-Fowler's position and apply oxygen. d. Contact the provider and prepare for intubation.

ANS: D Stridor on exhalation is a hallmark of respiratory distress, usually caused by obstruction resulting from edema. One emergency measure is to remove the surgical clips to relieve the pressure. This might be a physician function. The nurse should prepare to assist with emergency intubation or tracheostomy while notifying the provider or the Rapid Response Team. Stridor is an emergency situation; therefore, reassuring the client, documenting, and reassessing in an hour do not address the urgency of the situation. Oxygen should be applied, but this action will not keep the airway open.

A client is in the oncology clinic for a first visit since being diagnosed with cancer. The nurse reads in the client's chart that the cancer classification is TISN0M0. What does the nurse conclude about this client's cancer? a. The primary site of the cancer cannot be determined. b. Regional lymph nodes could not be assessed. c. There are multiple lymph nodes involved already. d. There are no distant metastases noted in the report.

ANS: D TIS stands for carcinoma in situ; N0 stands for no regional lymph node metastasis; and M0 stands for no distant metastasis.

A nurse prepares to palpate a client's thyroid gland. Which action should the nurse take when performing this assessment? a. Stand in front of the client instead of behind the client. b. Ask the client to swallow after palpating the thyroid. c. Palpate the right lobe with the nurse's left hand. d. Place the client in a sitting position with the chin tucked down.

ANS: D The client should be in a sitting position with the chin tucked down as the examiner stands behind the client. The nurse feels for the thyroid isthmus while the client swallows and turns the head to the right, and the nurse palpates the right lobe with the right hand. The technique is repeated in the opposite fashion for the left lobe.

An emergency room nurse initiates care for a client with a cervical spinal cord injury who arrives via emergency medical services. Which action should the nurse take first? a. Assess level of consciousness. b. Obtain vital signs. c. Administer oxygen therapy. d. Evaluate respiratory status.

ANS: D The first priority for a client with a spinal cord injury is assessment of respiratory status and airway patency. Clients with cervical spine injuries are particularly prone to respiratory compromise and may even require intubation. The other assessments should be performed after airway and breathing are assessed.

A circulating nurse wishes to provide emotional support to a client who was just transferred to the operating room. What action by the nurse would be best? a. Administer anxiolytics. b. Give the client warm blankets. c. Introduce the surgical staff. d. Remain with the client.

ANS: D The nurse can provide emotional support by remaining with the client until anesthesia has been provided. An extremely anxious client may need anxiolytics, but not all clients require this for emotional support. Physical comfort and introductions can also help decrease anxiety.

After teaching a client newly diagnosed with epilepsy, the nurse assesses the client's understanding. Which statement by the client indicates a need for additional teaching? a. "I will wear my medical alert bracelet at all times." b. "While taking my epilepsy medications, I will not drink any alcoholic beverages." c. "I will tell my doctor about my prescription and over-the-counter medications." d. "If I am nauseated, I will not take my epilepsy medication."

ANS: D The nurse must emphasize that antiepileptic drugs must be taken even if the client is nauseous. Discontinuing the medication can predispose the client to seizure activity and status epilepticus. The client should not drink alcohol while taking seizure medications. The client should wear a medical alert bracelet and should make the doctor aware of all medications to prevent complications of polypharmacy.

A nurse cares for a client who has been diagnosed with the Huntington gene but has no symptoms. The client asks for options related to family planning. What is the nurse's best response? a. "Most clients with the Huntington gene do not pass on Huntington disease to their children." b. "I understand that they can diagnose this disease in embryos. Therefore, you could select a healthy embryo from your fertilized eggs for implantation to avoid passing on Huntington disease." c. "The need for family planning is limited because one of the hallmarks of Huntington disease is infertility." d. "Tell me more specifically what information you need about family planning so that I can direct you to the right information or health care provider."

ANS: D The presence of the Huntington gene means that the trait will be passed on to all offspring of the affected person. Understanding options for contraception and conception (e.g., surrogacy options) and implications for children may require the expertise of a genetic counselor or a reproductive specialist. The other statements are not accurate.

A client with a stroke is being evaluated for fibrinolytic therapy. What information from the client or family is most important for the nurse to obtain? a. Loss of bladder control b. Other medical conditions c. Progression of symptoms d. Time of symptom onset

ANS: D The time limit for initiating fibrinolytic therapy for a stroke is 3 to 4.5 hours, so the exact time of symptom onset is the most important information for this client. The other information is not as critical.

A client in the intensive care unit is scheduled for a lumbar puncture (LP) today. On assessment, the nurse finds the client breathing irregularly with one pupil fixed and dilated. What action by the nurse is best? a. Ensure that informed consent is on the chart. b. Document these findings in the client's record. c. Give the prescribed preprocedure sedation. d. Notify the provider of the findings immediately.

ANS: D This client is exhibiting signs of increased intracranial pressure. The nurse should notify the provider immediately because performing the LP now could lead to herniation. Informed consent is needed for an LP, but this is not the priority. Documentation should be thorough, but again this is not the priority. The preprocedure sedation (or other preprocedure medications) should not be given as the LP will most likely be canceled.

A new nurse has been assigned a client who is in the hospital to receive iodine-131 treatment. Which action by the nurse is best? a. Ensure the client is placed in protective isolation. b. Hand off a pregnant client to another nurse. c. No special action is necessary to care for this client. d. Read the policy on handling radioactive excreta.

ANS: D This type of radioisotope is excreted in body fluids and excreta (urine and feces) and should not be handled directly. The nurse should read the facility's policy for handling and disposing of this type of waste. The other actions are not warranted.

A client presents to the family practice clinic reporting a week of watery, somewhat bloody diarrhea. The nurse assists the client to obtain a stool sample. What action by the nurse is most important? a. Ask the client about recent exposure to illness. b. Assess the client's stool for obvious food particles. c. Include the date and time on the specimen container. d. Put on gloves prior to collecting the sample.

ANS: D To avoid possible exposure to infectious agents, the nurse dons gloves prior to handling any bodily secretions. Recent exposure to illness is not related to collecting a stool sample. The nurse can visually inspect the stool for food particles, but it still needs analysis in the laboratory. The container should be dated and timed, but safety for the staff and other clients comes first.

The nurse in the rheumatology clinic is assessing clients with rheumatoid arthritis (RA). Which client should the nurse see first? a. Client taking celecoxib (Celebrex) and ranitidine (Zantac) b. Client taking etanercept (Enbrel) with a red injection site c. Client with a blood glucose of 190 mg/dL who is taking steroids d. Client with a fever and cough who is taking tofacitinib (Xeljanz)

ANS: D Tofacitinib carries a Food and Drug Administration black box warning about opportunistic infections, tuberculosis, and cancer. Fever and cough may indicate tuberculosis. Ranitidine is often taken with celecoxib, which can cause gastrointestinal distress. Redness and itchy rashes are frequently seen with etanercept injections. Steroids are known to raise blood glucose levels.

A client is being taught about drug therapy for Helicobacter pylori infection. What assessment by the nurse is most important? a. Alcohol intake of 1 to 2 drinks per week b. Family history of H. pylori infection c. Former smoker still using nicotine patches d. Willingness to adhere to drug therapy

ANS: D Treatment for this infection involves either triple or quadruple drug therapy, which may make it difficult for clients to remain adherent. The nurse should assess the client's willingness and ability to follow the regimen. The other assessment findings are not as critical.

Which statement about carcinogenesis is accurate? a. An initiated cell will always become clinical cancer. b. Cancer becomes a health problem once it is 1 cm in size. c. Normal hormones and proteins do not promote cancer growth. d. Tumor cells need to develop their own blood supply.

ANS: D Tumors need to develop their own blood supply through a process called angiogenesis. An initiated cell needs a promoter to continue its malignant path. Normal hormones and proteins in the body can act as promoters. A 1-cm tumor is a detectable size, but other events have to occur for it to become a health problem.

A client with myasthenia gravis has the priority client problem of inadequate nutrition. What assessment finding indicates that the priority goal for this client problem has been met? a. Ability to chew and swallow without aspiration b. Eating 75% of meals and between-meal snacks c. Intake greater than output 3 days in a row d. Weight gain of 3 pounds in 1 month

ANS: D Weight gain is the best indicator that the client is receiving enough nutrition. Being able to chew and swallow is important for eating, but adequate nutrition can be accomplished through enteral means if needed. Swallowing without difficulty indicates an intact airway. Since the question does not indicate what the client's meals and snacks consist of, eating 75% may or may not be adequate. Intake and output refers to fluid balance.

A nurse prepares to provide perineal care to a client with meningococcal meningitis. Which personal protective equipment should the nurse wear? (Select all that apply.) a. Particulate respirator b. Isolation gown c. Shoe covers d. Surgical mask e. Gloves

ANS: D, E Meningeal meningitis is spread via saliva and droplets, and Droplet Precautions are necessary. Caregivers should wear a surgical mask when within 6 feet of the client and should continue to use Standard Precautions, including gloves. A particulate respirator, an isolation gown, and shoe covers are not necessary for Droplet Precautions.

A client has a possible connective tissue disease and the nurse is reviewing the client's laboratory values. Which laboratory values and their related connective tissue diseases (CTDs) are correctly matched? (Select all that apply.) a. Elevated antinuclear antibody (ANA) - Normal value; no connective tissue disease b. Elevated sedimentation rate - Rheumatoid arthritis c. Lowered albumin - Indicative only of nutritional deficit d. Positive human leukocyte antigen B27 (HLA-B27) - Reiter's syndrome or ankylosing spondylitis e. Positive rheumatoid factor - Possible kidney disease

ANS: D, E The HLA-B27 is diagnostic for Reiter's syndrome or ankylosing spondylitis. A positive rheumatoid factor can be seen in autoimmune CTDs, kidney and liver disease, or leukemia. An elevated ANA is indicative of inflammatory CTDs, although a small minority of healthy adults also have this finding. An elevated sedimentation rate indicates inflammation, whether from an infection, an injury, or an autoimmune CTD. Lowered albumin is seen in nutritional deficiencies but also in chronic infection or inflammation.

A nurse assesses an older adult's skin. Which findings require immediate referral? (Select all that apply.) a. Excessive moisture under axilla b. Increased hair thinning c. Increased presence of fungal toenails d. Lesion with various colors e. Spider veins on legs f. Asymmetric 6-mm dark lesion on forehead

ANS: D, F The lesion with various colors, as well as the asymmetric 6-mm dark lesion, fits two of the American Cancer Society's hallmark signs for cancer according to the ABCD method. Other manifestations are variants of normal seen in various age groups.

After teaching a client who has alcohol-induced cirrhosis, a nurse assesses the client's understanding. Which statement made by the client indicates a need for additional teaching? a. "I cannot drink any alcohol at all anymore." b. "I need to avoid protein in my diet." c. "I should not take over-the-counter medications." d. "I should eat small, frequent, balanced meals."

Based on the degree of liver involvement and decreased function, protein intake may have to be decreased. However, some protein is necessary for the synthesis of albumin and normal healing. The other statements indicate accurate understanding of self-care measures for this client

A client is receiving plasmapheresis. What action by the nurse best prevents infection in this client? a. Giving antibiotics prior to treatments b. Monitoring the client's vital signs c. Performing appropriate hand hygiene d. Placing the client in protective isolation

Plasmapheresis is an invasive procedure, and the nurse uses good hand hygiene before and after client contact to prevent infection. Antibiotics are not necessary. Monitoring vital signs does not prevent infection but could alert the nurse to its possibility. The client does not need isolation.

A nurse cares for a client who is recovering from an open Whipple procedure. Which action should the nurse take? a. Clamp the nasogastric tube. b. Place the client in semi-Fowler's position. c. Assess vital signs once every shift. d. Provide oral rehydration.

Postoperative care for a client recovering from an open Whipple procedure should include placing the client in a semi-Fowler's position to reduce tension on the suture line and anastomosis sites, setting the nasogastric tube to low suction to remove free air buildup and pressure, assessing vital signs frequently to assess fluid and electrolyte complications, and providing intravenous fluids.

A nurse prepares to administer prescribed regular and NPH insulin. Place the nurse's actions in the correct order to administer these medications. 1. Inspect bottles for expiration dates. 2. Gently roll the bottle of NPH between the hands. 3. Wash your hands. 4. Inject air into the regular insulin. 5. Withdraw the NPH insulin. 6. Withdraw the regular insulin. 7. Inject air into the NPH bottle. 8. Clean rubber stoppers with an alcohol swab.

ANS: 3, 1, 2, 8, 7, 4, 6, 5 After washing hands, it is important to inspect the bottles and then to roll the NPH to mix the insulin. Rubber stoppers should be cleaned with alcohol after rolling the NPH and before sticking a needle into either bottle. It is important to inject air into the NPH bottle before placing the needle in a regular insulin bottle to avoid mixing of regular and NPH insulin. The shorter-acting insulin is always drawn up first.

A preoperative nurse assesses a client who has type 1 diabetes mellitus prior to a surgical procedure. The client's blood glucose level is 160 mg/dL. Which action should the nurse take? a. Document the finding in the client's chart. b. Administer a bolus of regular insulin IV. c. Call the surgeon to cancel the procedure. d. Draw blood gases to assess the metabolic state.

ANS: A Clients who have type 1 diabetes and are having surgery have been found to have fewer complications, lower rates of infection, and better wound healing if blood glucose levels are maintained at between 140 and 180 mg/dL throughout the perioperative period. The nurse should document the finding and proceed with other operative care. The need for a bolus of insulin, canceling the procedure, or drawing arterial blood gases is not required.

A nurse assesses a client who has a chronic wound. The client states, "I do not clean the wound and change the dressing every day because it costs too much for supplies." How should the nurse respond? a."You can use tap water instead of sterile saline to clean your wound." b."If you don't clean the wound properly, you could end up in the hospital." c."Sterile procedure is necessary to keep this wound from getting infected." d."Good hand hygiene is the only thing that really matters with wound care."

ANS: A For chronic wounds in the home, clean tap water and nonsterile supplies are acceptable and serve as cheaper alternatives to sterile supplies. Of course, if the wound becomes grossly infected, the client may end up in the hospital, but this response does not provide any helpful information. Good handwashing is important, but it is not the only consideration.

A nurse teaches a client who is diagnosed with diabetes mellitus. Which statement should the nurse include in this client's plan of care to delay the onset of microvascular and macrovascular complications? a. "Maintain tight glycemic control and prevent hyperglycemia." b. "Restrict your fluid intake to no more than 2 liters a day." c. "Prevent hypoglycemia by eating a bedtime snack." d. "Limit your intake of protein to prevent ketoacidosis."

ANS: A Hyperglycemia is a critical factor in the pathogenesis of long-term diabetic complications. Maintaining tight glycemic control will help delay the onset of complications. Restricting fluid intake is not part of the treatment plan for clients with diabetes. Preventing hypoglycemia and ketosis, although important, are not as important as maintaining daily glycemic control.

After teaching a client who is recovering from pancreas transplantation, the nurse assesses the client's understanding. Which statement made by the client indicates a need for additional education? a. "If I develop an infection, I should stop taking my corticosteroid." b. "If I have pain over the transplant site, I will call the surgeon immediately." c. "I should avoid people who are ill or who have an infection." d. "I should take my cyclosporine exactly the way I was taught."

ANS: A Immunosuppressive agents should not be stopped without the consultation of the transplantation physician, even if an infection is present. Stopping immunosuppressive therapy endangers the transplanted organ. The other statements are correct. Pain over the graft site may indicate rejection. Anti-rejection drugs cause immunosuppression, and the client should avoid crowds and people who are ill. Changing the routine of anti-rejection medications may cause them to not work optimally.

A nurse assesses a client who is prescribed alosetron (Lotronex). Which assessment question should the nurse ask this client? a. "Have you been experiencing any constipation?" b. "Are you eating a diet high in fiber and fluids?" c. "Do you have a history of high blood pressure?" d. "What vitamins and supplements are you taking?"

ANS: A Ischemic colitis is a life-threatening complication of alosetron. The nurse should assess the client for constipation. The other questions do not identify complications related to alosetron.

A nurse prepares to administer insulin to a client at 1800. The client's medication administration record contains the following information: • Insulin glargine: 12 units daily at 1800 • Regular insulin: 6 units QID at 0600, 1200, 1800, 2400 Based on the client's medication administration record, which action should the nurse take? a. Draw up and inject the insulin glargine first, and then draw up and inject the regular insulin. b. Draw up and inject the insulin glargine first, wait 20 minutes, and then draw up and inject the regular insulin. c. First draw up the dose of regular insulin, then draw up the dose of insulin glargine in the same syringe, mix, and inject the two insulins together. d. First draw up the dose of insulin glargine, then draw up the dose of regular insulin in the same syringe, mix, and inject the two insulins together.

ANS: A Insulin glargine must not be diluted or mixed with any other insulin or solution. Mixing results in an unpredictable alteration in the onset of action and time to peak action. The correct instruction is to draw up and inject first the glargine and then the regular insulin right afterward.

A nurse cares for a client with diabetes mellitus who is visually impaired. The client asks, "Can I ask my niece to prefill my syringes and then store them for later use when I need them?" How should the nurse respond? a. "Yes. Prefilled syringes can be stored for 3 weeks in the refrigerator in a vertical position with the needle pointing up." b. "Yes. Syringes can be filled with insulin and stored for a month in a location that is protected from light." c. "Insulin reacts with plastic, so prefilled syringes are okay, but you will need to use glass syringes." d. "No. Insulin syringes cannot be prefilled and stored for any length of time outside of the container."

ANS: A Insulin is relatively stable when stored in a cool, dry place away from light. When refrigerated, prefilled plastic syringes are stable for up to 3 weeks. They should be stored in the refrigerator in the vertical position with the needle pointing up to prevent suspended insulin particles from clogging the needle.

An emergency department nurse assesses a client with ketoacidosis. Which clinical manifestation should the nurse correlate with this condition? a. Increased rate and depth of respiration b. Extremity tremors followed by seizure activity c. Oral temperature of 102° F (38.9° C) d. Severe orthostatic hypotension

ANS: A Ketoacidosis decreases the pH of the blood, stimulating the respiratory control areas of the brain to buffer the effects of increasing acidosis. The rate and depth of respiration are increased (Kussmaul respirations) in an attempt to excrete more acids by exhalation. Tremors, elevated temperature, and orthostatic hypotension are not associated with ketoacidosis.

A nurse who manages client placements prepares to place four clients on a medical-surgical unit. Which client should be placed in isolation awaiting possible diagnosis of infection with methicillin-resistant Staphylococcus aureus(MRSA)? a.Client admitted from a nursing home with furuncles and folliculitis b.Client with a leg cut and other trauma from a motorcycle crash c.Client with a rash noticed after participating in sporting events d.Client transferred from intensive care with an elevated white blood cell count

ANS: A The client in long-term care and other communal environments is at high risk for MRSA. The presence of furuncles and folliculitis is also an indication that MRSA may be present. A client with an open wound from a motorcycle crash would have the potential to develop MRSA, but no signs are visible at present. The rash following participation in a sporting event could be caused by several different things. A client with an elevated white blood cell count has the potential for infection but should be at lower risk for MRSA than the client admitted from the communal environment.

A nurse assesses a client with diabetes mellitus and notes the client only responds to a sternal rub by moaning, has capillary blood glucose of 33 g/dL, and has an intravenous line that is infiltrated with 0.45% normal saline. Which action should the nurse take first? a. Administer 1 mg of intramuscular glucagon. b. Encourage the client to drink orange juice. c. Insert a new intravenous access line. d. Administer 25 mL dextrose 50% (D50) IV push.

ANS: A The client's blood glucose level is dangerously low. The nurse needs to administer glucagon IM immediately to increase the client's blood glucose level. The nurse should insert a new IV after administering the glucagon and can use the new IV site for future doses of D50 if the client's blood glucose level does not rise. Once the client is awake, orange juice may be administered orally along with a form of protein such as a peanut butter.

A nurse assesses an older client who is scratching and rubbing white ridges on the skin between the fingers and on the wrists. Which action should the nurse take? a.Place the client in a single room. b.Administer an antihistamine. c.Assess the client's airway. d.Apply gloves to minimize friction.

ANS: A The client's presentation is most likely to be scabies, a contagious mite infestation. The client needs to be admitted to a single room and treated for the infestation. Secondary interventions may include medication to decrease the itching. This is not an allergic manifestation; therefore, antihistamine and airway assessments are not indicated. Gloves may decrease skin breakdown but would not address the client's infectious disorder.

A nurse prepares to discharge a client who has a wound and is prescribed home health care. Which information should the nurse include in the hand-off report to the home health nurse? a.Recent wound assessment, including size and appearance b.Insurance information for billing and coding purposes c.Complete health history and physical assessment findings d.Resources available to the client for wound care supplies

ANS: A The hospital nurse should provide details about the wound, including size and appearance and any special wound needs, in a hand-off report to the home health nurse. Insurance information is important to the home health agency and manager, but this is not appropriate during this hand-off report. The nurse should report focused assessment findings instead of a complete health history and physical assessment. The home health nurse should work with the client to identify community resources.

A nurse cares for a client who has a deep wound that is being treated with a wet-to-damp dressing. Which intervention should the nurse include in this client's plan of care? a.Change the dressing every 6 hours. b.Assess the wound bed once a day. c.Change the dressing when it is saturated. d.Contact the provider when the dressing leaks.

ANS: A Wet-to-damp dressings are changed every 4 to 6 hours to provide maximum débridement. The wound should be assessed each time the dressing is changed. Dry gauze dressings should be changed when the outer layer becomes saturated. Synthetic dressings can be left in place for extended periods of time but need to be changed if the seal breaks and the exudate leaks.

A nurse is caring for four clients in the neurologic intensive care unit. After receiving the hand-off report, which client should the nurse see first? a. Client with a Glasgow Coma Scale score that was 10 and is now is 8 b. Client with a Glasgow Coma Scale score that was 9 and is now is 12 c. Client with a moderate brain injury who is amnesic for the event d. Client who is requesting pain medication for a headache

ANS: A A 2-point decrease in the Glasgow Coma Scale score is clinically significant and the nurse needs to see this client first. An improvement in the score is a good sign. Amnesia is an expected finding with brain injuries, so this client is lower priority. The client requesting pain medication should be seen after the one with the declining Glasgow Coma Scale score.

A nurse prepares to discharge a client with chronic pancreatitis. Which question should the nurse ask to ensure safety upon discharge? a. "Do you have a one- or two-story home?" b. "Can you check your own pulse rate?" c. "Do you have any alcohol in your home?" d. "Can you prepare your own meals?"

ANS: A A client recovering from chronic pancreatitis should be limited to one floor until strength and activity increase. The client will need a bathroom on the same floor for frequent defecation. Assessing pulse rate and preparation of meals is not specific to chronic pancreatitis. Although the client should be encouraged to stop drinking alcoholic beverages, asking about alcohol availability is not adequate to assess this client's safety.

A nurse in the family clinic is teaching a client newly diagnosed with osteoarthritis (OA) about drugs used to treat the disease. For which medication does the nurse plan primary teaching? a. Acetaminophen (Tylenol) b. Cyclobenzaprine hydrochloride (Flexeril) c. Hyaluronate (Hyalgan) d. Ibuprofen (Motrin)

ANS: A All of the drugs are appropriate to treat OA. However, the first-line drug is acetaminophen. Cyclobenzaprine is a muscle relaxant given to treat muscle spasms. Hyaluronate is a synthetic joint fluid implant. Ibuprofen is a nonsteroidal anti-inflammatory drug.

After a stroke, a client has ataxia. What intervention is most appropriate to include on the client's plan of care? a. Ambulate only with a gait belt. b. Encourage double swallowing. c. Monitor lung sounds after eating. d. Perform post-void residuals.

ANS: A Ataxia is a gait disturbance. For the client's safety, he or she should have assistance and use a gait belt when ambulating. Ataxia is not related to swallowing, aspiration, or voiding.

A client who has had systemic lupus erythematosus (SLE) for many years is in the clinic reporting hip pain with ambulation. Which action by the nurse is best? a. Assess medication records for steroid use. b. Facilitate a consultation with physical therapy. c. Measure the range of motion in both hips. d. Notify the health care provider immediately.

ANS: A Chronic steroid use is seen in clients with SLE and can lead to osteonecrosis (bone necrosis). The nurse should determine if the client has been taking a steroid. Physical therapy may be beneficial, but there is not enough information about the client yet. Measuring range of motion is best done by the physical therapist. Notifying the provider immediately is not warranted.

The nurse is teaching a client with gout dietary strategies to prevent exacerbations or other problems. Which statement by the nurse is most appropriate? a. "Drink 1 to 2 liters of water each day." b. "Have 10 to 12 ounces of juice a day." c. "Liver is a good source of iron." d. "Never eat hard cheeses or sardines."

ANS: A Kidney stones are common in clients with gout, so drinking plenty of water will help prevent this from occurring. Citrus juice is high in ash, which can help prevent the formation of stones, but the value of this recommendation is not clear. Clients with gout should not eat organ meats or fish with bones, such as sardines.

A nurse delegates care for a client with Parkinson disease to an unlicensed assistive personnel (UAP). Which statement should the nurse include when delegating this client's care? a. "Allow the client to be as independent as possible with activities." b. "Assist the client with frequent and meticulous oral care." c. "Assess the client's ability to eat and swallow before each meal." d. "Schedule appointments early in the morning to ensure rest in the afternoon."

ANS: A Clients with Parkinson disease do not move as quickly and can have functional problems. The client should be encouraged to be as independent as possible and provided time to perform activities without rushing. Although oral care is important for all clients, instructing the UAP to provide frequent and meticulous oral is not a priority for this client. This statement would be a priority if the client was immune-compromised or NPO. The nurse should assess the client's ability to eat and swallow; this should not be delegated. Appointments and activities should not be scheduled early in the morning because this may cause the client to be rushed and discourage the client from wanting to participate in activities of daily living.

A nurse assesses a client who is recovering from a Whipple procedure. Which assessment finding alerts the nurse to urgently contact the health care provider? a. Drainage from a fistula b. Absent bowel sounds c. Pain at the incision site d. Nasogastric (NG) tube drainage

ANS: A Complications of a Whipple procedure include secretions that drain from a fistula and peritonitis. Absent bowel sounds, pain at the incision site, and NG tube drainage are normal postoperative findings.

A client is taking long-term corticosteroids for myasthenia gravis. What teaching is most important? a. Avoid large crowds and people who are ill. b. Check blood sugars four times a day. c. Use two forms of contraception. d. Wear properly fitting socks and shoes.

ANS: A Corticosteroids reduce immune function, so clients taking these medications must avoid being exposed to illness. Long-term use can lead to secondary diabetes, but the client would not need to start checking blood glucose unless diabetes had been detected. Corticosteroids do not affect the effectiveness of contraception. Wearing well-fitting shoes would be important to avoid injury, but not just because the client takes corticosteroids.

A client asks the nurse if eating only preservative- and dye-free foods will decrease cancer risk. What response by the nurse is best? a. "Maybe; preservatives, dyes, and preparation methods may be risk factors." b. "No; research studies have never shown those things to cause cancer." c. "There are other things you can do that will more effectively lower your risk." d. "Yes; preservatives and dyes are well known to be carcinogens."

ANS: A Dietary factors related to cancer development are poorly understood, although dietary practices are suspected to alter cancer risk. Suspected dietary risk factors include low fiber intake and a high intake of red meat or animal fat. Preservatives, preparation methods, and additives (dyes, flavorings, sweeteners) may have cancer-promoting effects. It is correct to say that other things can lower risk more effectively, but this does not give the client concrete information about how to do so, and also does not answer the client's question.

A client in the operating room has developed malignant hyperthermia. The client's potassium is 6.5 mEq/L. What action by the nurse takes priority? a. Administer 10 units of regular insulin. b. Administer nifedipine (Procardia). c. Assess urine for myoglobin or blood. d. Monitor the client for dysrhythmias.

ANS: A For hyperkalemia in a client with malignant hyperthermia, the nurse administers 10 units of regular insulin in 50 mL of 50% dextrose. This will force potassium back into the cells rapidly. Nifedipine is a calcium channel blocker used to treat hypertension and dysrhythmias, and should not be used in a client with malignant hyperthermia. Assessing the urine for blood or myoglobin is important, but does not take priority. Monitoring the client for dysrhythmias is also important due to the potassium imbalance, but again does not take priority over treating the potassium imbalance.

A client is having surgery. The circulating nurse notes the client's oxygen saturation is 90% and the heart rate is 110 beats/min. What action by the nurse is best? a. Assess the client's end-tidal carbon dioxide level. b. Document the findings in the client's chart. c. Inform the anesthesia provider of these values. d. Prepare to administer dantrolene sodium (Dantrium).

ANS: A Malignant hyperthermia is a rare but serious reaction to anesthesia. The triad of early signs include decreased oxygen saturation, tachycardia, and elevated end-tidal carbon dioxide (CO2) level. The nurse should quickly check the end-tidal CO2 and then report findings to the anesthesia provider and surgeon. Documentation is vital, but not the most important action at this stage. Dantrolene sodium is the drug of choice if the client does have malignant hyperthermia.

A nurse cares for a client who is prescribed a 24-hour urine collection. The unlicensed assistive personnel (UAP) reports that, while pouring urine into the collection container, some urine splashed his hand. Which action should the nurse take next? a. Ask the UAP if he washed his hands afterward. b. Have the UAP fill out an incident report. c. Ask the laboratory if the container has preservative in it. d. Send the UAP to Employee Health right away.

ANS: A For safety, the nurse should find out if the UAP washed his or her hands. The UAP should do this for two reasons. First, it is part of Standard Precautions to wash hands after client care. Second, if the container did have preservative in it, this would wash it away. The preservative may be caustic to the skin. The nurse can call the laboratory while the UAP is washing hands, if needed. The UAP would then need to fill out an incident or exposure report and may or may not need to go to Employee Health. The UAP also needs further education on Standard Precautions, which include wearing gloves.

A client recently diagnosed with systemic lupus erythematosus (SLE) is in the clinic for a follow-up visit. The nurse evaluates that the client practices good self-care when the client makes which statement? a. "I always wear long sleeves, pants, and a hat when outdoors." b. "I try not to use cosmetics that contain any type of sunblock." c. "Since I tend to sweat a lot, I use a lot of baby powder." d. "Since I can't be exposed to the sun, I have been using a tanning bed."

ANS: A Good self-management of the skin in SLE includes protecting the skin from sun exposure, using sunblock, avoiding drying agents such as powder, and avoiding tanning beds.

A nurse is providing community screening for risk factors associated with stroke. Which client would the nurse identify as being at highest risk for a stroke? a. A 27-year-old heavy cocaine user b. A 30-year-old who drinks a beer a day c. A 40-year-old who uses seasonal antihistamines d. A 65-year-old who is active and on no medications

ANS: A Heavy drug use, particularly cocaine, is a risk factor for stroke. Heavy alcohol use is also a risk factor, but one beer a day is not considered heavy drinking. Antihistamines may contain phenylpropanolamine, which also increases the risk for stroke, but this client uses them seasonally and there is no information that they are abused or used heavily. The 65-year-old has only age as a risk factor.

An older client is hospitalized with Guillain-Barré syndrome. A family member tells the nurse the client is restless and seems confused. What action by the nurse is best? a. Assess the client's oxygen saturation. b. Check the medication list for interactions. c. Place the client on a bed alarm. d. Put the client on safety precautions.

ANS: A In the older adult, an early sign of hypoxia is often confusion and restlessness. The nurse should first assess the client's oxygen saturation. The other actions are appropriate, but only after this assessment occurs.

A client is receiving interleukins along with chemotherapy. What assessment by the nurse takes priority? a. Blood pressure b. Lung assessment c. Oral mucous membranes d. Skin integrity

ANS: A Interleukins can cause capillary leak syndrome and fluid shifting, leading to intravascular volume depletion. Although all assessments are important in caring for clients with cancer, blood pressure and other assessments of fluid status take priority

A client is receiving chemotherapy through a peripheral IV line. What action by the nurse is most important? a. Assessing the IV site every hour b. Educating the client on side effects c. Monitoring the client for nausea d. Providing warm packs for comfort

ANS: A Intravenous chemotherapy can cause local tissue destruction if it extravasates into the surrounding tissues. Peripheral IV lines are more prone to this than centrally placed lines. The most important intervention is prevention, so the nurse should check hourly to ensure the IV site is patent, or frequently depending on facility policy. Education and monitoring for side effects such as nausea are important for all clients receiving chemotherapy. Warm packs may be helpful for comfort, but if the client reports that an IV site is painful, the nurse needs to assess further.

The nurse working with clients who have gastrointestinal problems knows that which laboratory values are related to what organ dysfunctions? (Select all that apply.) a. Alanine aminotransferase: biliary system b. Ammonia: liver c. Amylase: liver d. Lipase: pancreas e. Urine urobilinogen: stomach

ANS: B, D Alanine aminotransferase and ammonia are related to the liver. Amylase and lipase are related to the pancreas. Urobilinogen evaluates both hepatic and biliary function.

A circulating nurse has transferred an older client to the operating room. What action by the nurse is most important for this client? a. Allow the client to keep hearing aids in until anesthesia begins. b. Pad the table as appropriate for the surgical procedure. c. Position the client for maximum visualization of the site. d. Stay with the client, providing emotional comfort and support.

ANS: A Many older clients have sensory loss. To help prevent disorientation, facilities often allow older clients to keep their eyeglasses on and hearing aids in until the start of anesthesia. The other actions are appropriate for all operative clients.

A nurse obtains a focused health history for a client who is suspected of having bacterial meningitis. Which question should the nurse ask? a. "Do you live in a crowded residence?" b. "When was your last tetanus vaccination?" c. "Have you had any viral infections recently?" d. "Have you traveled out of the country in the last month?"

ANS: A Meningococcal meningitis tends to occur in multiple outbreaks. It is most likely to occur in areas of high-density population, such as college dormitories, prisons, and military barracks. A tetanus vaccination would not place the client at increased risk for meningitis or protect the client from meningitis. A viral infection would not lead to bacterial meningitis but could lead to viral meningitis. Simply knowing if the client traveled out of the country does not provide enough information. The nurse should ask about travel to specific countries in which the disease is common, for example, sub-Saharan Africa.

A client had a nerve laceration repair to the forearm and is being discharged in a cast. What statement by the client indicates a poor understanding of discharge instructions relating to cast care? a. "I can scratch with a coat hanger." b. "I should feel my fingers for warmth." c. "I will keep the cast clean and dry." d. "I will return to have the cast removed."

ANS: A Nothing should be placed under the cast to use for scratching. The other statements show good indication that the client has understood the discharge instructions.

A nurse assesses a client who has open lesions. Which action should the nurse take first? a. Put on gloves. b. Ask the client about his or her occupation. c. Assess the client's pain. d. Obtain vital signs.

ANS: A Nurses should wear gloves as part of Standard Precautions when examining skin that is not intact. The other options should be completed after gloves are put on.

After teaching a client with a spinal cord injury, the nurse assesses the client's understanding. Which client statement indicates a correct understanding of how to prevent respiratory problems at home? a. "I'll use my incentive spirometer every 2 hours while I'm awake." b. "I'll drink thinned fluids to prevent choking." c. "I'll take cough medicine to prevent excessive coughing." d. "I'll position myself on my right side so I don't aspirate."

ANS: A Often, the person with a spinal cord injury will have weak intercostal muscles and is at higher risk for developing atelectasis and stasis pneumonia. Using an incentive spirometer every 2 hours helps the client expand the lungs more fully and prevents atelectasis. Clients should drink fluids that they can tolerate; usually thick fluids are easier to tolerate. The client should be encouraged to cough and clear secretions. Clients should be placed in high-Fowler's position to prevent aspiration.

An older client is hospitalized after an operation. When assessing the client for postoperative infection, the nurse places priority on which assessment? a. Change in behavior b. Daily white blood cell count c. Presence of fever and chills d. Tolerance of increasing activity

ANS: A Older people have an age-related decrease in immune system functioning and may not show classic signs of infection such as increased white blood cell count, fever and chills, or obvious localized signs of infection. A change in behavior often signals an infection or onset of other illness in the older client.

A client hospitalized for chemotherapy has a hemoglobin of 6.1 mg/dL. What medication should the nurse prepare to administer? a. Epoetin alfa (Epogen) b. Filgrastim (Neupogen) c. Mesna (Mesnex) d. Oprelvekin (Neumega)

ANS: A The client's hemoglobin is low, so the nurse should prepare to administer epoetin alfa, a colony-stimulating factor that increases production of red blood cells. Filgrastim is for neutropenia. Mesna is used to decrease bladder toxicity from some chemotherapeutic agents. Oprelvekin is used to increase platelet count.

A nurse reads on a hospitalized client's chart that the client is receiving teletherapy. What action by the nurse is best? a. Coordinate continuation of the therapy. b. Place the client on radiation precautions. c. No action by the nurse is needed at this time. d. Restrict visitors to only adults over age 18.

ANS: A The client needs to continue with radiation therapy, and the nurse can coordinate this with the appropriate department. The client is not radioactive, so radiation precautions and limiting visitors are not necessary.

A client is scheduled for a colonoscopy and the nurse has provided instructions on the bowel cleansing regimen. What statement by the client indicates a need for further teaching? a. "It's a good thing I love orange and cherry gelatin." b. "My spouse will be here to drive me home." c. "I should refrigerate the GoLYTELY before use." d. "I will buy a case of Gatorade before the prep."

ANS: A The client should be advised to avoid beverages and gelatin that are red, orange, or purple in color as their residue can appear to be blood. The other statements show a good understanding of the preparation for the procedure.

A nurse cares for a client who is recovering from laparoscopic cholecystectomy surgery. The client reports pain in the shoulder blades. How should the nurse respond? a. "Ambulating in the hallway twice a day will help." b. "I will apply a cold compress to the painful area on your back." c. "Drinking a warm beverage can relieve this referred pain." d. "You should cough and deep breathe every hour."

ANS: A The client who has undergone a laparoscopic cholecystectomy may report free air pain due to retention of carbon dioxide in the abdomen. The nurse assists the client with early ambulation to promote absorption of the carbon dioxide. Cold compresses and drinking a warm beverage would not be helpful. Coughing and deep breathing are important postoperative activities, but they are not related to discomfort from carbon dioxide.

A client is getting out of bed into the chair for the first time after an uncemented hip replacement. What action by the nurse is most important? a. Have adequate help to transfer the client. b. Provide socks so the client can slide easier. c. Tell the client full weight bearing is allowed. d. Use a footstool to elevate the client's leg.

ANS: A The client with an uncemented hip will be on toe-touch only right after surgery. The nurse should ensure there is adequate help to transfer the client while preventing falls. Slippery socks will encourage a fall. Elevating the leg greater than 90 degrees is not allowed.

A client has dumping syndrome after a partial gastrectomy. Which action by the nurse would be most helpful? a. Arrange a dietary consult. b. Increase fluid intake. c. Limit the client's foods. d. Make the client NPO.

ANS: A The client with dumping syndrome after a gastrectomy has multiple dietary needs. A referral to the registered dietitian will be extremely helpful. Food and fluid intake is complicated and needs planning. The client should not be NPO.

A nurse collaborates with an unlicensed assistive personnel (UAP) to provide care for a client who is prescribed a 24-hour urine specimen collection. Which statement should the nurse include when delegating this activity to the UAP? a. "Note the time of the client's first void and collect urine for 24 hours." b. "Add the preservative to the container at the end of the test." c. "Start the collection by saving the first urine of the morning." d. "It is okay if one urine sample during the 24 hours is not collected."

ANS: A The collection of a 24-hour urine specimen is often delegated to a UAP. The nurse must ensure that the UAP understands the proper process for collecting the urine. The 24-hour urine collection specimen is started after the client's first urination. The first urine specimen is discarded because there is no way to know how long it has been in the bladder, but the time of the client's first void is noted. The client adds all urine voided after that first discarded specimen during the next 24 hours. When the 24-hour mark is reached, the client voids one last time and adds this specimen to the collection. The preservative, if used, must be added to the container at the beginning of the collection. All urine samples need to be collected for the test results to be accurate.

A nurse assesses a client who is admitted with inflamed soft-tissue folds around the nail plates. Which question should the nurse ask to elicit useful information about the possible condition? a. "What do you do for a living?" b. "Are your nails professionally manicured?" c. "Do you have diabetes mellitus?" d. "Have you had a recent fungal infection?"

ANS: A The condition chronic paronychia is common in people with frequent intermittent exposure to water, such as homemakers, bartenders, and laundry workers. The other questions would not provide information specifically related to this assessment finding.

A client in the emergency department is having a stroke and needs a carotid artery angioplasty with stenting. The client's mental status is deteriorating. What action by the nurse is most appropriate? a. Attempt to find the family to sign a consent. b. Inform the provider that the procedure cannot occur. c. Nothing; no consent is needed in an emergency. d. Sign the consent form for the client.

ANS: A The nurse should attempt to find the family to give consent. If no family is present or can be found, under the principle of emergency consent, a life-saving procedure can be performed without formal consent. The nurse should not just sign the consent form.

A nurse cares for a client who has a new colostomy. Which action should the nurse take? a. Empty the pouch frequently to remove excess gas collection. b. Change the ostomy pouch and wafer every morning. c. Allow the pouch to completely fill with stool prior to emptying it. d. Use surgical tape to secure the pouch and prevent leakage.

ANS: A The nurse should empty the new ostomy pouch frequently because of excess gas collection, and empty the pouch when it is one-third to one-half full of stool. The ostomy pouch does not need to be changed every morning. Ostomy wafers with paste should be used to secure and seal the ostomy appliance; surgical tape should not be used.

A nurse cares for a client who had a colostomy placed in the ascending colon 2 weeks ago. The client states, "The stool in my pouch is still liquid." How should the nurse respond? a. "The stool will always be liquid with this type of colostomy." b. "Eating additional fiber will bulk up your stool and decrease diarrhea." c. "Your stool will become firmer over the next couple of weeks." d. "This is abnormal. I will contact your health care provider."

ANS: A The stool from an ascending colostomy can be expected to remain liquid because little large bowel is available to reabsorb the liquid from the stool. This finding is not abnormal. Liquid stool from an ascending colostomy will not become firmer with the addition of fiber to the client's diet or with the passage of time.

A client has a traumatic brain injury. The nurse assesses the following: pulse change from 82 to 60 beats/min, pulse pressure increase from 26 to 40 mm Hg, and respiratory irregularities. What action by the nurse takes priority? a. Call the provider or Rapid Response Team. b. Increase the rate of the IV fluid administration. c. Notify respiratory therapy for a breathing treatment. d. Prepare to give IV pain medication.

ANS: A These manifestations indicate Cushing's syndrome, a potentially life-threatening increase in intracranial pressure (ICP), which is an emergency. Immediate medical attention is necessary, so the nurse notifies the provider or the Rapid Response Team. Increasing fluids would increase the ICP. The client does not need a breathing treatment or pain medication.

A client who had a severe traumatic brain injury is being discharged home, where the spouse will be a full-time caregiver. What statement by the spouse would lead the nurse to provide further education on home care? a. "I know I can take care of all these needs by myself." b. "I need to seek counseling because I am very angry." c. "Hopefully things will improve gradually over time." d. "With respite care and support, I think I can do this."

ANS: A This caregiver has unrealistic expectations about being able to do everything without help. Acknowledging anger and seeking counseling show a realistic outlook and plans for accomplishing goals. Hoping for improvement over time is also realistic, especially with the inclusion of the word "hopefully." Realizing the importance of respite care and support also is a realistic outlook.

The nurse on the postoperative inpatient unit assesses a client after a total hip replacement. The client's surgical leg is visibly shorter than the other one and the client reports extreme pain. While a co-worker calls the surgeon, what action by the nurse is best? a. Assess neurovascular status in both legs. b. Elevate the affected leg and apply ice. c. Prepare to administer pain medication. d. Try to place the affected leg in abduction.

ANS: A This client has manifestations of hip dislocation, a critical complication of this surgery. Hip dislocation can cause neurovascular compromise. The nurse should assess neurovascular status, comparing both legs. The nurse should not try to move the extremity to elevate or abduct it. Pain medication may be administered if possible, but first the nurse should thoroughly assess the client.

A client with a history of prostate cancer is in the clinic and reports new onset of severe low back pain. What action by the nurse is most important? a. Assess the client's gait and balance. b. Ask the client about the ease of urine flow. c. Document the report completely. d. Inquire about the client's job risks.

ANS: A This client has manifestations of spinal cord compression, which can be seen with prostate cancer. This may affect both gait and balance and urinary function. For client safety, assessing gait and balance is the priority. Documentation should be complete. The client may or may not have occupational risks for low back pain, but with his history of prostate cancer, this should not be where the nurse starts investigating.

A nurse assesses a client in the preoperative holding area and finds brittle nails and hair, dry skin turgor, and muscle wasting. What action by the nurse is best? a. Consult the surgeon about a postoperative dietitian referral. b. Document the findings thoroughly in the client's chart. c. Encourage the client to eat more after recovering from surgery. d. Refer the client to Meals on Wheels after discharge.

ANS: A This client has signs of malnutrition, which can impact recovery from surgery. The nurse should consult the surgeon about prescribing a consultation with a dietitian in the postoperative period. The nurse should document the findings but needs to do more. Encouraging the client to eat more may be helpful, but the client needs a professional nutritional assessment so that the appropriate diet and supplements can be ordered. The client may or may not need Meals on Wheels after discharge.

A nurse is teaching a client with psoriatic arthritis about the medication golimumab (Simponi). What information is most important to include? a. "Avoid large crowds or people who are ill." b. "Stay upright for 1 hour after taking this drug." c. "This drug may cause your hair to fall out." d. "You may double the dose if pain is severe."

ANS: A This drug has a Food and Drug Administration black box warning about opportunistic or other serious infections. Teach the client to avoid large crowds and people who are ill. The other instructions do not pertain to golimumab.

What action by the perioperative nursing staff is most important to prevent surgical wound infection in a client having a total joint replacement? a. Administer preoperative antibiotic as ordered. b. Assess the client's white blood cell count. c. Instruct the client to shower the night before. d. Monitor the client's temperature postoperatively.

ANS: A To prevent surgical wound infection, antibiotics are given preoperatively within an hour of surgery. Simply taking a shower will not help prevent infection unless the client is told to use special antimicrobial soap. The other options are processes to monitor for infection, not prevent it.

A nurse teaches a client with hepatitis C who is prescribed ribavirin (Copegus). Which statement should the nurse include in this client's discharge education? a. "Use a pill organizer to ensure you take this medication as prescribed." b. "Transient muscle aching is a common side effect of this medication." c. "Follow up with your provider in 1 week to test your blood for toxicity." d. "Take your radial pulse for 1 minute prior to taking this medication."

ANS: A Treatment of hepatitis C with ribavirin takes up to 48 weeks, making compliance a serious issue. The nurse should work with the client on a strategy to remain compliant for this length of time. Muscle aching is not a common side effect. The client will be on this medication for many weeks and does not need a blood toxicity examination. There is no need for the client to assess his or her radial pulse prior to taking the medication.

A nurse witnesses a client with late-stage Alzheimer's disease eat breakfast. Afterward the client states, "I am hungry and want breakfast." How should the nurse respond? a. "I see you are still hungry. I will get you some toast." b. "You ate your breakfast 30 minutes ago." c. "It appears you are confused this morning." d. "Your family will be here soon. Let's get you dressed."

ANS: A Use of validation therapy with clients who have Alzheimer's disease involves acknowledgment of the client's feelings and concerns. This technique has proved more effective in later stages of the disease, when using reality orientation only increases agitation. Telling the client that he or she already ate breakfast may agitate the client. The other statements do not validate the client's concerns.

A nurse is seeing many clients in the neurosurgical clinic. With which clients should the nurse plan to do more teaching? (Select all that apply.) a. Client with an aneurysm coil placed 2 months ago who is taking ibuprofen (Motrin) for sinus headaches b. Client with an aneurysm clip who states that his family is happy there is no chance of recurrence c. Client who had a coil procedure who says that there will be no problem following up for 1 year d. Client who underwent a flow diversion procedure 3 months ago who is taking docusate sodium (Colace) for constipation e. Client who underwent surgical aneurysm ligation 3 months ago who is planning to take a Caribbean cruise

ANS: A, B After a coil procedure, up to 20% of clients experience re-bleeding in the first year. The client with this coil should not be taking drugs that interfere with clotting. An aneurysm clip can move up to 5 years after placement, so this client and family need to be watchful for changing neurologic status. The other statements show good understanding.

A nurse assesses clients who have endocrine disorders. Which assessment findings are paired correctly with the endocrine disorder? (Select all that apply.) a. Excessive thyroid-stimulating hormone - Increased bone formation b. Excessive melanocyte-stimulating hormone - Darkening of the skin c. Excessive parathyroid hormone - Synthesis and release of corticosteroids d. Excessive antidiuretic hormone - Increased urinary output e. Excessive adrenocorticotropic hormone - Increased bone resorption

ANS: A, B Thyroid-stimulating hormone targets thyroid tissue and stimulates the formation of bone. Melanocyte-stimulating hormone stimulates melanocytes and promotes pigmentation or the darkening of the skin. Parathyroid hormone stimulates bone resorption. Antidiuretic hormone targets the kidney and promotes water reabsorption, causing a decrease in urinary output. Adrenocorticotropic hormone targets the adrenal cortex and stimulates the synthesis and release of corticosteroids.

A nurse provides diabetic education at a public health fair. Which disorders should the nurse include as complications of diabetes mellitus? (Select all that apply.) a. Stroke b. Kidney failure c. Blindness d. Respiratory failure e. Cirrhosis

ANS: A, B, C Complications of diabetes mellitus are caused by macrovascular and microvascular changes. Macrovascular complications include coronary artery disease, cerebrovascular disease, and peripheral vascular disease. Microvascular complications include nephropathy, retinopathy, and neuropathy. Respiratory failure and cirrhosis are not complications of diabetes mellitus.

A nurse assesses clients on a medical-surgical unit. Which clients should the nurse identify as at risk for secondary seizures? (Select all that apply.) a. A 26-year-old woman with a left temporal brain tumor b. A 38-year-old male client in an alcohol withdrawal program c. A 42-year-old football player with a traumatic brain injury d. A 66-year-old female client with multiple sclerosis e. A 72-year-old man with chronic obstructive pulmonary disease

ANS: A, B, C Clients at risk for secondary seizures include those with a brain lesion from a tumor or trauma, and those who are experiencing a metabolic disorder, acute alcohol withdrawal, electrolyte disturbances, and high fever. Clients with a history of stroke, heart disease, and substance abuse are also at risk. Clients with multiple sclerosis or chronic obstructive pulmonary disease are not at risk for secondary seizures.

A nurse cares for clients with hormone disorders. Which are common key features of hormones? (Select all that apply.) a. Hormones may travel long distances to get to their target tissues. b. Continued hormone activity requires continued production and secretion. c. Control of hormone activity is caused by negative feedback mechanisms. d. Most hormones are stored in the target tissues for use later. e. Most hormones cause target tissues to change activities by changing gene activity.

ANS: A, B, C Hormones are secreted by endocrine glands and travel through the body to reach their target tissues. Hormone activity can increase or decrease according to the body's needs, and continued hormone activity requires continued production and secretion. Control is maintained via negative feedback. Hormones are not stored for later use, and they do not alter genetic activity

A nurse is preparing to administer IV chemotherapy. What supplies does this nurse need? (Select all that apply.) a. "Chemo" gloves b. Facemask c. Isolation gown d. N95 respirator e. Shoe covers

ANS: A, B, C The Occupational Safety and Health Administration (OSHA) and the Oncology Nurses Society have developed safety guidelines for those preparing or administering IV chemotherapy. These include double gloves (or "chemo" gloves), a facemask, and a gown. An N95 respirator and shoe covers are not required.

A client's family members are concerned that telling the client about a new finding of cancer will cause extreme emotional distress. They approach the nurse and ask if this can be kept from the client. What actions by the nurse are most appropriate? (Select all that apply.) a. Ask the family to describe their concerns more fully. b. Consult with a social worker, chaplain, or ethics committee. c. Explain the client's right to know and ask for their assistance. d. Have the unit manager take over the care of this client and family. e. Tell the family that this secret will not be kept from the client.

ANS: A, B, C The client's right of autonomy means that the client must be fully informed as to his or her diagnosis and treatment options. The nurse cannot ethically keep this information from the client. The nurse can ask the family to explain their concerns more fully so everyone understands the concerns. A social worker, chaplain, or ethics committee can become involved to assist the nurse, client, and family. The nurse should explain the client's right to know and ask the family how best to proceed. The nurse should not abdicate responsibility for this difficult situation by transferring care to another nurse. Simply telling the family that he or she will not keep this secret sets up an adversarial relationship. Explaining this fact along with the concept of autonomy would be acceptable, but this by itself is not.

The nurse working with oncology clients understands that interacting factors affect cancer development. Which factors does this include? (Select all that apply.) a. Exposure to carcinogens b. Genetic predisposition c. Immune function d. Normal doubling time e. State of euploidy

ANS: A, B, C The three interacting factors needed for cancer development are exposure to carcinogens, genetic predisposition, and immune function.

The nurse working in the emergency department assesses a client who has symptoms of stroke. For what modifiable risk factors should the nurse assess? (Select all that apply.) a. Alcohol intake b. Diabetes c. High-fat diet d. Obesity e. Smoking

ANS: A, C, D, E Alcohol intake, a high-fat diet, obesity, and smoking are all modifiable risk factors for stroke. Diabetes is not modifiable but is a risk factor that can be controlled with medical intervention.

The nurse has taught a client about lifestyle modifications for gastroesophageal reflux disease (GERD). What statements by the client indicate good understanding of the teaching? (Select all that apply.) a. "I just joined a gym, so I hope that helps me lose weight." b. "I sure hate to give up my coffee, but I guess I have to." c. "I will eat three small meals and three small snacks a day." d. "Sitting upright and not lying down after meals will help." e. "Smoking a pipe is not a problem and I don't have to stop."

ANS: A, B, C, D Lifestyle modifications can help control GERD and include losing weight if needed; avoiding chocolate, caffeine, and carbonated beverages; eating frequent small meals or snacks; and remaining upright after meals. Tobacco is a risk factor for GERD and should be avoided in all forms.

The nurse is aware that which factors are related to the development of gastroesophageal reflux disease (GERD)? (Select all that apply.) a. Delayed gastric emptying b. Eating large meals c. Hiatal hernia d. Obesity e. Viral infections

ANS: A, B, C, D Many factors predispose a person to GERD, including delayed gastric emptying, eating large meals, hiatal hernia, and obesity. Viral infections are not implicated in the development of GERD, although infection with Helicobacter pylori is.

A client has a small-bore feeding tube (Dobhoff tube) inserted for continuous enteral feedings while recovering from a traumatic brain injury. What actions should the nurse include in the client's care? (Select all that apply.) a. Assess tube placement per agency policy. b. Keep the head of the bed elevated at least 30 degrees. c. Listen to lung sounds at least every 4 hours. d. Run continuous feedings on a feeding pump. e. Use blue dye to determine proper placement.

ANS: A, B, C, D All of these options are important for client safety when continuous enteral feedings are in use. Blue dye is not used because it can cause lung injury if aspirated.

A client with fibromyalgia is in the hospital for an unrelated issue. The client reports that sleep, which is always difficult, is even harder now. What actions by the nurse are most appropriate? (Select all that apply.) a. Allow the client uninterrupted rest time. b. Assess the client's usual bedtime routine. c. Limit environmental noise as much as possible. d. Offer a massage or warm shower at night. e. Request an order for a strong sleeping pill.

ANS: A, B, C, D Clients with fibromyalgia often have sleep disturbances, which can be exacerbated by the stress, noise, and unfamiliar environment of the hospital. Allowing uninterrupted rest time, adhering to the client's usual bedtime routine as much as possible, limiting noise and light, and offering massages or warm showers can help. The client does not need a strong sleeping pill unless all other options fail and the client requests something for sleep. At that point a mild sleeping agent can be tried.

The nurse working in the gastrointestinal clinic sees clients who are anemic. What are common causes for which the nurse assesses in these clients? (Select all that apply.) a. Colon cancer b. Diverticulitis c. Inflammatory bowel disease d. Peptic ulcer disease e. Pernicious anemia

ANS: A, B, C, D In adults, the most common cause of anemia is GI bleeding. This is commonly associated with colon cancer, diverticulitis, inflammatory bowel disease, and peptic ulcer disease. Pernicious anemia is not associated with GI bleeding.

A student nurse asks why older adults are at higher risk for complications after surgery. What reasons does the registered nurse give? (Select all that apply.) a. Decreased cardiac output b. Decreased oxygenation c. Frequent nocturia d. Mobility alterations e. Inability to adapt to changes

ANS: A, B, C, D Older adults have many age-related physiologic changes that put them at higher risk of falling and other complications after surgery. Some of these include decreased cardiac output, decreased oxygenation of tissues, nocturia, and mobility alterations. They also have a decreased ability to adapt to new surroundings, but that is not the same as being unable to adapt.

What actions by the circulating nurse are important to promote client comfort? (Select all that apply.) a. Correct positioning b. Introducing one's self c. Providing warmth d. Remaining present e. Removing hearing aids

ANS: A, B, C, D The circulating nurse can do many things to promote client comfort, including positioning the client correctly and comfortably, introducing herself or himself to the client, keeping the client warm, and remaining present with the client. Removing hearing aids does not promote comfort and, if the client is still awake when they are removed, may contribute to disorientation and anxiety.

The student nurse learns about risk factors for gastric cancer. Which factors does this include? (Select all that apply.) a. Achlorhydria b. Chronic atrophic gastritis c. Helicobacter pylori infection d. Iron deficiency anemia e. Pernicious anemia

ANS: A, B, C, E Achlorhydria, chronic atrophic gastritis, H. pylori infection, and pernicious anemia are all risk factors for developing gastric cancer. Iron deficiency anemia is not a risk factor.

The circulating nurse reviews the day's schedule and notes clients who are at higher risk of anesthetic overdose and other anesthesia-related complications. Which clients does this include? (Select all that apply.) a. A 75-year-old client scheduled for an elective procedure b. Client who drinks a 6-pack of beer each day c. Client with a serum creatinine of 3.8 mg/dL d. Client who is taking birth control pills e. Young male client with a RYR1 gene mutation

ANS: A, B, C, E People at higher risk for anesthetic overdose or other anesthesia-related complications include people with a slowed metabolism (older adults generally have slower metabolism than younger adults), those with kidney or liver impairments, and those with mutations of the RYR1 gene. Drinking a 6-pack of beer per day possibly indicates some liver disease; a creatinine of 3.8 is high, indicating renal disease; and the genetic mutation increases the chance of malignant hyperthermia. Taking birth control pills is not a risk factor.

The student nurse studying stomach disorders learns that the risk factors for acute gastritis include which of the following? (Select all that apply.) a. Alcohol b. Caffeine c. Corticosteroids d. Fruit juice e. Nonsteroidal anti-inflammatory drugs (NSAIDs)

ANS: A, B, C, E Risk factors for acute gastritis include alcohol, caffeine, corticosteroids, and chronic NSAID use. Fruit juice is not a risk factor, although in some people it does cause distress.

A nurse assesses a client with early-onset multiple sclerosis (MS). Which clinical manifestation should the nurse expect to find? a. Hyperresponsive reflexes b. Excessive somnolence c. Nystagmus d. Heat intolerance

ANS: C Early signs and symptoms of MS include changes in motor skills, vision, and sensation. Hyperresponsive reflexes, excessive somnolence, and heat intolerance are later manifestations of MS.

The nurse working with older clients understands age-related changes in the gastrointestinal system. Which changes does this include? (Select all that apply.) a. Decreased hydrochloric acid production b. Diminished sensation that can lead to constipation c. Fat not digested as well in older adults d. Increased peristalsis in the large intestine e. Pancreatic vessels become calcified

ANS: A, B, C, E Several age-related changes occur in the gastrointestinal system. These include decreased hydrochloric acid production, diminished nerve function that leads to decreased sensation of the need to pass stool, decreased fat digestion, decreased peristalsis in the large intestine, and calcification of pancreatic vessels.

A nurse is providing community education on the seven warning signs of cancer. Which signs are included? (Select all that apply.) a. A sore that does not heal b. Changes in menstrual patterns c. Indigestion or trouble swallowing d. Near-daily abdominal pain e. Obvious change in a mole

ANS: A, B, C, E The seven warning signs for cancer can be remembered with the acronym CAUTION: changes in bowel or bladder habits, a sore that does not heal, unusual bleeding or discharge, thickening or lump in the breast or elsewhere, indigestion or difficulty swallowing, obvious change in a wart or mole, and nagging cough or hoarseness. Abdominal pain is not a warning sign.

A nurse collaborates with the interdisciplinary team to develop a plan of care for a client who is newly diagnosed with diabetes mellitus. Which team members should the nurse include in this interdisciplinary team meeting? (Select all that apply.) a. Registered dietitian b. Clinical pharmacist c. Occupational therapist d. Health care provider e. Speech-language pathologist

ANS: A, B, D When planning care for a client newly diagnosed with diabetes mellitus, the nurse should collaborate with a registered dietitian, clinical pharmacist, and health care provider. The focus of treatment for a newly diagnosed client would be nutrition, medication therapy, and education. The nurse could also consult with a diabetic educator. There is no need for occupational therapy or speech therapy at this time.

A nurse assesses a client with hypothyroidism who is admitted with acute appendicitis. The nurse notes that the client's level of consciousness has decreased. Which actions should the nurse take? (Select all that apply.) a. Infuse intravenous fluids. b. Cover the client with warm blankets. c. Monitor blood pressure every 4 hours. d. Maintain a patent airway. e. Administer oral glucose as prescribed.

ANS: A, B, D A client with hypothyroidism and an acute illness is at risk for myxedema coma. A decrease in level of consciousness is a symptom of myxedema. The nurse should infuse IV fluids, cover the client with warm blankets, monitor blood pressure every hour, maintain a patent airway, and administer glucose intravenously as prescribed.

A client has thrombocytopenia. What actions does the nurse delegate to the unlicensed assistive personnel (UAP)? (Select all that apply.) a. Apply the client's shoes before getting the client out of bed. b. Assist the client with ambulation. c. Shave the client with a safety razor only. d. Use a lift sheet to move the client up in bed. e. Use the Waterpik on a low setting for oral care.

ANS: A, B, D Clients with thrombocytopenia are at risk of significant bleeding even with minor injuries. The nurse instructs the UAP to put the client's shoes on before getting the client out of bed, assist with ambulation, shave the client with an electric razor, use a lift sheet when needed to reposition the client, and use a soft-bristled toothbrush for oral care.

A client has rheumatoid arthritis (RA) and the visiting nurse is conducting a home assessment. What options can the nurse suggest for the client to maintain independence in activities of daily living (ADLs)? (Select all that apply.) a. Grab bars to reach high items b. Long-handled bath scrub brush c. Soft rocker-recliner chair d. Toothbrush with built-up handle e. Wheelchair cushion for comfort

ANS: A, B, D Grab bars, long-handled bath brushes, and toothbrushes with built-up handles all provide modifications for daily activities, making it easier for the client with RA to complete ADLs independently. The rocker-recliner and wheelchair cushion are comfort measures but do not help increase independence.

A client with myasthenia gravis is prescribed pyridostigmine (Mestinon). What teaching should the nurse plan regarding this medication? (Select all that apply.) a. "Do not eat a full meal for 45 minutes after taking the drug." b. "Seek immediate care if you develop trouble swallowing." c. "Take this drug on an empty stomach for best absorption." d. "The dose may change frequently depending on symptoms." e. "Your urine may turn a reddish-orange color while on this drug."

ANS: A, B, D Pyridostigmine should be given with a small amount of food to prevent GI upset, but the client should wait to eat a full meal due to the potential for aspiration. If difficulty with swallowing occurs, the client should seek immediate attention. The dose can change on a day-to-day basis depending on the client's manifestations. Taking the drug on an empty stomach is not related although the client needs to eat within 45 to 60 minutes afterwards. The client's urine will not turn reddish-orange while on this drug.

A client has been diagnosed with fibromyalgia syndrome but does not want to take the prescribed medications. What nonpharmacologic measures can the nurse suggest to help manage this condition? (Select all that apply.) a. Acupuncture b. Stretching c. Supplements d. Tai chi e. Vigorous aerobics

ANS: A, B, D There are many nonpharmacologic means for controlling the symptoms of fibromyalgia, including acupuncture, stretching, tai chi, low-impact aerobics, swimming, biking, strengthening, massage, stress management, and hypnosis. Dietary supplements and vigorous aerobics are not recommended.

A home health care nurse is visiting a client discharged home after a hip replacement. The client is still on partial weight bearing and using a walker. What safety precautions can the nurse recommend to the client? (Select all that apply.) a. Buy and install an elevated toilet seat. b. Install grab bars in the shower and by the toilet. c. Step into the bathtub with the affected leg first. d. Remove all throw rugs throughout the house. e. Use a shower chair while taking a shower.

ANS: A, B, D, E Buying and installing an elevated toilet seat, installing grab bars, removing throw rugs, and using a shower chair will all promote safety for this client. The client is still on partial weight bearing, so he or she cannot step into the bathtub leading with the operative side. Stepping into a bathtub may also require the client to bend the hip more than the allowed 90 degrees.

A nurse is working with many stroke clients. Which clients would the nurse consider referring to a mental health provider on discharge? (Select all that apply.) a. Client who exhibits extreme emotional lability b. Client with an initial National Institutes of Health (NIH) Stroke Scale score of 38 c. Client with mild forgetfulness and a slight limp d. Client who has a past hospitalization for a suicide attempt e. Client who is unable to walk or eat 3 weeks post-stroke

ANS: A, B, D, E Clients most at risk for post-stroke depression are those with a previous history of depression, severe stroke (NIH Stroke Scale score of 38 is severe), and post-stroke physical or cognitive impairment. The client with mild forgetfulness and a slight limp would be a low priority for this referral.

A nurse is teaching a female client with rheumatoid arthritis (RA) about taking methotrexate (MTX) (Rheumatrex) for disease control. What information does the nurse include? (Select all that apply.) a. "Avoid acetaminophen in over-the-counter medications." b. "It may take several weeks to become effective on pain." c. "Pregnancy and breast-feeding are not affected by MTX." d. "Stay away from large crowds and people who are ill." e. "You may find that folic acid, a B vitamin, reduces side effects."

ANS: A, B, D, E MTX is a disease-modifying antirheumatic drug and is used as a first-line drug for RA. MTX can cause liver toxicity, so the client should be advised to avoid medications that contain acetaminophen. It may take 4 to 6 weeks for effectiveness. MTX can cause immunosuppression, so avoiding sick people and crowds is important. Folic acid helps reduce side effects for some people. Pregnancy and breast-feeding are contraindicated while on this drug.

A client has mucositis. What actions by the nurse will improve the client's nutrition? (Select all that apply.) a. Assist with rinsing the mouth with saline frequently. b. Encourage the client to eat room-temperature foods. c. Give the client hot liquids to hold in the mouth. d. Provide local anesthetic medications to swish and spit. e. Remind the client to brush teeth gently after each meal.

ANS: A, B, D, E Mucositis can interfere with nutrition. The nurse can help with rinsing the mouth frequently with water or saline; encouraging the client to eat cool, slightly warm, or room-temperature foods; providing swish-and-spit anesthetics; and reminding the client to keep the mouth clean by brushing gently after each meal. Hot liquids would be painful for the client.

A client has a gastrointestinal hemorrhage and is prescribed two units of packed red blood cells. What actions should the nurse perform prior to hanging the blood? (Select all that apply.) a. Ask a second nurse to double-check the blood. b. Prime the IV tubing with normal saline. c. Prime the IV tubing with dextrose in water. d. Take and record a set of vital signs. e. Teach the client about reaction manifestations.

ANS: A, B, D, E Prior to starting a blood transfusion, the nurse asks another nurse to double-check the blood (and client identity), primes the IV tubing with normal saline, takes and records a baseline set of vital signs, and teaches the client about manifestations to report. The IV tubing is not primed with dextrose in water.

A client is clearly uncomfortable and anxious in the preoperative holding room waiting for emergent abdominal surgery. What actions can the nurse perform to increase comfort? (Select all that apply.) a. Allow the client to assume a position of comfort. b. Allow the client's family to remain at the bedside. c. Give the client a warm, non-caffeinated drink. d. Provide warm blankets or cool washcloths as desired. e. Pull the curtains around the bed to provide privacy.

ANS: A, B, D, E There are many nonpharmacologic comfort measures the nurse can employ, such as allowing the client to remain in the position that is most comfortable, letting the family stay with the client, providing warmth or cooling measures as requested by the client, and providing privacy. The client in the preoperative holding area is NPO, so drinks should not be provided.

A nurse cares for older adult clients in a long-term acute care facility. Which interventions should the nurse implement to prevent skin breakdown in these clients? (Select all that apply.) a.Use a lift sheet when moving the client in bed. b.Avoid tape when applying dressings. c.Avoid whirlpool therapy. d.Use loose dressing on all wounds. e.Implement pressure-relieving devices.

ANS: A, B, E Using a lift sheet will prevent shearing forces from tearing skin. Tape should be avoided so that the skin won't tear. Using pressure-relieving devices for clients who are at risk for pressure ulcer formation, including older adults, is a proactive approach to prevent skin breakdown. No contraindication to using whirlpool therapy for the older client is known. Dressings should be applied as prescribed, not so loose that they do not provide required treatment, and not so tight that they decrease blood flow to tissues.

The nurse is working with clients who have connective tissue diseases. Which disorders are correctly paired with their manifestations? (Select all that apply.) a. Dry, scaly skin rash - Systemic lupus erythematosus (SLE) b. Esophageal dysmotility - Systemic sclerosis c. Excess uric acid excretion - Gout d. Footdrop and paresthesias - Osteoarthritis e. Vasculitis causing organ damage - Rheumatoid arthritis

ANS: A, B, E A dry, scaly skin rash is the most frequent dermatologic manifestation of SLE. Systemic sclerosis can lead to esophageal motility problems. Vasculitis leads to organ damage in rheumatoid arthritis. Gout is caused by hyperuricemia; the production of uric acid exceeds the excretion capability of the kidneys. Footdrop and paresthesias occur in rheumatoid arthritis.

A nurse teaches a client with hyperthyroidism. Which dietary modifications should the nurse include in this client's teaching? (Select all that apply.) a. Increased carbohydrates b. Decreased fats c. Increased calorie intake d. Supplemental vitamins e. Increased proteins

ANS: A, C, E The client is hypermetabolic and has an increased need for carbohydrates, calories, and proteins. Proteins are especially important because the client is at risk for a negative nitrogen balance. There is no need to decrease fat intake or take supplemental vitamins.

A nurse plans care for a client with a halo fixator. Which interventions should the nurse include in this client's plan of care? (Select all that apply.) a. Tape a halo wrench to the client's vest. b. Assess the pin sites for signs of infection. c. Loosen the pins when sleeping. d. Decrease the client's oral fluid intake. e. Assess the chest and back for skin breakdown.

ANS: A, B, E A special halo wrench should be taped to the client's vest in case of a cardiopulmonary emergency. The nurse should assess the pin sites for signs of infection or loose pins and for complications from the halo. The nurse should also increase fluids and fiber to decrease bowel straining and assess the client's chest and back for skin breakdown from the halo vest.

After teaching a client with a spinal cord tumor, the nurse assesses the client's understanding. Which statements by the client indicate a correct understanding of the teaching? (Select all that apply.) a. "Even though turning hurts, I will remind you to turn me every 2 hours." b. "Radiation therapy can shrink the tumor but also can cause more problems." c. "Surgery will be scheduled to remove the tumor and reverse my symptoms." d. "I put my affairs in order because this type of cancer is almost always fatal." e. "My family is moving my bedroom downstairs for when I am discharged home."

ANS: A, B, E Although surgery may relieve symptoms by reducing pressure on the spine and debulking the tumor, some motor and sensory deficits may remain. Spinal tumors usually cause disability but are not usually fatal. Radiation therapy is often used to shrink spinal tumors but can cause progressive spinal cord degeneration and neurologic deficits. The client should be turned every 2 hours to prevent skin breakdown and arrangements should be made at home so that the client can complete activities of daily living without needing to go up and down stairs.

A nurse assesses a client who is experiencing a cluster headache. Which clinical manifestations should the nurse expect to find? (Select all that apply.) a. Ipsilateral tearing of the eye b. Miosis c. Abrupt loss of consciousness d. Neck and shoulder tenderness e. Nasal congestion f. Exophthalmos

ANS: A, B, E Cluster headache is usually accompanied by ipsilateral tearing, miosis, rhinorrhea or nasal congestion, ptosis, eyelid edema, and facial sweating. Abrupt loss of consciousness, neck and shoulder tenderness, and exophthalmos are not associated with cluster headaches.

The nurse caring for a client with Guillain-Barré syndrome has identified the priority client problem of decreased mobility for the client. What actions by the nurse are best? (Select all that apply.) a. Ask occupational therapy to help the client with activities of daily living. b. Consult with the provider about a physical therapy consult. c. Provide the client with information on support groups. d. Refer the client to a medical social worker or chaplain. e. Work with speech therapy to design a high-protein diet.

ANS: A, B, E Improving mobility and strength involves the collaborative assistance of occupational therapy, physical therapy, and speech therapy. While support groups, social work, or chaplain referrals may be needed, they do not help with mobility.

A nurse assesses a client with irritable bowel syndrome (IBS). Which questions should the nurse include in this client's assessment? (Select all that apply.) a. "Which food types cause an exacerbation of symptoms?" b. "Where is your pain and what does it feel like?" c. "Have you lost a significant amount of weight lately?" d. "Are your stools soft, watery, and black in color?" e. "Do you experience nausea associated with defecation?"

ANS: A, B, E The nurse should ask the client about factors that may cause exacerbations of symptoms, including food, stress, and anxiety. The nurse should also assess the location, intensity, and quality of the client's pain, and nausea associated with defecation or meals. Clients who have IBS do not usually lose weight and stools are not black in color.

A nurse assesses a male client who has symptoms of cirrhosis. Which questions should the nurse ask to identify potential factors contributing to this laboratory result? (Select all that apply.) a. "How frequently do you drink alcohol?" b. "Have you ever had sex with a man?" c. "Do you have a family history of cancer?" d. "Have you ever worked as a plumber?" e. "Were you previously incarcerated?"

ANS: A, B, E When assessing a client with suspected cirrhosis, the nurse should ask about alcohol consumption, including amount and frequency; sexual history and orientation (specifically men having sex with men); illicit drug use; history of tattoos; and history of military service, incarceration, or work as a firefighter, police officer, or health care provider. A family history of cancer and work as a plumber do not put the client at risk for cirrhosis.

A nurse teaches a client with diabetes mellitus about foot care. Which statements should the nurse include in this client's teaching? (Select all that apply.) a. "Do not walk around barefoot." b. "Soak your feet in a tub each evening." c. "Trim toenails straight across with a nail clipper." d. "Treat any blisters or sores with Epsom salts." e. "Wash your feet every other day."

ANS: A, C Clients who have diabetes mellitus are at high risk for wounds on the feet secondary to peripheral neuropathy and poor arterial circulation. The client should be instructed to not walk around barefoot or wear sandals with open toes. These actions place the client at higher risk for skin breakdown of the feet. The client should be instructed to trim toenails straight across with a nail clipper. Feet should be washed daily with lukewarm water and soap, but feet should not be soaked in the tub. The client should contact the provider immediately if blisters or sores appear and should not use home remedies to treat these wounds.

A client is having shoulder surgery with regional anesthesia. What actions by the nurse are most important to enhance client safety related to this anesthesia? (Select all that apply.) a. Assessing distal circulation to the operative arm after positioning b. Keeping the client warm during the operative procedure c. Padding the client's shoulder and arm on the operating table d. Preparing to suction the client's airway if the client vomits e. Speaking in a low, quiet voice as anesthesia is administered

ANS: A, C After regional anesthesia is administered, the client loses all sensation distally. The nurse ensures client safety by assessing distal circulation and padding the shoulder and arm appropriately. Although awake, the client will not be able to report potential injury. Keeping the client warm is not related to this anesthesia, nor is suctioning or speaking quietly.

A nurse is caring for a client with meningitis. Which laboratory values should the nurse monitor to identify potential complications of this disorder? (Select all that apply.) a. Sodium level b. Liver enzymes c. Clotting factors d. Cardiac enzymes e. Creatinine level

ANS: A, C Inflammation associated with meningitis can stimulate the hypothalamus and result in excessive production of antidiuretic hormone. The nurse should monitor sodium levels for early identification of syndrome of inappropriate antidiuretic hormone. A systemic inflammatory response (SIR) can also occur with meningitis. A SIR can result in a coagulopathy that leads to disseminated intravascular coagulation. The nurse should monitor clotting factors to identify this complication. The other laboratory values are not specific to complications of meningitis.

A client on interferon therapy is reporting severe skin itching and irritation. What actions does the nurse delegate to the unlicensed assistive personnel (UAP)? (Select all that apply.) a. Apply moisturizers to dry skin. b. Apply steroid creams to the skin. c. Bathe the client using mild soap. d. Help the client with a hot water bath. e. Teach the client to avoid sunlight.

ANS: A, C The nurse can delegate applying unscented moisturizer and using mild soap for bathing. Steroid creams are not used for this condition. Hot water will worsen the irritation. Client teaching is a nursing function.

A nurse plans care for a client who has hepatopulmonary syndrome. Which interventions should the nurse include in this client's plan of care? (Select all that apply.) a. Oxygen therapy b. Prone position c. Feet elevated on pillows d. Daily weights e. Physical therapy

ANS: A, C, D Care for a client who has hepatopulmonary syndrome should include oxygen therapy, the head of bed elevated at least 30 degrees or as high as the client wants to improve breathing, elevated feet to decrease dependent edema, and daily weights. There is no need to place the client in a prone position, on the client's stomach. Although physical therapy may be helpful to a client who has been hospitalized for several days, physical therapy is not an intervention specifically for hepatopulmonary syndrome.

A nurse delegates hygiene care for a client who has advanced cirrhosis to an unlicensed nursing personnel (UAP). Which statements should the nurse include when delegating this task to the UAP? (Select all that apply.) a. "Apply lotion to the client's dry skin areas." b. "Use a basin with warm water to bathe the client." c. "For the client's oral care, use a soft toothbrush." d. "Provide clippers so the client can trim the fingernails." e. "Bathe with antibacterial and water-based soaps."

ANS: A, C, D Clients with advanced cirrhosis often have pruritus. Lotion will help decrease itchiness from dry skin. A soft toothbrush should be used to prevent gum bleeding, and the client's nails should be trimmed short to prevent the client from scratching himself or herself. These clients should use cool, not warm, water on their skin, and should not use excessive amounts of soap.

A nurse evaluates the results of diagnostic tests on a client's cerebrospinal fluid (CSF). Which fluid results alerts the nurse to possible viral meningitis? (Select all that apply.) a. Clear b. Cloudy c. Increased protein level d. Normal glucose level e. Bacterial organisms present f. Increased white blood cells

ANS: A, C, D In viral meningitis, CSF fluid is clear, protein levels are slightly increased, and glucose levels are normal. Viral meningitis does not cause cloudiness or increased turbidity of CSF. In bacterial meningitis, the presence of bacteria and white blood cells causes the fluid to be cloudy.

A nurse assesses a client who experienced a spinal cord injury at the T5 level 12 hours ago. Which manifestations should the nurse correlate with neurogenic shock? (Select all that apply.) a. Heart rate of 34 beats/min b. Blood pressure of 185/65 mm Hg c. Urine output less than 30 mL/hr d. Decreased level of consciousness e. Increased oxygen saturation

ANS: A, C, D Neurogenic shock with acute spinal cord injury manifests with decreased oxygen saturation, symptomatic bradycardia, decreased level of consciousness, decreased urine output, and hypotension.

A nurse plans care for a client who has a wound that is not healing. Which focused assessments should the nurse complete to develop the client's plan of care? (Select all that apply.) a. Height b. Allergies c. Alcohol use d. Prealbumin laboratory results e. Liver enzyme laboratory results

ANS: A, C, D Nutritional status can have a significant impact on skin health and wound healing. The care plan for a client with poor nutritional status should include a high-protein, high-calorie diet. To determine the client's nutritional status, the nurse should assess height and weight, alcohol use, and prealbumin laboratory results. These data will provide information related to vitamin and protein deficiencies, and obesity. Allergies and liver enzyme laboratory results will not provide information about nutrition status or wound healing.

A nurse cares for older clients who have traumatic brain injury. What should the nurse understand about this population? (Select all that apply.) a. Admission can overwhelm the coping mechanisms for older clients. b. Alcohol is typically involved in most traumatic brain injuries for this age group. c. These clients are more susceptible to systemic and wound infections. d. Other medical conditions can complicate treatment for these clients. e. Very few traumatic brain injuries occur in this age group.

ANS: A, C, D Older clients often tolerate stress poorly, which includes being admitted to a hospital that is unfamiliar and noisy. Because of decreased protective mechanisms, they are more susceptible to both local and systemic infections. Other medical conditions can complicate their treatment and recovery. Alcohol is typically not related to traumatic brain injury in this population; such injury is most often from falls and motor vehicle crashes. The 65- to 76-year-old age group has the second highest rate of brain injuries compared to other age groups.

A nurse is teaching clients with gastroesophageal reflux disease (GERD) about foods to avoid. Which foods should the nurse include in the teaching? (Select all that apply.) a. Chocolate b. Decaffeinated coffee c. Citrus fruits d. Peppermint e. Tomato sauce

ANS: A, C, D, E Chocolate, citrus fruits such as oranges and grapefruit, peppermint and spearmint, and tomato-based products all contribute to the reflux associated with GERD. Caffeinated teas, coffee, and sodas should be avoided.

A nurse is caring for several clients prior to surgery. Which medications taken by the clients require the nurse to consult with the physician about their administration? (Select all that apply.) a. Metformin (Glucophage) b. Omega-3 fatty acids (Sea Omega 30) c. Phenytoin (Dilantin) d. Pilocarpine hydrochloride (Isopto Carpine) e. Warfarin (Coumadin)

ANS: A, C, D, E Although the client will be on NPO status before surgery, the nurse should check with the provider about allowing the client to take medications prescribed for diabetes, hypertension, cardiac disease, seizure disorders, depression, glaucoma, anticoagulation, or depression. Metformin is used to treat diabetes; phenytoin is for seizures; pilocarpine is for glaucoma, and warfarin is an anticoagulant. The omega-3 fatty acids can be held the day of surgery.

A client receiving chemotherapy has a white blood cell count of 1000/mm3. What actions by the nurse are most appropriate? (Select all that apply.) a. Assess all mucous membranes every 4 to 8 hours. b. Do not allow the client to eat meat or poultry. c. Listen to lung sounds and monitor for cough. d. Monitor the venous access device appearance with vital signs. e. Take and record vital signs every 4 to 8 hours.

ANS: A, C, D, E Depending on facility protocol, the nurse should assess this client for infection every 4 to 8 hours by assessing all mucous membranes, listening to lung sounds, monitoring for cough, monitoring the appearance of the venous access device, and recording vital signs. Eating meat and poultry is allowed.

A student nurse is caring for clients on the postoperative unit. The student asks the registered nurse why malnutrition can lead to poor surgical outcomes. What responses by the nurse are best? (Select all that apply.) a. "A malnourished client will have fragile skin." b. "Malnourished clients always have other problems." c. "Many drugs are bound to protein in the body." d. "Protein stores are needed for wound healing." e. "Weakness and fatigue are common in malnutrition."

ANS: A, C, D, E Malnutrition can lead to poorer surgical outcomes for several reasons, including fragile skin that might break down, altered pharmacokinetics, poorer wound healing, and weakness or fatigue that can interfere with recovery. Malnutrition can exist without other comorbidities.

An infection control nurse develops a plan to decrease the number of health care professionals who contract viral hepatitis at work. Which ideas should the nurse include in this plan? (Select all that apply.) a. Policies related to consistent use of Standard Precautions b. Hepatitis vaccination mandate for workers in high-risk areas c. Implementation of a needleless system for intravenous therapy d. Number of sharps used in client care reduced where possible e. Postexposure prophylaxis provided in a timely manner

ANS: A, C, D, E Nurses should always use Standard Precautions for client care, and policies should reflect this. Needleless systems and reduction of sharps can help prevent hepatitis. Postexposure prophylaxis should be provided immediately. All health care workers should receive the hepatitis vaccinations that are available.

A client has been diagnosed with Bell's palsy. About what drugs should the nurse anticipate possibly teaching the client? (Select all that apply.) a. Acyclovir (Zovirax) b. Carbamazepine (Tegretol) c. Famciclovir (Famvir) d. Prednisone (Deltasone) e. Valacyclovir (Valtrex)

ANS: A, C, D, E Possible pharmacologic treatment for Bell's palsy includes acyclovir, famciclovir, prednisone, and valacyclovir. Carbamazepine is an anticonvulsant and mood-stabilizing drug and is not used for Bell's palsy.

A nurse studying cancer knows that job-related risks for developing oral cancer include which occupations? (Select all that apply.) a. Coal miner b. Electrician c. Metal worker d. Plumber e. Textile worker

ANS: A, C, D, E The occupations of coal mining, metal working, plumbing, and textile work produce exposure to polycyclic aromatic hydrocarbons (PAHs), which are known carcinogens. Electricians do not have this risk.

A nurse assesses a client who recently experienced a traumatic spinal cord injury. Which assessment data should the nurse obtain to assess the client's coping strategies? (Select all that apply.) a. Spiritual beliefs b. Level of pain c. Family support d. Level of independence e. Annual income f. Previous coping strategies

ANS: A, C, D, F Information about the client's preinjury psychosocial status, usual methods of coping with illness, difficult situations, and disappointments should be obtained. Determine the client's level of independence or dependence and his or her comfort level in discussing feelings and emotions with family members or close friends. Clients who are emotionally secure and have a positive self-image, a supportive family, and financial and job security often adapt to their injury. Information about the client's spiritual and religious beliefs or cultural background also assists the nurse in developing the plan of care. The other options do not supply as much information about coping.

A nurse assesses a client who is experiencing diabetic ketoacidosis (DKA). For which manifestations should the nurse monitor the client? (Select all that apply.) a. Deep and fast respirations b. Decreased urine output c. Tachycardia d. Dependent pulmonary crackles e. Orthostatic hypotension

ANS: A, C, E DKA leads to dehydration, which is manifested by tachycardia and orthostatic hypotension. Usually clients have Kussmaul respirations, which are fast and deep. Increased urinary output (polyuria) is severe. Because of diuresis and dehydration, peripheral edema and crackles do not occur.

A nurse plans care for a client who has acute pancreatitis and is prescribed nothing by mouth (NPO). With which health care team members should the nurse collaborate to provide appropriate nutrition to this client? (Select all that apply.) a. Registered dietitian b. Nursing assistant c. Clinical pharmacist d. Certified herbalist e. Health care provider

ANS: A, C, E Clients who are prescribed NPO while experiencing an acute pancreatitis episode may need enteral or parenteral nutrition. The nurse should collaborate with the registered dietitian, clinical pharmacist, and health care provider to plan and implement the more appropriate nutritional interventions. The nursing assistant and certified herbalist would not assist with this clinical decision.

An older client returning to the postoperative nursing unit after a hip replacement is disoriented and restless. What actions does the nurse delegate to the unlicensed assistive personnel (UAP)? (Select all that apply.) a. Apply an abduction pillow to the client's legs. b. Assess the skin under the abduction pillow straps. c. Place pillows under the heels to keep them off the bed. d. Monitor cognition to determine when the client can get up. e. Take and record vital signs per unit/facility policy.

ANS: A, C, E The UAP can apply an abduction pillow, elevate the heels on a pillow, and take/record vital signs. Assessing skin is the nurse's responsibility, although if the UAP notices abnormalities, he or she should report them. Determining when the client is able to get out of bed is also a nursing responsibility.

A nurse inserts a nasogastric (NG) tube for an adult client who has a bowel obstruction. Which actions does the nurse perform correctly? (Select all that apply.) a. Performs hand hygiene and positions the client in high-Fowler's position, with pillows behind the head and shoulders b. Instructs the client to extend the neck against the pillow once the NG tube has reached the oropharynx c. Checks for correct placement by checking the pH of the fluid aspirated from the tube d. Secures the NG tube by taping it to the client's nose and pinning the end to the pillowcase e. Connects the NG tube to intermittent medium suction with an anti-reflux valve on the air vent

ANS: A, C, E The client's head should be flexed forward once the NG tube has reached the oropharynx. The NG tube should be secured to the client's gown, not to the pillowcase, because it could become dislodged easily. All the other actions are appropriate.

A nurse cares for a client with a hypofunctioning anterior pituitary gland. Which hormones should the nurse expect to be affected by this condition? (Select all that apply.) a. Thyroid-stimulating hormone b. Vasopressin c. Follicle-stimulating hormone d. Calcitonin e. Growth hormone

ANS: A, C, E Thyroid-stimulating hormone, follicle-stimulating hormone, and growth hormone all are secreted by the anterior pituitary gland. Vasopressin is secreted from the posterior pituitary gland. Calcitonin is secreted from the thyroid gland.

A client has dumping syndrome. What menu selections indicate the client understands the correct diet to manage this condition? (Select all that apply.) a. Canned unsweetened apricots b. Coffee cake c. Milk shake d. Potato soup e. Steamed broccoli

ANS: A, D Canned apricots and potato soup are appropriate selections as they are part of a high-protein, high-fat, low- to moderate-carbohydrate diet. Coffee cake and other sweets must be avoided. Milk products and sweet drinks such as shakes must be avoided. Gas-forming foods such as broccoli must also be avoided.

A nursing student studying traumatic brain injuries (TBIs) should recognize which facts about these disorders? (Select all that apply.) a. A client with a moderate trauma may need hospitalization. b. A Glasgow Coma Scale score of 10 indicates a mild brain injury. c. Only open head injuries can cause a severe TBI. d. A client with a Glasgow Coma Scale score of 3 has severe TBI. e. The terms "mild TBI" and "concussion" have similar meanings.

ANS: A, D, E "Mild TBI" is a term used synonymously with the term "concussion." A moderate TBI has a Glasgow Coma Scale (GCS) score of 9 to 12, and these clients may need to be hospitalized. Both open and closed head injuries can cause a severe TBI, which is characterized by a GCS score of 3 to 8.

A nurse teaches a client who is prescribed an unsealed radioactive isotope. Which statements should the nurse include in this client's education? (Select all that apply.) a. "Do not share utensils, plates, and cups with anyone else." b. "You can play with your grandchildren for 1 hour each day." c. "Eat foods high in vitamins such as apples, pears, and oranges." d. "Wash your clothing separate from others in the household." e. "Take a laxative 2 days after therapy to excrete the radiation."

ANS: A, D, E A client who is prescribed an unsealed radioactive isotope should be taught to not share utensils, plates, and cups with anyone else; to avoid contact with pregnant women and children; to avoid eating foods with cores or bones, which will leave contaminated remnants; to wash clothing separate from others in the household and run an empty cycle before washing other people's clothing; and to take a laxative on days 2 and 3 after receiving treatment to help excrete the contaminated stool faster.

The nursing student learning about cancer development remembers characteristics of normal cells. Which characteristics does this include? (Select all that apply.) a. Differentiated function b. Large nucleus-to-cytoplasm ratio c. Loose adherence d. Nonmigratory e. Specific morphology

ANS: A, D, E Normal cells have the characteristics of differentiated function, nonmigratory, specific morphology, a smaller nucleus-to-cytoplasm ratio, tight adherence, and orderly and well-regulated growth.

A student nurse observing in the operating room notes that the functions of the Certified Registered Nurse First Assistant (CRNFA) include which activities? (Select all that apply.) a. Dressing the surgical wound b. Grafting new or synthetic skin c. Reattaching severed nerves d. Suctioning the surgical site e. Suturing the surgical wound

ANS: A, D, E The CRNFA can perform tasks under the direction of the surgeon such as suturing and dressing surgical wounds, cutting away tissue, suctioning the wound to improve visibility, and holding retractors. Reattaching severed nerves and performing grafts would be the responsibility of the surgeon.

A nurse assesses clients at a health fair. Which clients should the nurse counsel to be tested for diabetes? (Select all that apply.) a. 56-year-old African-American male b. Female with a 30-pound weight gain during pregnancy c. Male with a history of pancreatic trauma d. 48-year-old woman with a sedentary lifestyle e. Male with a body mass index greater than 25 kg/m2 f. 28-year-old female who gave birth to a baby weighing 9.2 pounds

ANS: A, D, E, F Risk factors for type 2 diabetes include certain ethnic/racial groups (African Americans, American Indians, Hispanics), obesity and physical inactivity, and giving birth to large babies. Pancreatic trauma and a 30-pound gestational weight gain are not risk factors.

A nurse plans care for a client who is immobile. Which interventions should the nurse include in this client's plan of care to prevent pressure sores? (Select all that apply.) a.Place a small pillow between bony surfaces. b.Elevate the head of the bed to 45 degrees. c.Limit fluids and proteins in the diet. d.Use a lift sheet to assist with re-positioning. e.Re-position the client who is in a chair every 2 hours. f.Keep the client's heels off the bed surfaces. g.Use a rubber ring to decrease sacral pressure when up in the chair.

ANS: A, D, F A small pillow decreases the risk for pressure between bony prominences, a lift sheet decreases friction and shear, and heels have poor circulation and are at high risk for pressure sores, so they should be kept off hard surfaces. Head-of-the-bed elevation greater than 30 degrees increases pressure on pelvic soft tissues. Fluids and proteins are important for maintaining tissue integrity. Clients should be repositioned every hour while sitting in a chair. A rubber ring impairs capillary blood flow, increasing the risk for a pressure sore.

A nurse assesses a client with cholelithiasis. Which assessment findings should the nurse identify as contributors to this client's condition? (Select all that apply.) a. Body mass index of 46 b. Vegetarian diet c. Drinking 4 ounces of red wine nightly d. Pregnant with twins e. History of metabolic syndrome f. Glycosylated hemoglobin level of 15%

ANS: A, D, F Obesity, pregnancy, and diabetes are all risk factors for the development of cholelithiasis. A diet low in saturated fats and moderate alcohol intake may decrease the risk. Although metabolic syndrome is a precursor to diabetes, it is not a risk factor for cholelithiasis. The client should be informed of the connection.

A nurse plans care for a client with epilepsy who is admitted to the hospital. Which interventions should the nurse include in this client's plan of care? (Select all that apply.) a. Have suction equipment at the bedside. b. Place a padded tongue blade at the bedside. c. Permit only clear oral fluids. d. Keep bed rails up at all times. e. Maintain the client on strict bedrest. f. Ensure that the client has IV access.

ANS: A, D, F Oxygen and suctioning equipment with an airway must be readily available. The bed rails should be up at all times while the client is in the bed to prevent injury from a fall if the client has a seizure. If the client does not have an IV access, insert a saline lock, especially for those clients who are at significant risk for generalized tonic-clonic seizures. The saline lock provides ready access if IV drug therapy must be given to stop the seizure. Padded tongue blades may pose a danger to the client during a seizure and should not be used. Dietary restrictions and strict bedrest are not interventions associated with epilepsy. The client should be encouraged to eat a well-balanced diet and ambulate while in the hospital.

A nurse assesses a wife who is caring for her husband. She has a Braden Scale score of 9. Which question should the nurse include in this assessment? a."Do you have a bedpan at home?" b."How are you coping with providing this care?" c."What are you doing to prevent pediculosis?" d."Are you sharing a bed with your husband?"

ANS: B A client with a Braden Scale score of 9 is at high risk for skin breakdown and requires moderate to maximum assistance to prevent further breakdown. Family members who care for clients at home may experience a disruption in family routines and added stress. The nurse should assess the wife's feelings and provide support for coping with changes. Asking about the client's toileting practices, prevention of pediculosis, and sleeping arrangements do not provide information about the caregiver's support and coping mechanisms and ability to continue to care for her husband.

A client with an esophageal tumor has difficulty swallowing and has been working with a speech-language pathologist. What assessment finding by the nurse indicates that the priority goal for this problem is being met? a. Choosing foods that are easy to swallow b. Lungs clear after meals and snacks c. Properly performing swallowing exercises d. Weight unchanged after 2 weeks

ANS: B All these assessment findings are positive for this client. However, this client is at high risk for aspiration. Clear lungs after eating indicates no aspiration has occurred. Choosing easy-to-swallow foods, performing swallowing checks, and having an unchanged weight do not assess aspiration, and therefore do not indicate that the priority goal has been met.

A nurse cares for a client who is diagnosed with acute rejection 2 months after receiving a simultaneous pancreas-kidney transplant. The client states, "I was doing so well with my new organs, and the thought of having to go back to living on hemodialysis and taking insulin is so depressing." How should the nurse respond? a. "Following the drug regimen more closely would have prevented this." b. "One acute rejection episode does not mean that you will lose the new organs." c. "Dialysis is a viable treatment option for you and may save your life." d. "Since you are on the national registry, you can receive a second transplantation."

ANS: B An episode of acute rejection does not automatically mean that the client will lose the transplant. Pharmacologic manipulation of host immune responses at this time can limit damage to the organ and allow the graft to be maintained. The other statements either belittle the client or downplay his or her concerns. The client may not be a candidate for additional organ transplantation.

A nurse is teaching a client with diabetes mellitus who asks, "Why is it necessary to maintain my blood glucose levels no lower than about 60 mg/dL?" How should the nurse respond? a. "Glucose is the only fuel used by the body to produce the energy that it needs." b. "Your brain needs a constant supply of glucose because it cannot store it." c. "Without a minimum level of glucose, your body does not make red blood cells." d. "Glucose in the blood prevents the formation of lactic acid and prevents acidosis."

ANS: B Because the brain cannot synthesize or store significant amounts of glucose, a continuous supply from the body's circulation is needed to meet the fuel demands of the central nervous system. The nurse would want to educate the client to prevent hypoglycemia. The body can use other sources of fuel, including fat and protein, and glucose is not involved in the production of red blood cells. Glucose in the blood will encourage glucose metabolism but is not directly responsible for lactic acid formation.

A client is scheduled for a traditional esophagogastrostomy. All preoperative teaching has been completed and the client and family show good understanding. What action by the nurse is best? a. Arrange an intensive care unit tour. b. Assess the client's psychosocial status. c. Document the teaching and response. d. Have the client begin nutritional supplements.

ANS: B Clients facing this long, difficult procedure are often anxious and fearful. The nurse should now assess the client's psychosocial status and provide the care and teaching required based on this assessment. An intensive care unit tour may help decrease stress but is too limited in scope to be the best response. Documentation should be thorough, but the nurse needs to do more than document. The client should begin nutritional supplements prior to the operation, but again this response is too limited in scope.

A nurse assesses a client who has psoriasis. Which action should the nurse take first? a.Don gloves and an isolation gown. b.Shake the client's hand and introduce self. c.Assess for signs and symptoms of infections. d.Ask the client if she might be pregnant.

ANS: B Clients with psoriatic lesions are often self-conscious of their skin. The nurse should first provide direct contact and touch without gloves to establish a good report with the client. Psoriasis is not an infectious disease, nor is it contagious. The nurse would not need to wear gloves or an isolation gown. Obtaining a health history and assessing for an infection and pregnancy should be completed after establishing a report with the client.

A nurse reviews the medication list of a client with a 20-year history of diabetes mellitus. The client holds up the bottle of prescribed duloxetine (Cymbalta) and states, "My cousin has depression and is taking this drug. Do you think I'm depressed?" How should the nurse respond? a. "Many people with long-term diabetes become depressed after a while." b. "It's for peripheral neuropathy. Do you have burning pain in your feet or hands?" c. "This antidepressant also has anti-inflammatory properties for diabetic pain." d. "No. Many medications can be used for several different disorders."

ANS: B Damage along nerves causes peripheral neuropathy and leads to burning pain along the nerves. Many drugs, including duloxetine (Cymbalta), can be used to treat peripheral neuropathy. The nurse should assess the client for this condition and then should provide an explanation of why this drug is being used. This medication, although it is used for depression, is not being used for that reason in this case. Duloxetine does not have anti-inflammatory properties. Telling the client that many medications are used for different disorders does not provide the client with enough information to be useful.

A client with an esophageal tumor is having extreme difficulty swallowing. For what procedure does the nurse prepare this client? a. Enteral tube feeding b. Esophageal dilation c. Nissen fundoplication d. Photodynamic therapy

ANS: B Esophageal dilation can provide immediate relief of esophageal strictures that impair swallowing. Enteral tube feeding is a method of providing nutrition when dysphagia is severe, but esophageal dilation would be attempted before this measure is taken. Nissen fundoplication is performed for severe gastroesophageal reflux disease. Photodynamic therapy is performed for esophageal cancer.

A nurse cares for a client with diabetes mellitus who asks, "Why do I need to administer more than one injection of insulin each day?" How should the nurse respond? a. "You need to start with multiple injections until you become more proficient at self-injection." b. "A single dose of insulin each day would not match your blood insulin levels and your food intake patterns." c. "A regimen of a single dose of insulin injected each day would require that you eat fewer carbohydrates." d. "A single dose of insulin would be too large to be absorbed, predictably putting you at risk for insulin shock."

ANS: B Even when a single injection of insulin contains a combined dose of different-acting insulin types, the timing of the actions and the timing of food intake may not match well enough to prevent wide variations in blood glucose levels. One dose of insulin would not be appropriate even if the client decreased carbohydrate intake. Additional injections are not required to allow the client practice with injections, nor will one dose increase the client's risk of insulin shock.

A client has been taught about alginic acid and sodium bicarbonate (Gaviscon). What statement by the client indicates that teaching has been effective? a. "I can only take this medicine at night." b. "I should take this on a full stomach." c. "This drug decreases stomach acid." d. "This should be taken 1 hour before meals."

ANS: B Gaviscon should be taken with food in the stomach. It can be taken with meals at any time. Its mechanism of action is not to decrease stomach acid.

After educating a caregiver of a home care client, a nurse assesses the caregiver's understanding. Which statement indicates that the caregiver needs additional education? a."I can help him shift his position every hour when he sits in the chair." b."If his tailbone is red and tender in the morning, I will massage it with baby oil." c."Applying lotion to his arms and legs every evening will decrease dryness." d."Drinking a nutritional supplement between meals will help maintain his weight."

ANS: B Massage of reddened areas over bony prominences such as the coccyx, or tailbone, is contraindicated because the pressure of the massage can cause damage to the skin and subcutaneous tissue layers. The other statements are appropriate for the care of a client at home.

A nurse cares for a client who has diabetes mellitus. The nurse administers 6 units of regular insulin and 10 units of NPH insulin at 0700. At which time should the nurse assess the client for potential problems related to the NPH insulin? a. 0800 b. 1600 c. 2000 d. 2300

ANS: B Neutral protamine Hagedorn (NPH) is an intermediate-acting insulin with an onset of 1.5 hours, peak of 4 to 12 hours, and duration of action of 22 hours. Checking the client at 0800 would be too soon. Checking the client at 2000 and 2300 would be too late. The nurse should check the client at 1600.

A nurse performs a skin screening for a client who has numerous skin lesions. Which lesion does the nurse evaluate first? a.Beige freckles on the backs of both hands b.Irregular blue mole with white specks on the lower leg c.Large cluster of pustules in the right axilla d.Thick, reddened papules covered by white scales

ANS: B This mole fits two of the criteria for being cancerous or precancerous: variation of color within one lesion, and an indistinct or irregular border. Melanoma is an invasive malignant disease with the potential for a fatal outcome. Freckles are a benign condition. Pustules could mean an infection, but it is more important to take care of the potentially cancerous lesion first. Psoriasis vulgaris manifests as thick reddened papules covered by white scales. This is a chronic disorder and is not the priority.

A nurse teaches a client with diabetes mellitus about sick day management. Which statement should the nurse include in this client's teaching? a. "When ill, avoid eating or drinking to reduce vomiting and diarrhea." b. "Monitor your blood glucose levels at least every 4 hours while sick." c. "If vomiting, do not use insulin or take your oral antidiabetic agent." d. "Try to continue your prescribed exercise regimen even if you are sick."

ANS: B When ill, the client should monitor his or her blood glucose at least every 4 hours. The client should continue taking the medication regimen while ill. The client should continue to eat and drink as tolerated but should not exercise while sick.

A nurse assesses a client who has diabetes mellitus. Which arterial blood gas values should the nurse identify as potential ketoacidosis in this client? a. pH 7.38, HCO3- 22 mEq/L, PCO2 38 mm Hg, PO2 98 mm Hg b. pH 7.28, HCO3- 18 mEq/L, PCO2 28 mm Hg, PO2 98 mm Hg c. pH 7.48, HCO3- 28 mEq/L, PCO2 38 mm Hg, PO2 98 mm Hg d. pH 7.32, HCO3- 22 mEq/L, PCO2 58 mm Hg, PO2 88 mm Hg

ANS: B When the lungs can no longer offset acidosis, the pH decreases to below normal. A client who has diabetic ketoacidosis would present with arterial blood gas values that show primary metabolic acidosis with decreased bicarbonate levels and a compensatory respiratory alkalosis with decreased carbon dioxide levels.

A nurse cares for a client with hepatic portal-systemic encephalopathy (PSE). The client is thin and cachectic in appearance, and the family expresses distress that the client is receiving little dietary protein. How should the nurse respond? a. "A low-protein diet will help the liver rest and will restore liver function." b. "Less protein in the diet will help prevent confusion associated with liver failure." c. "Increasing dietary protein will help the client gain weight and muscle mass." d. "Low dietary protein is needed to prevent fluid from leaking into the abdomen."

ANS: B A low-protein diet is ordered when serum ammonia levels increase and/or the client shows signs of PSE. A low-protein diet helps reduce excessive breakdown of protein into ammonia by intestinal bacteria. Encephalopathy is caused by excess ammonia. A low-protein diet has no impact on restoring liver function. Increasing the client's dietary protein will cause complications of liver failure and should not be suggested. Increased intravascular protein will help prevent ascites, but clients with liver failure are not able to effectively synthesize dietary protein.

A nurse cares for a client who has cirrhosis of the liver. Which action should the nurse take to decrease the presence of ascites? a. Monitor intake and output. b. Provide a low-sodium diet. c. Increase oral fluid intake. d. Weigh the client daily.

ANS: B A low-sodium diet is one means of controlling abdominal fluid collection. Monitoring intake and output does not control fluid accumulation, nor does weighing the client. These interventions merely assess or monitor the situation. Increasing fluid intake would not be helpful.

A nurse assesses a client after administering prescribed levetiracetam (Keppra). Which laboratory tests should the nurse monitor for potential adverse effects of this medication? a. Serum electrolyte levels b. Kidney function tests c. Complete blood cell count d. Antinuclear antibodies

ANS: B Adverse effects of levetiracetam include coordination problems and renal toxicity. The other laboratory tests are not affected by levetiracetam.

After teaching a client who is recovering from laparoscopic cholecystectomy surgery, the nurse assesses the client's understanding. Which statement made by the client indicates a correct understanding of the teaching? a. "Drinking at least 2 liters of water each day is suggested." b. "I will decrease the amount of fatty foods in my diet." c. "Drinking fluids with my meals will increase bloating." d. "I will avoid concentrated sweets and simple carbohydrates."

ANS: B After cholecystectomy, clients need a nutritious diet without a lot of excess fat; otherwise a special diet is not recommended for most clients. Good fluid intake is healthy for all people but is not related to the surgery. Drinking fluids between meals helps with dumping syndrome, which is not seen with this procedure. Restriction of sweets is not required.

After teaching a client who is recovering from a complete thyroidectomy, the nurse assesses the client's understanding. Which statement made by the client indicates a need for additional instruction? a. "I may need calcium replacement after surgery." b. "After surgery, I won't need to take thyroid medication." c. "I'll need to take thyroid hormones for the rest of my life." d. "I can receive pain medication if I feel that I need it."

ANS: B After the client undergoes a thyroidectomy, the client must be given thyroid replacement medication for life. He or she may also need calcium if the parathyroid is damaged during surgery, and can receive pain medication postoperatively.

The circulating nurse and preoperative nurse are reviewing the chart of a client scheduled for minimally invasive surgery (MIS). What information on the chart needs to be reported to the surgeon as a priority? a. Allergies noted and allergy band on b. Consent for MIS procedure only c. No prior anesthesia exposure d. NPO status for the last 8 hours

ANS: B All MIS procedures have the potential for becoming open procedures depending on findings and complications. The client's consent should include this possibility. The nurse should report this finding to the surgeon prior to surgery taking place. Having allergies noted and an allergy band applied is standard procedure. Not having any prior surgical or anesthesia exposure is not the priority. Maintaining NPO status as prescribed is standard procedure.

A client has been given hydroxyzine (Atarax) in the preoperative holding area. What action by the nurse is most important for this client? a. Document giving the drug. b. Raise the siderails on the bed. c. Record the client's vital signs. d. Teach relaxation techniques.

ANS: B All actions are appropriate for a preoperative client. However, for client safety, the nurse should raise the siderails on the bed because hydroxyzine can make the client sleepy.

A client has an intraventricular catheter. What action by the nurse takes priority? a. Document intracranial pressure readings. b. Perform hand hygiene before client care. c. Measure intracranial pressure per hospital policy. d. Teach the client and family about the device.

ANS: B All of the actions are appropriate for this client. However, performing hand hygiene takes priority because it prevents infection, which is a possibly devastating complication.

A nurse works in the rheumatology clinic and sees clients with rheumatoid arthritis (RA). Which client should the nurse see first? a. Client who reports jaw pain when eating b. Client with a red, hot, swollen right wrist c. Client who has a puffy-looking area behind the knee d. Client with a worse joint deformity since the last visit

ANS: B All of the options are possible manifestations of RA. However, the presence of one joint that is much redder, hotter, or more swollen that the other joints may indicate infection. The nurse needs to see this client first.

The clinic nurse assesses a client with diabetes during a checkup. The client also has osteoarthritis (OA). The nurse notes the client's blood glucose readings have been elevated. What question by the nurse is most appropriate? a. "Are you compliant with following the diabetic diet?" b. "Have you been taking glucosamine supplements?" c. "How much exercise do you really get each week?" d. "You're still taking your diabetic medication, right?"

ANS: B All of the topics are appropriate for a client whose blood glucose readings have been higher than usual. However, since this client also has OA, and glucosamine can increase blood glucose levels, the nurse should ask about its use. The other questions all have an element of nontherapeutic communication in them. "Compliant" is a word associated with negative images, and the client may deny being "noncompliant." Asking how much exercise the client "really" gets is accusatory. Asking if the client takes his or her medications "right?" is patronizing.

An older female client has been prescribed esomeprazole (Nexium) for treatment of chronic gastric ulcers. What teaching is particularly important for this client? a. Check with the pharmacist before taking other medications. b. Increase intake of calcium and vitamin D. c. Report any worsening of symptoms to the provider. d. Take the medication as prescribed by the provider.

ANS: B All of this advice is appropriate for any client taking this medication. However, long-term use is associated with osteoporosis and osteoporosis-related fractures. This client is already at higher risk for this problem and should be instructed to increase calcium and vitamin D intake. The other options are appropriate for any client taking any medication and are not specific to the use of esomeprazole.

A nurse cares for a client with hepatitis C. The client's brother states, "I do not want to contract this infection, so I will not go into his hospital room." How should the nurse respond? a. "If you wear a gown and gloves, you will not get this virus." b. "Viral hepatitis is not spread through casual contact." c. "This virus is only transmitted through a fecal specimen." d. "I can give you an update on your brother's status from here."

ANS: B Although family members may be afraid that they will contract hepatitis C, the nurse should educate the client's family about how the virus is spread. Viral hepatitis, or hepatitis C, is spread via blood-to-blood transmission and is associated with illicit IV drug needle sharing, blood and organ transplantation, accidental needle sticks, unsanitary tattoo equipment, and sharing of intranasal cocaine paraphernalia. Wearing a gown and gloves will not decrease the transmission of this virus. Hepatitis C is not spread through casual contact or a fecal specimen. The nurse would be violating privacy laws by sharing the client's status with the brother.

A nurse cares for a client newly diagnosed with Graves' disease. The client's mother asks, "I have diabetes mellitus. Am I responsible for my daughter's disease?" How should the nurse respond? a. "The fact that you have diabetes did not cause your daughter to have Graves' disease. No connection is known between Graves' disease and diabetes." b. "An association has been noted between Graves' disease and diabetes, but the fact that you have diabetes did not cause your daughter to have Graves' disease." c. "Graves' disease is associated with autoimmune diseases such as rheumatoid arthritis, but not with a disease such as diabetes mellitus." d. "Unfortunately, Graves' disease is associated with diabetes, and your diabetes could have led to your daughter having Graves' disease."

ANS: B An association between autoimmune diseases such as rheumatoid arthritis and diabetes mellitus has been noted. The predisposition is probably polygenic, and the mother's diabetes did not cause her daughter's Graves' disease. The other statements are inaccurate.

A nurse assesses a client with Huntington disease. Which motor changes should the nurse monitor for in this client? a. Shuffling gait b. Jerky hand movements c. Continuous chewing motions d. Tremors of the hands

ANS: B An imbalance between excitatory and inhibitory neurotransmitters leads to uninhibited motor movements, such as brisk, jerky, purposeless movements of the hands, face, tongue, and legs. Shuffling gait, continuous chewing motions, and tremors are associated with Parkinson disease.

A nurse cares for a client who is experiencing status epilepticus. Which prescribed medication should the nurse prepare to administer? a. Atenolol (Tenormin) b. Lorazepam (Ativan) c. Phenytoin (Dilantin) d. Lisinopril (Prinivil)

ANS: B Initially, intravenous lorazepam is administered to stop motor movements. This is followed by the administration of phenytoin. Atenolol, a beta blocker, and lisinopril, an angiotensin-converting enzyme inhibitor, are not administered for seizure activity. These medications are typically administered for hypertension and heart failure.

A client is having robotic surgery. The circulating nurse observes the instruments being inserted, then the surgeon appears to "break scrub" when going to the console and sitting down. What action by the nurse is best? a. Call a "time-out" to discuss sterile procedure and scrub technique. b. Document the time the robotic portion of the procedure begins. c. Inform the surgeon that the scrub preparation has been compromised. d. Report the surgeon's actions to the charge nurse and unit manager.

ANS: B During a robotic operative procedure, the surgeon inserts the articulating arms into the client, then "breaks scrub" to sit at the viewing console to perform the operation. The nurse should document the time the robotic portion of the procedure began. There is no need for the other interventions.

A client is in stage 2 of general anesthesia. What action by the nurse is most important? a. Keeping the room quiet and calm b. Being prepared to suction the airway c. Positioning the client correctly d. Warming the client with blankets

ANS: B During stage 2 of general anesthesia (excitement, delirium), the client can vomit and aspirate. The nurse must be ready to react to this potential occurrence by being prepared to suction the client's airway. Keeping the room quiet and calm does help the client enter the anesthetic state, but is not the priority. Positioning the client usually occurs during stage 3 (operative anesthesia). Keeping the client warm is important throughout to prevent hypothermia.

A new perioperative nurse is receiving orientation to the surgical area and learns about the Surgical Care Improvement Project (SCIP) goals. What major areas do these measures focus on preventing? (Select all that apply.) a. Hemorrhage b. Infection c. Serious cardiac events d. Stroke e. Thromboembolism

ANS: B, C, E The SCIP project includes core measures to prevent infection, serious cardiac events, and thromboembolic events such as deep vein thrombosis.

A telehealth nurse speaks with a client who is recovering from a liver transplant 2 weeks ago. The client states, "I am experiencing right flank pain and have a temperature of 101° F." How should the nurse respond? a. "The anti-rejection drugs you are taking make you susceptible to infection." b. "You should go to the hospital immediately to have your new liver checked out." c. "You should take an additional dose of cyclosporine today." d. "Take acetaminophen (Tylenol) every 4 hours until you feel better."

ANS: B Fever, right quadrant or flank pain, and jaundice are signs of liver transplant rejection; the client should be admitted to the hospital as soon as possible for intervention. Anti-rejection drugs do make a client more susceptible to infection, but this client has signs of rejection, not infection. The nurse should not advise the client to take an additional dose of cyclosporine or acetaminophen as these medications will not treat the acute rejection.

A nurse cares for a client who is scheduled for a paracentesis. Which intervention should the nurse delegate to an unlicensed assistive personnel (UAP)? a. Have the client sign the informed consent form. b. Assist the client to void before the procedure. c. Help the client lie flat in bed on the right side. d. Get the client into a chair after the procedure.

ANS: B For safety, the client should void just before a paracentesis. The nurse or the provider should have the client sign the consent form. The proper position for a paracentesis is sitting upright in bed or, alternatively, sitting on the side of the bed and leaning over the bedside table. The client will be on bedrest after the procedure.

A client is being prepared for a mechanical embolectomy. What action by the nurse takes priority? a. Assess for contraindications to fibrinolytics. b. Ensure that informed consent is on the chart. c. Perform a full neurologic assessment. d. Review the client's medication lists.

ANS: B For this invasive procedure, the client needs to give informed consent. The nurse ensures that this is on the chart prior to the procedure beginning. Fibrinolytics are not used. A neurologic assessment and medication review are important, but the consent is the priority.

A nurse assesses a female client who presents with hirsutism. Which question should the nurse ask when assessing this client? a. "How do you plan to pay for your treatments?" b. "How do you feel about yourself?" c. "What medications are you prescribed?" d. "What are you doing to prevent this from happening?"

ANS: B Hirsutism, or excessive hair growth on the face and body, can result from endocrine disorders. This may cause a disruption in body image, especially for female clients. The nurse should inquire into the client's body image and self-perception. Asking about the client's financial status or current medications does not address the client's immediate problem. The client is not doing anything to herself to cause the problem, nor can the client prevent it from happening.

A nurse cares for a client who is prescribed a drug that blocks a hormone's receptor site. Which therapeutic effect should the nurse expect? a. Greater hormone metabolism b. Decreased hormone activity c. Increased hormone activity d. Unchanged hormone response

ANS: B Hormones cause activity in the target tissues by binding with their specific cellular receptor sites, thereby changing the activity of the cell. When receptor sites are occupied by other substances that block hormone binding, the cell's response is the same as when the level of the hormone is decreased.

A client has received intravenous anesthesia during an operation. What action by the postanesthesia care nurse is most important? a. Assist with administering muscle relaxants to the client. b. Place the client on a cardiac monitor and pulse oximeter. c. Prepare to administer intravenous antiemetics to the client. d. Prevent the client from experiencing postoperative shivering.

ANS: B Intravenous anesthetic agents have the potential to cause respiratory and circulatory depression. The nurse should ensure the client is on a cardiac monitor and pulse oximeter. Muscle relaxants are not indicated for this client at this time. Intravenous anesthetics have a lower rate of postoperative nausea and vomiting than other types. Shivering can occur in any client, but is more common after inhalation agents.

An emergency room nurse assesses a client after a motor vehicle crash and notes ecchymotic areas across the client's lower abdomen. Which action should the nurse take first? a. Measure the client's abdominal girth. b. Assess for abdominal guarding or rigidity. c. Check the client's hemoglobin and hematocrit. d. Obtain the client's complete health history.

ANS: B On noticing the ecchymotic areas, the nurse should check to see if abdominal guarding or rigidity is present, because this could indicate major organ injury. The nurse should then notify the provider. Measuring abdominal girth or obtaining a complete health history is not appropriate at this time. Laboratory test results can be checked after assessment for abdominal guarding or rigidity.

A client with systemic lupus erythematosus (SLE) was recently discharged from the hospital after an acute exacerbation. The client is in the clinic for a follow-up visit and is distraught about the possibility of another hospitalization disrupting the family. What action by the nurse is best? a. Explain to the client that SLE is an unpredictable disease. b. Help the client create backup plans to minimize disruption. c. Offer to talk to the family and educate them about SLE. d. Tell the client to remain compliant with treatment plans.

ANS: B SLE is an unpredictable disease and acute exacerbations can occur without warning, creating chaos in the family. Helping the client make backup plans for this event not only will decrease the disruption but will give the client a sense of having more control. Explaining facts about the disease is helpful as well but does not engage the client in problem solving. The family may need education, but again this does not help the client to problem-solve. Remaining compliant may help decrease exacerbations, but is not as powerful an intervention as helping the client plan for such events.

A nurse assesses a client with a history of epilepsy who experiences stiffening of the muscles of the arms and legs, followed by an immediate loss of consciousness and jerking of all extremities. How should the nurse document this activity? a. Atonic seizure b. Tonic-clonic seizure c. Myoclonic seizure d. Absence seizure

ANS: B Seizure activity that begins with stiffening of the arms and legs, followed by loss of consciousness and jerking of all extremities, is characteristic of a tonic-clonic seizure. An atonic seizure presents as a sudden loss of muscle tone followed by postictal confusion. A myoclonic seizure presents with a brief jerking or stiffening of extremities that may occur singly or in groups. Absence seizures present with automatisms, and the client is unaware of his or her environment.

A nurse cares for a client with advanced Alzheimer's disease. The client's caregiver states, "She is always wandering off. What can I do to manage this restless behavior?" How should the nurse respond? a. "This is a sign of fatigue. The client would benefit from a daily nap." b. "Engage the client in scheduled activities throughout the day." c. "It sounds like this is difficult for you. I will consult the social worker." d. "The provider can prescribe a mild sedative for restlessness."

ANS: B Several strategies may be used to cope with restlessness and wandering. One strategy is to engage the client in structured activities. Another is to take the client for frequent walks. Daily naps and a mild sedative will not be as effective in the management of restless behavior. Consulting the social worker does not address the caregiver's concern.

A nurse obtains a health history on a client prior to administering prescribed sumatriptan succinate (Imitrex) for migraine headaches. Which condition should alert the nurse to hold the medication and contact the health care provider? a. Bronchial asthma b. Prinzmetal's angina c. Diabetes mellitus d. Chronic kidney disease

ANS: B Sumatriptan succinate effectively reduces pain and other associated symptoms of migraine headache by binding to serotonin receptors and triggering cranial vasoconstriction. Vasoconstrictive effects are not confined to the cranium and can cause coronary vasospasm in clients with Prinzmetal's angina. The other conditions would not affect the client's treatment.

A nurse cares for a client who is recovering from a hemorrhoidectomy. The client states, "I need to have a bowel movement." Which action should the nurse take? a. Obtain a bedside commode for the client to use. b. Stay with the client while providing privacy. c. Make sure the call light is in reach to signal completion. d. Gather supplies to collect a stool sample for the laboratory.

ANS: B The first bowel movement after hemorrhoidectomy can be painful enough to induce syncope. The nurse should stay with the client. The nurse should instruct clients who are discharged the same day to have someone nearby when they have their first postoperative bowel movement. Making sure the call light is within reach is an important nursing action too, but it does not take priority over client safety. Obtaining a bedside commode and taking a stool sample are not needed in this situation.

An emergency room nurse assesses a client after a motor vehicle crash. The nurse notices a "steering wheel mark" across the client's chest. Which action should the nurse take? a. Ask the client where in the car he or she was sitting during the crash. b. Assess the client by gently palpating the abdomen for tenderness. c. Notify the laboratory to draw blood for blood type and crossmatch. d. Place the client on the stretcher in reverse Trendelenburg position.

ANS: B The liver is often injured by a steering wheel in a motor vehicle crash. Because the client's chest was marked by the steering wheel, the nurse should perform an abdominal assessment. Assessing the client's position in the crash is not needed because of the steering wheel imprint. The client may or may not need a blood transfusion. The client does not need to be in reverse Trendelenburg position.

A nurse cares for a client who presents with bradycardia secondary to hypothyroidism. Which medication should the nurse anticipate being prescribed to the client? a. Atropine sulfate b. Levothyroxine sodium (Synthroid) c. Propranolol (Inderal) d. Epinephrine (Adrenalin)

ANS: B The treatment for bradycardia from hypothyroidism is to treat the hypothyroidism using levothyroxine sodium. If the heart rate were so slow that it became an emergency, then atropine or epinephrine might be an option for short-term management. Propranolol is a beta blocker and would be contraindicated for a client with bradycardia.

The nurse is caring for a client with peptic ulcer disease who reports sudden onset of sharp abdominal pain. On palpation, the client's abdomen is tense and rigid. What action takes priority? a. Administer the prescribed pain medication. b. Notify the health care provider immediately. c. Percuss all four abdominal quadrants. d. Take and document a set of vital signs.

ANS: B This client has manifestations of a perforated ulcer, which is an emergency. The priority is to get the client medical attention. The nurse can take a set of vital signs while someone else calls the provider. The nurse should not percuss the abdomen or give pain medication since the client may need to sign consent for surgery.

After a craniotomy, the nurse assesses the client and finds dry, sticky mucous membranes and restlessness. The client has IV fluids running at 75 mL/hr. What action by the nurse is best? a. Assess the client's magnesium level. b. Assess the client's sodium level. c. Increase the rate of the IV infusion. d. Provide oral care every hour.

ANS: B This client has manifestations of hypernatremia, which is a possible complication after craniotomy. The nurse should assess the client's serum sodium level. Magnesium level is not related. The nurse does not independently increase the rate of the IV infusion. Providing oral care is also a good option but does not take priority over assessing laboratory results.

An older client is scheduled to have hip replacement in 2 months and has the following laboratory values: white blood cell count: 8900/mm3, red blood cell count: 3.2/mm3, hemoglobin: 9 g/dL, hematocrit: 32%. What intervention by the nurse is most appropriate? a. Instruct the client to avoid large crowds. b. Prepare to administer epoetin alfa (Epogen). c. Teach the client about foods high in iron. d. Tell the client that all laboratory results are normal.

ANS: B This client is anemic, which needs correction prior to surgery. While eating iron-rich foods is helpful, to increase the client's red blood cells, hemoglobin, and hematocrit within 2 months, epoetin alfa is needed. This colony-stimulating factor will encourage the production of red cells. The client's white blood cell count is normal, so avoiding infection is not the priority.

A client is in the emergency department reporting a brief episode during which he was dizzy, unable to speak, and felt like his legs were very heavy. Currently the client's neurologic examination is normal. About what drug should the nurse plan to teach the client? a. Alteplase (Activase) b. Clopidogrel (Plavix) c. Heparin sodium d. Mannitol (Osmitrol)

ANS: B This client's manifestations are consistent with a transient ischemic attack, and the client would be prescribed aspirin or clopidogrel on discharge. Alteplase is used for ischemic stroke. Heparin and mannitol are not used for this condition.

A client is having a catheter placed in the femoral artery to deliver yttrium-90 beads into a liver tumor. What action by the nurse is most important? a. Assessing the client's abdomen beforehand b. Ensuring that informed consent is on the chart c. Marking the client's bilateral pedal pulses d. Reviewing client teaching done previously

ANS: B This is an invasive procedure requiring informed consent. The nurse should ensure that consent is on the chart. The other actions are also appropriate but not the priority.

A client presents to the emergency department reporting severe abdominal pain. On assessment, the nurse finds a bulging, pulsating mass in the abdomen. What action by the nurse is the priority? a. Auscultate for bowel sounds. b. Notify the provider immediately. c. Order an abdominal flat-plate x-ray. d. Palpate the mass and measure its size.

ANS: B This observation could indicate an abdominal aortic aneurysm, which could be life threatening and should never be palpated. The nurse notifies the provider at once. An x-ray may be indicated. Auscultation is part of assessment, but the nurse's priority action is to notify the provider

The nursing student studying rheumatoid arthritis (RA) learns which facts about the disease? (Select all that apply.) a. It affects single joints only. b. Antibodies lead to inflammation. c. It consists of an autoimmune process. d. Morning stiffness is rare. e. Permanent damage is inevitable.

ANS: B, C RA is a chronic autoimmune systemic inflammatory disorder leading to arthritis-type symptoms in the joints and other symptoms that can be seen outside the joints. Antibodies are created that lead to inflammation. Clients often report morning stiffness. Permanent damage can be avoided with aggressive, early treatment.

The nurse is working with clients who have esophageal disorders. The nurse should assess the clients for which manifestations? (Select all that apply.) a. Aphasia b. Dysphagia c. Eructation d. Halitosis e. Weight gain

ANS: B, C, D Common signs of esophageal disorders include dysphagia, eructation, halitosis, and weight loss. Aphasia is difficulty with speech, commonly seen after stroke.

A nurse collaborates with an unlicensed assistive personnel (UAP) to provide care for a client who is in the healing phase of acute pancreatitis. Which statements focused on nutritional requirements should the nurse include when delegating care for this client? (Select all that apply.) a. "Do not allow the client to eat between meals." b. "Make sure the client receives a protein shake." c. "Do not allow caffeine-containing beverages." d. "Make sure the foods are bland with little spice." e. "Do not allow high-carbohydrate food items."

ANS: B, C, D During the healing phase of pancreatitis, the client should be provided small, frequent, moderate- to high-carbohydrate, high-protein, low-fat meals. Protein shakes can be provided to supplement the diet. Foods and beverages should not contain caffeine and should be bland.

A nurse cares for a client with pancreatic cancer who is prescribed implanted radioactive iodine seeds. Which actions should the nurse take when caring for this client? (Select all that apply.) a. Dispose of dirty linen in a red "biohazard" bag. b. Place the client in a private room. c. Wear a lead apron when providing client care. d. Bundle care to minimize exposure to the client. e. Initiate Transmission-Based Precautions.

ANS: B, C, D The client should be placed in a private room and dirty linens kept in the client's room until the radiation source is removed. The nurse should wear a lead apron while providing care, ensuring that the apron always faces the client. The nurse should also bundle care to minimize exposure to the client. Transmission-Based Precautions will not protect the nurse from the implanted radioactive iodine seeds.

A nurse works with several clients who have gout. Which types of gout and their drug treatments are correctly matched? (Select all that apply.) a. Allopurinol (Zyloprim) - Acute gout b. Colchicine (Colcrys) - Acute gout c. Febuxostat (Uloric) - Chronic gout d. Indomethacin (Indocin) - Acute gout e. Probenecid (Benemid) - Chronic gout

ANS: B, C, D, E Acute gout can be treated with colchicine and indomethacin. Chronic gout can be treated with febuxostat and probenecid. Allopurinol is used for chronic gout.

A nurse recently hired to the preoperative area learns that certain clients are at higher risk for venous thromboembolism (VTE). Which clients are considered at high risk? (Select all that apply.) a. Client with a humerus fracture b. Morbidly obese client c. Client who underwent a prolonged surgical procedure d. Client with severe heart failure e. Wheelchair-bound client

ANS: B, C, D, E All surgical clients should be assessed for VTE risk. Those considered at higher risk include those who are obese; are over 40; have cancer; have decreased mobility, immobility, or a spinal cord injury; have a history of any thrombotic event, varicose veins, or edema; take oral contraceptives or smoke; have decreased cardiac output; have a hip fracture; or are having total hip or knee surgery. Prolonged surgical time increases risk due to mobility and positioning needs.

A nurse assesses a client who is recovering from a Whipple procedure. Which clinical manifestations alert the nurse to a complication from this procedure? (Select all that apply.) a. Clay-colored stools b. Substernal chest pain c. Shortness of breath d. Lack of bowel sounds or flatus e. Urine output of 20 mL/6 hr

ANS: B, C, D, E Myocardial infarction (chest pain), pulmonary embolism (shortness of breath), adynamic ileus (lack of bowel sounds or flatus), and renal failure (urine output of 20 mL/6 hr) are just some of the complications for which the nurse must assess the client after the Whipple procedure. Clay-colored stools are associated with cholecystitis and are not a complication of a Whipple procedure.

A nurse working in the preoperative holding area performs which functions to ensure client safety? (Select all that apply.) a. Allow small sips of plain water. b. Check that consent is on the chart. c. Ensure the client has an armband on. d. Have the client help mark the surgical site. e. Allow the client to use the toilet before giving sedation.

ANS: B, C, D, E Providing for client safety is a priority function of the preoperative nurse. Checking for appropriately completed consent, verifying the client's identity, having the client assist in marking the surgical site if applicable, and allowing the client to use the toilet prior to sedating him or her are just some examples of important safety measures. The preoperative client should be NPO, so water should not be provided.

The student nurse caring for clients who have cancer understands that the general consequences of cancer include which client problems? (Select all that apply.) a. Clotting abnormalities from thrombocythemia b. Increased risk of infection from white blood cell deficits c. Nutritional deficits such as early satiety and cachexia d. Potential for reduced gas exchange e. Various motor and sensory deficits

ANS: B, C, D, E The general consequences of cancer include reduced immunity and blood-producing functions, altered GI structure and function, decreased respiratory function, and motor and sensory deficits. Clotting problems often occur due to thrombocytopenia (not enough platelets), not thrombocythemia (too many platelets).

A nurse prepares to admit a client who has herpes zoster. Which actions should the nurse take? (Select all that apply.) a.Prepare a room for reverse isolation. b.Assess staff for a history of or vaccination for chickenpox. c.Check the admission orders for analgesia. d.Choose a roommate who also is immune suppressed. e.Ensure that gloves are available in the room.

ANS: B, C, E Herpes zoster (shingles) is caused by reactivation of the same virus, varicella zoster, in clients who have previously had chickenpox. Anyone who has not had the disease or has not been vaccinated for it is at high risk for getting chickenpox. Herpes zoster is very painful and requires analgesia. Use of gloves and good handwashing are sufficient to prevent spread. It is best to put this client in a private room. Herpes zoster is a disease of immune suppression, so no one who is immune-suppressed should be in the same room.

An older adult client is hospitalized with Guillain-Barré syndrome. The client is given amitriptyline (Elavil). After receiving the hand-off report, what actions by the nurse are most important? (Select all that apply.) a. Administering the medication as ordered b. Advising the client to have help getting up c. Consulting the provider about the drug d. Cutting the dose of the drug in half e. Placing the client on safety precautions

ANS: B, C, E Amitriptyline is a tricyclic antidepressant and is considered inappropriate for use in older clients due to concerns of anticholinergic effects, confusion, and safety risks. The nurse should tell the client to have help getting up, place the client on safety precautions, and consult the provider. Since this drug is not appropriate for older clients, cutting the dose in half is not warranted.

A nurse is participating in primary prevention efforts directed against cancer. In which activities is this nurse most likely to engage? (Select all that apply.) a. Demonstrating breast self-examination methods to women b. Instructing people on the use of chemoprevention c. Providing vaccinations against certain cancers d. Screening teenage girls for cervical cancer e. Teaching teens the dangers of tanning booths

ANS: B, C, E Primary prevention aims to prevent the occurrence of a disease or disorder, in this case cancer. Secondary prevention includes screening and early diagnosis. Primary prevention activities include teaching people about chemoprevention, providing approved vaccinations to prevent cancer, and teaching teens the dangers of tanning beds. Breast examinations and screening for cervical cancer are secondary prevention methods.

A client with myasthenia gravis is malnourished. What actions to improve nutrition may the nurse delegate to the unlicensed assistive personnel (UAP)? (Select all that apply.) a. Assessing the client's gag reflex b. Cutting foods up into small bites c. Monitoring prealbumin levels d. Thickening liquids prior to drinking e. Weighing the client daily

ANS: B, D Cutting food up into smaller bites makes it easier for the client to chew and swallow. Thickened liquids help prevent aspiration. The UAP can weigh the client, but this does not help improve nutrition. The nurse assesses the gag reflex and monitors laboratory values.

An emergency room nurse assesses a client with potential liver trauma. Which clinical manifestations should alert the nurse to internal bleeding and hypovolemic shock? (Select all that apply.) a. Hypertension b. Tachycardia c. Flushed skin d. Confusion e. Shallow respirations

ANS: B, D Symptoms of hemorrhage and hypovolemic shock include hypotension, tachycardia, tachypnea, pallor, diaphoresis, cool and clammy skin, and confusion

A nurse evaluates the following laboratory results for a client who has hypoparathyroidism: Calcium 7.2 mg/dL Sodium 144 mEq/L Magnesium 1.2 mEq/L Potassium 5.7 mEq/L Based on these results, which medications should the nurse anticipate administering? (Select all that apply.) a. Oral potassium chloride b. Intravenous calcium chloride c. 3% normal saline IV solution d. 50% magnesium sulfate e. Oral calcitriol (Rocaltrol)

ANS: B, D The client has hypocalcemia (treated with calcium chloride) and hypomagnesemia (treated with magnesium sulfate). The potassium level is high, so replacement is not needed. The client's sodium level is normal, so hypertonic IV solution is not needed. No information about a vitamin D deficiency is evident, so calcitriol is not needed.

A nurse is dismissing a client from the emergency department who has a mild traumatic brain injury. What information obtained from the client represents a possible barrier to self-management? (Select all that apply.) a. Does not want to purchase a thermometer b. Is allergic to acetaminophen (Tylenol) c. Laughing, says "Strenuous? What's that?" d. Lives alone and is new in town with no friends e. Plans to have a beer and go to bed once home

ANS: B, D, E Clients should take acetaminophen for headache. An allergy to this drug may mean the client takes aspirin or ibuprofen (Motrin), which should be avoided. The client needs neurologic checks every 1 to 2 hours, and this client does not seem to have anyone available who can do that. Alcohol needs to be avoided for at least 24 hours. A thermometer is not needed. The client laughing at strenuous activity probably does not engage in any kind of strenuous activity, but the nurse should confirm this.

A nurse assesses a client who has encephalitis. Which manifestations should the nurse recognize as signs of increased intracranial pressure (ICP), a complication of encephalitis? (Select all that apply.) a. Photophobia b. Dilated pupils c. Headache d. Widened pulse pressure e. Bradycardia

ANS: B, D, E Increased ICP is a complication of encephalitis. The nurse should monitor for signs of increased ICP, including dilated pupils, widened pulse pressure, bradycardia, irregular respirations, and less responsive pupils. Photophobia and headache are not related to increased ICP.

A nurse is teaching a client who has chronic headaches. Which statements about headache triggers should the nurse include in this client's plan of care? (Select all that apply.) a. "Increase your intake of caffeinated beverages." b. "Incorporate physical exercise into your daily routine." c. "Avoid all alcoholic beverages." d. "Participate in a smoking cessation program." e. "Increase your intake of fruits and vegetables."

ANS: B, D, E Triggers for headaches include caffeine, smoking, and ingestion of pickled foods, so these factors should be avoided. Clients are taught to eat a balanced diet and to get adequate exercise and rest. Alcohol does not trigger chronic headaches but can enhance headaches during the headache period.

A nurse cares for a client with a lower motor neuron injury who is experiencing a flaccid bowel elimination pattern. Which actions should the nurse take to assist in relieving this client's constipation? (Select all that apply.) a. Pour warm water over the perineum. b. Provide a diet high in fluids and fiber. c. Administer daily tap water enemas. d. Implement a consistent daily time for elimination. e. Massage the abdomen from left to right. f. Perform manual disimpaction.

ANS: B, D, F For the client with a lower motor neuron injury, the resulting flaccid bowel may require a bowel program for the client that includes stool softeners, increased fluid intake, a high-fiber diet, and a consistent elimination time. If the client becomes impacted, the nurse would need to perform manual disimpaction. Pouring warm water over the perineum, administering daily enemas, and massaging the abdomen would not assist this client.

A nurse teaches a client who is recovering from acute pancreatitis. Which statements should the nurse include in this client's teaching? (Select all that apply.) a. "Take a 20-minute walk at least 5 days each week." b. "Attend local Alcoholics Anonymous (AA) meetings weekly." c. "Choose whole grains rather than foods with simple sugars." d. "Use cooking spray when you cook rather than margarine or butter." e. "Stay away from milk and dairy products that contain lactose." f. "We can talk to your doctor about a prescription for nicotine patches."

ANS: B, D, F The client should be advised to stay sober, and AA is a great resource. The client requires a low-fat diet, and cooking spray is low in fat compared with butter or margarine. If the client smokes, he or she must stop because nicotine can precipitate an exacerbation. A nicotine patch may help the client quit smoking. The client must rest until his or her strength returns. The client requires high carbohydrates and calories for healing; complex carbohydrates are not preferred over simple ones. Dairy products do not cause a problem.

A nursing instructor is teaching students about different surgical procedures and their classifications. Which examples does the instructor include? (Select all that apply.) a. Hemicolectomy: diagnostic b. Liver biopsy: diagnostic c. Mastectomy: restorative d. Spinal cord decompression: palliative e. Total shoulder replacement: restorative

ANS: B, E A diagnostic procedure is used to determine cell type of cancer and to determine the cause of a problem. An example is a liver biopsy. A restorative procedure aims to improve functional ability. An example would be a total shoulder replacement or a spinal cord decompression (not palliative). A curative procedure either removes or repairs the causative problem. An example would be a mastectomy (not restorative) or a hemicolectomy (not diagnostic). A palliative procedure relieves symptoms but will not cure the disease. An example is an ileostomy. A cosmetic procedure is done to improve appearance. An example is rhinoplasty (a "nose job").

A nurse is caring for a client after a stroke. What actions may the nurse delegate to the unlicensed assistive personnel (UAP)? (Select all that apply.) a. Assess neurologic status with the Glasgow Coma Scale. b. Check and document oxygen saturation every 1 to 2 hours. c. Cluster client care to allow periods of uninterrupted rest. d. Elevate the head of the bed to 45 degrees to prevent aspiration. e. Position the client supine with the head in a neutral midline position.

ANS: B, E The UAP can take and document vital signs, including oxygen saturation, and keep the client's head in a neutral, midline position with correct direction from the nurse. The nurse assesses the Glasgow Coma Scale score. The nursing staff should not cluster care because this can cause an increase in the intracranial pressure. The head of the bed should be minimally elevated, up to 30 degrees.

A nurse assesses a client who presents with an increase in psoriatic lesions. Which questions should the nurse ask to identify a possible trigger for worsening of this client's psoriatic lesions? (Select all that apply.) a."Have you eaten a large amount of chocolate lately?" b."Have you been under a lot of stress lately?" c."Have you recently used a public shower?" d."Have you been out of the country recently?" e."Have you recently had any other health problems?" f."Have you changed any medications recently?"

ANS: B, E, F Systemic factors, hormonal changes, psychological stress, medications, and general health factors can aggravate psoriasis. Psoriatic lesions are not triggered by chocolate, public showers, or international travel.

A nurse assesses a client who is experiencing an absence seizure. For which clinical manifestations should the nurse assess? (Select all that apply.) a. Intermittent rigidity b. Lip smacking c. Sudden loss of muscle tone d. Brief jerking of the extremities e. Picking at clothing f. Patting of the hand on the leg

ANS: B, E, F Automatisms are characteristic of absence seizures. These behaviors consist of lip smacking, picking at clothing, and patting. Rigidity of muscles is associated with the tonic phase of a seizure, and jerking of the extremities is associated with the clonic phase of a seizure. Loss of muscle tone occurs with atonic seizures.

A nurse assesses clients on a medical-surgical unit. Which client should the nurse evaluate for a wound infection? a.Client with blood cultures pending b.Client who has thin, serous wound drainage c.Client with a white blood cell count of 23,000/mm3 d.Client whose wound has decreased in size

ANS: C A client with an elevated white blood cell count should be evaluated for sources of infection. Pending cultures, thin drainage, and a decrease in wound size are not indications that the client may have an infection.

A nurse assesses a client who is being treated for hyperglycemic-hyperosmolar state (HHS). Which clinical manifestation indicates to the nurse that the therapy needs to be adjusted? a. Serum potassium level has increased. b. Blood osmolarity has decreased. c. Glasgow Coma Scale score is unchanged. d. Urine remains negative for ketone bodies.

ANS: C A slow but steady improvement in central nervous system functioning is the best indicator of therapy effectiveness for HHS. Lack of improvement in the level of consciousness may indicate inadequate rates of fluid replacement. The Glasgow Coma Scale assesses the client's state of consciousness against criteria of a scale including best eye, verbal, and motor responses. An increase in serum potassium, decreased blood osmolality, and urine negative for ketone bodies do not indicate adequacy of treatment.

A nurse assesses clients on a medical-surgical unit. Which client is at greatest risk for pressure ulcer development? a.A 44-year-old prescribed IV antibiotics for pneumonia b.A 26-year-old who is bedridden with a fractured leg c.A 65-year-old with hemi-paralysis and incontinence d.A 78-year-old requiring assistance to ambulate with a walker

ANS: C Being immobile and being incontinent are two significant risk factors for the development of pressure ulcers. The client with pneumonia does not have specific risk factors. The young client who has a fractured leg and the client who needs assistance with ambulation might be at moderate risk if they do not move about much, but having two risk factors makes the 65-year-old the person at highest risk.

A nurse assesses a client who has a 15-year history of diabetes and notes decreased tactile sensation in both feet. Which action should the nurse take first? a. Document the finding in the client's chart. b. Assess tactile sensation in the client's hands. c. Examine the client's feet for signs of injury. d. Notify the health care provider.

ANS: C Diabetic neuropathy is common when the disease is of long duration. The client is at great risk for injury in any area with decreased sensation because he or she is less able to feel injurious events. Feet are common locations for neuropathy and injury, so the nurse should inspect them for any signs of injury. After assessment, the nurse should document findings in the client's chart. Testing sensory perception in the hands may or may not be needed. The health care provider can be notified after assessment and documentation have been completed.

A nurse works on the surgical unit. After receiving the hand-off report, which client should the nurse see first? a. Client who underwent diverticula removal with a pulse of 106/min b. Client who had esophageal dilation and is attempting first postprocedure oral intake c. Client who had an esophagectomy with a respiratory rate of 32/min d. Client who underwent hernia repair, reporting incisional pain of 7/10

ANS: C The client who had an esophagectomy has a respiratory rate of 32/min, which is an early sign of sepsis; this client needs to be assessed first. The client who underwent diverticula removal has a pulse that is out of the normal range (106/min), but not terribly so. The client reporting pain needs pain medication, but the client with the elevated respiratory rate needs investigation first. The nurse should see the client who had esophageal dilation prior to and during the first attempt at oral feedings, but this can wait until the other clients are cared for.

A nurse cares for a client who is prescribed vancomycin (Vancocin) 500 mg IV every 6 hours for a methicillin-resistant Staphylococcus aureus (MRSA) infection. Which action should the nurse take? a.Administer it over 30 minutes using an IV pump. b.Give the client diphenhydramine (Benadryl) before the drug. c.Assess the IV site at least every 2 hours for thrombophlebitis. d.Ensure that the client has increased oral intake during therapy.

ANS: C Vancomycin is very irritating to the veins and can easily cause thrombophlebitis. This drug is given over at least 60 minutes; although it can cause histamine release (leading to "red man syndrome"), it is not customary to administer diphenhydramine before starting the infusion. Increasing oral intake is not specific to vancomycin therapy.

During skin inspection of a client, a nurse observes lesions with wavy borders that are widespread across the client's chest. Which descriptors should the nurse use to document these observations? a. Clustered and annular b. Linear and circinate c. Diffuse and serpiginous d. Coalesced and circumscribed

ANS: C "Diffuse" is used to describe lesions that are widespread. "Serpiginous" describes lesions with wavy borders. "Clustered" describes lesions grouped together. "Linear" describes lesions occurring in a straight line. Annular lesions are ringlike with raised borders, circinate lesions are circular, and circumscribed lesions have well-defined sharp borders. "Coalesced" describes lesions that merge with one another and appear confluent.

A nurse cares for a client who is prescribed a serum catecholamine test. Which action should the nurse take when obtaining the sample? a. Discard the first sample and then begin the collection. b. Draw the blood sample after the client eats breakfast. c. Place the sample on ice and send to the laboratory immediately. d. Add preservatives before sending the sample to the laboratory.

ANS: C A blood sample for catecholamine must be placed on ice and taken to the laboratory immediately. This sample is not urine, and therefore the first sample should not be discarded nor should preservatives be added to the sample. The nurse should use the appropriate tube and obtain the sample based on which drugs are administered, not dietary schedules.

A nurse plans care for a client who has hypothyroidism and is admitted for pneumonia. Which priority intervention should the nurse include in this client's plan of care? a. Monitor the client's intravenous site every shift. b. Administer acetaminophen (Tylenol) for fever. c. Ensure that working suction equipment is in the room. d. Assess the client's vital signs every 4 hours.

ANS: C A client with hypothyroidism who develops another illness is at risk for myxedema coma. In this emergency situation, maintaining an airway is a priority. The nurse should ensure that suction equipment is available in the client's room because it may be needed if myxedema coma develops. The other interventions are necessary for any client with pneumonia, but having suction available is a safety feature for this client.

A nurse cares for several clients on a neurologic unit. Which prescription for a client should direct the nurse to ensure that an informed consent has been obtained before the test or procedure? a. Sensation measurement via the pinprick method b. Computed tomography of the cranial vault c. Lumbar puncture for cerebrospinal fluid sampling d. Venipuncture for autoantibody analysis

ANS: C A lumbar puncture is an invasive procedure with many potentially serious complications. The other assessments or tests are considered noninvasive and do not require an informed consent.

A client had a colonoscopy and biopsy yesterday and calls the gastrointestinal clinic to report a spot of bright red blood on the toilet paper today. What response by the nurse is best? a. Ask the client to call back if this happens again today. b. Instruct the client to go to the emergency department. c. Remind the client that a small amount of bleeding is possible. d. Tell the client to come in to the clinic this afternoon.

ANS: C After a colonoscopy with biopsy, a small amount of bleeding is normal. The nurse should remind the client of this and instruct him or her to go to the emergency department for large amounts of bleeding, severe pain, or dizziness.

A nurse is teaching a client with chronic migraine headaches. Which statement related to complementary therapy should the nurse include in this client's teaching? a. "Place a warm compress on your forehead at the onset of the headache." b. "Wear dark sunglasses when you are in brightly lit spaces." c. "Lie down in a darkened room when you experience a headache." d. "Set your alarm to ensure you do not sleep longer than 6 hours at one time."

ANS: C At the onset of a migraine attack, the client may be able to alleviate pain by lying down and darkening the room. He or she may want both eyes covered and a cool cloth on the forehead. If the client falls asleep, he or she should remain undisturbed until awakening. The other options are not recognized therapies for migraines.

The nurse is preparing a client for a Tensilon (edrophonium chloride) test. What action by the nurse is most important? a. Administering anxiolytics b. Having a ventilator nearby c. Obtaining atropine sulfate d. Sedating the client

ANS: C Atropine is the antidote to edrophonium chloride and should be readily available when a client is having a Tensilon test. The nurse would not want to give medications that might cause increased weakness or sedation. A ventilator is not necessary to have nearby, although emergency equipment should be available.

A client has fibromyalgia and is prescribed duloxetine hydrochloride (Cymbalta). The client calls the clinic and asks the nurse why an antidepressant drug has been prescribed. What response by the nurse is best? a. "A little sedation will help you get some rest." b. "Depression often accompanies fibromyalgia." c. "This drug works in the brain to decrease pain." d. "You will have more energy after taking this drug."

ANS: C Duloxetine works to increase the release of the neurotransmitters serotonin and norepinephrine, which reduces the pain from fibromyalgia. The other answers are inaccurate.

A nurse evaluates laboratory results for a male client who reports fluid secretion from his breasts. Which hormone value should the nurse assess first? a. Posterior pituitary hormones b. Adrenal medulla hormones c. Anterior pituitary hormones d. Parathyroid hormone

ANS: C Breast fluid and milk production are induced by the presence of prolactin, secreted from the anterior pituitary gland. The other hormones would not cause fluid secretion from the client's breast.

A nurse teaches a client who is recovering from a colon resection. Which statement should the nurse include in this client's plan of care? a. "You may experience nausea and vomiting for the first few weeks." b. "Carbonated beverages can help decrease acid reflux from anastomosis sites." c. "Take a stool softener to promote softer stools for ease of defecation." d. "You may return to your normal workout schedule, including weight lifting."

ANS: C Clients recovering from a colon resection should take a stool softener as prescribed to keep stools a soft consistency for ease of passage. Nausea and vomiting are symptoms of intestinal obstruction and perforation and should be reported to the provider immediately. The client should be advised to avoid gas-producing foods and carbonated beverages, and avoid lifting heavy objects or straining on defecation.

A nurse cares for a client who has chronic cirrhosis from substance abuse. The client states, "All of my family hates me." How should the nurse respond? a. "You should make peace with your family." b. "This is not unusual. My family hates me too." c. "I will help you identify a support system." d. "You must attend Alcoholics Anonymous."

ANS: C Clients who have chronic cirrhosis may have alienated relatives over the years because of substance abuse. The nurse should assist the client to identify a friend, neighbor, or person in his or her recovery group for support. The nurse should not minimize the client's concerns by brushing off the client's comment. Attending AA may be appropriate, but this response doesn't address the client's concern. Making peace with the client's family may not be possible. This statement is not client-centered.

A nurse prepares to discharge a client with Alzheimer's disease. Which statement should the nurse include in the discharge teaching for this client's caregiver? a. "Allow the client to rest most of the day." b. "Place a padded throw rug at the bedside." c. "Install deadbolt locks on all outside doors." d. "Provide a high-calorie and high-protein diet."

ANS: C Clients with Alzheimer's disease have a tendency to wander, especially at night. If possible, alarms should be installed on all outside doors to alert family members if the client leaves. At a minimum, all outside doors should have deadbolt locks installed to prevent the client from going outdoors unsupervised. The client should be allowed to exercise within his or her limits. Throw rugs are a slip and fall hazard and should be removed. The client should eat a well-balanced diet. There is no need for a high-calorie or high-protein diet.

A nurse cares for an older adult client who has a chronic skin disorder. The client states, "I have not been to church in several weeks because of the discoloration of my skin." How should the nurse respond? a. "I will consult the chaplain to provide you with spiritual support." b. "You do not need to go to church; God is everywhere." c. "Tell me more about your concerns related to your skin." d. "Religious people are nonjudgmental and will accept you."

ANS: C Clients with chronic skin disorders often become socially isolated related to the fear of rejection by others. Nurses should assess how the client's skin changes are affecting the client's body image and encourage the client to express his or her feelings about a change in appearance. The other responses are not appropriate.

A nurse is teaching the daughter of a client who has Alzheimer's disease. The daughter asks, "Will the medication my mother is taking improve her dementia?" How should the nurse respond? a. "It will allow your mother to live independently for several more years." b. "It is used to halt the advancement of Alzheimer's disease but will not cure it." c. "It will not improve her dementia but can help control emotional responses." d. "It is used to improve short-term memory but will not improve problem solving."

ANS: C Drug therapy is not effective for treating dementia or halting the advancement of Alzheimer's disease. However, certain drugs may help suppress emotional disturbances and psychiatric manifestations. Medication therapy may not allow the client to safely live independently.

A nurse cares for a client with excessive production of thyrocalcitonin (calcitonin). For which electrolyte imbalance should the nurse assess? a. Potassium b. Sodium c. Calcium d. Magnesium

ANS: C Parafollicular cells produce thyrocalcitonin (calcitonin), which regulates serum calcium levels. Calcitonin has no impact on potassium, sodium, or magnesium balances.

A nurse cares for a client with acute pancreatitis. The client states, "I am hungry." How should the nurse reply? a. "Is your stomach rumbling or do you have bowel sounds?" b. "I need to check your gag reflex before you can eat." c. "Have you passed any flatus or moved your bowels?" d. "You will not be able to eat until the pain subsides."

ANS: C Paralytic ileus is a common complication of acute pancreatitis. The client should not eat until this has resolved. Bowel sounds and decreased pain are not reliable indicators of peristalsis. Instead, the nurse should assess for passage of flatus or bowel movement.

A nurse is caring for a client with paraplegia who is scheduled to participate in a rehabilitation program. The client states, "I do not understand the need for rehabilitation; the paralysis will not go away and it will not get better." How should the nurse respond? a. "If you don't want to participate in the rehabilitation program, I'll let the provider know." b. "Rehabilitation programs have helped many clients with your injury. You should give it a chance." c. "The rehabilitation program will teach you how to maintain the functional ability you have and prevent further disability." d. "When new discoveries are made regarding paraplegia, people in rehabilitation programs will benefit first."

ANS: C Participation in rehabilitation programs has many purposes, including prevention of disability, maintenance of functional ability, and restoration of function. The other responses do not meet this client's needs.

A client is admitted with superior vena cava syndrome. What action by the nurse is most appropriate? a. Administer a dose of allopurinol (Aloprim). b. Assess the client's serum potassium level. c. Gently inquire about advance directives. d. Prepare the client for emergency surgery.

ANS: C Superior vena cava syndrome is often a late-stage manifestation. After the client is stabilized and comfortable, the nurse should initiate a conversation about advance directives. Allopurinol is used for tumor lysis syndrome. Potassium levels are important in tumor lysis syndrome, in which cell destruction leads to large quantities of potassium being released into the bloodstream. Surgery is rarely done for superior vena cava syndrome.

Four clients are receiving tyrosine kinase inhibitors (TKIs). Which of these four clients should the nurse assess first? a. Client with dry, itchy, peeling skin b. Client with a serum calcium of 9.2 mg/dL c. Client with a serum potassium of 2.8 mEq/L d. Client with a weight gain of 0.5 pound (1.1 kg) in 1 day

ANS: C TKIs can cause electrolyte imbalances. This potassium level is very low, so the nurse should assess this client first. Dry, itchy, peeling skin can be a problem in clients receiving biologic response modifiers, and the nurse should assess that client next because of the potential for discomfort and infection. This calcium level is normal. TKIs can also cause weight gain, but the client with the low potassium level is more critical.

After teaching a client who has plans to travel to a non-industrialized country, the nurse assesses the client's understanding regarding the prevention of viral hepatitis. Which statement made by the client indicates a need for additional teaching? a. "I should drink bottled water during my travels." b. "I will not eat off another's plate or share utensils." c. "I should eat plenty of fresh fruits and vegetables." d. "I will wash my hands frequently and thoroughly."

ANS: C The client should be advised to avoid fresh, raw fruits and vegetables because they can be contaminated by tap water. Drinking bottled water, and not sharing plates, glasses, or eating utensils are good ways to prevent illness, as is careful handwashing.

A nurse assesses clients on the medical-surgical unit. Which client is at greatest risk for the development of carcinoma of the liver? a. A 22-year-old with a history of blunt liver trauma b. A 48-year-old with a history of diabetes mellitus c. A 66-year-old who has a history of cirrhosis d. An 82-year-old who has chronic malnutrition

ANS: C The risk of contracting a primary carcinoma of the liver is higher in clients with cirrhosis from any cause. Blunt liver trauma, diabetes mellitus, and chronic malnutrition do not increase a person's risk for developing liver cancer.

A client had an oral tumor removed this morning and now has a tracheostomy. What action by the nurse is the priority? a. Delegate oral care every 4 hours. b. Monitor and record the client's intake. c. Place the client in a high-Fowler's position. d. Remove the inner cannula for cleaning.

ANS: C To promote airway clearance, this client should be placed in a semi- or high-Fowler's position. Oral care can be delegated, but that is not the priority. Intake and output should also be recorded but again is not the priority. The inner cannula may or may not need to be cleaned, and the tracheostomy may or may not have a disposable cannula.

A nurse assesses a client who is recovering from a lumbar laminectomy. Which complications should alert the nurse to urgently communicate with the health care provider? (Select all that apply.) a. Surgical discomfort b. Redness and itching at the incision site c. Incisional bulging d. Clear drainage on the dressing e. Sudden and severe headache

ANS: C, D, E Bulging at the incision site or clear fluid on the dressing after a laminectomy strongly suggests a cerebrospinal fluid leak, which constitutes an emergency. Loss of cerebral spinal fluid may cause a sudden and severe headache, which is also an emergency situation. Pain, redness, and itching at the site are normal.

After teaching a male client with a spinal cord injury at the T4 level, the nurse assesses the client's understanding. Which client statements indicate a correct understanding of the teaching related to sexual effects of this injury? (Select all that apply.) a. "I will explore other ways besides intercourse to please my partner." b. "I will not be able to have an erection because of my injury." c. "Ejaculation may not be as predictable as before." d. "I may urinate with ejaculation but this will not cause infection." e. "I should be able to have an erection with stimulation."

ANS: C, D, E Men with injuries above T6 often are able to have erections by stimulating reflex activity. For example, stroking the penis will cause an erection. Ejaculation is less predictable and may be mixed with urine. However, urine is sterile, so the client's partner will not get an infection.

The nurse is caring for a client with systemic sclerosis (SSc). What comfort measures can the nurse delegate to the unlicensed assistive personnel (UAP)? (Select all that apply.) a. Collaborate with a registered dietitian for appropriate foods. b. Inspect the skin and note any areas of ulceration. c. Keep the room at a comfortably warm temperature. d. Place a foot cradle at the end of the bed to lift sheets. e. Remind the client to elevate the head of the bed after eating.

ANS: C, D, E The client with SSc should avoid cold temperatures, which may lead to vasospasms and Raynaud's phenomenon. The UAP can adjust the room temperature for the client's comfort. Keeping the sheets off the feet will help prevent injury; the UAP can apply a foot cradle to the bed to hold the sheets up. Because of esophageal problems, the client should remain in an upright position for 1 to 2 hours after meals. The UAP can remind the client of this once he or she has been taught. The other actions are performed by the registered nurse.

A nurse assesses a client with paraplegia from a spinal cord injury and notes reddened areas over the client's hips and sacrum. Which actions should the nurse take? (Select all that apply.) a. Apply a barrier cream to protect the skin from excoriation. b. Perform range-of-motion (ROM) exercises for the hip joint. c. Re-position the client off of the reddened areas. d. Get the client out of bed and into a chair once a day. e. Obtain a low-air-loss mattress to minimize pressure.

ANS: C, E Appropriate interventions to relieve pressure on these areas include frequent re-positioning and a low-air-loss mattress. Reddened areas should not be rubbed because this action could cause more extensive damage to the already fragile capillary system. Barrier cream will not protect the skin from pressure wounds. ROM exercises are used to prevent contractures. Sitting the client in a chair once a day will decrease the client's risk of respiratory complications but will not decrease pressure on the client's hips and sacrum.

A nurse cares for a client with elevated triiodothyronine and thyroxine, and normal thyroid-stimulating hormone levels. Which actions should the nurse take? (Select all that apply.) a. Administer levothyroxine (Synthroid). b. Administer propranolol (Inderal). c. Monitor the apical pulse. d. Assess for Trousseau's sign. e. Initiate telemetry monitoring.

ANS: C, E The client's laboratory findings suggest that the client is experiencing hyperthyroidism. The increased metabolic rate can cause an increase in the client's heart rate, and the client should be monitored for the development of dysrhythmias. Placing the client on a telemetry monitor might also be a precaution. Levothyroxine is given for hypothyroidism. Propranolol is a beta blocker often used to lower sympathetic nervous system activity in hyperthyroidism. Trousseau's sign is a test for hypocalcemia.

A nurse is caring for a client who has a pressure ulcer on the right ankle. Which action should the nurse take first? a.Draw blood for albumin, prealbumin, and total protein. b.Prepare for and assist with obtaining a wound culture. c.Place the client in bed and instruct the client to elevate the foot. d.Assess the right leg for pulses, skin color, and temperature.

ANS: D A client with an ulcer on the foot should be assessed for interruption in arterial flow to the area. This begins with the assessment of pulses and color and temperature of the skin. The nurse can also assess for pulses noninvasively with a Doppler flowmeter if unable to palpate with his or her fingers. Tests to determine nutritional status and risk assessment would be completed after the initial assessment is done. Wound cultures are done after it has been determined that drainage, odor, and other risks for infection are present. Elevation of the foot would impair the ability of arterial blood to flow to the area.

After hiatal hernia repair surgery, a client is on IV pantoprazole (Protonix). The client asks the nurse why this medication is given since there is no history of ulcers. What response by the nurse is best? a. "Bacteria can often cause ulcers." b. "This operation often causes ulcers." c. "The medication keeps your blood pH low." d. "It prevents stress-related ulcers."

ANS: D After surgery, anti-ulcer medications such as pantoprazole are often given to prevent stress-related ulcers. The other responses are incorrect.

After teaching a client how to care for a furuncle in the axilla, a nurse assesses the client's understanding. Which statement indicates the client correctly understands the teaching? a."I'll apply cortisone cream to reduce the inflammation." b."I'll apply a clean dressing after squeezing out the pus." c."I'll keep my arm down at my side to prevent spread." d."I'll cleanse the area prior to applying antibiotic cream."

ANS: D Cleansing and topical antibiotics can eliminate the infection. Warm compresses enhance comfort and open the lesion, allowing better penetration of the topical antibiotic. Cortisone cream reduces the inflammatory response but increases the infectious process. Squeezing the lesion may introduce infection to deeper tissues and cause cellulitis. Keeping the arm down increases moisture in the area and promotes bacterial growth.

A nurse teaches a client with diabetes mellitus who is experiencing numbness and reduced sensation. Which statement should the nurse include in this client's teaching to prevent injury? a. "Examine your feet using a mirror every day." b. "Rotate your insulin injection sites every week." c. "Check your blood glucose level before each meal." d. "Use a bath thermometer to test the water temperature."

ANS: D Clients with diminished sensory perception can easily experience a burn injury when bathwater is too hot. Instead of checking the temperature of the water by feeling it, they should use a thermometer. Examining the feet daily does not prevent injury, although daily foot examinations are important to find problems so they can be addressed. Rotating insulin and checking blood glucose levels will not prevent injury.

When teaching a client recently diagnosed with type 1 diabetes mellitus, the client states, "I will never be able to stick myself with a needle." How should the nurse respond? a. "I can give your injections to you while you are here in the hospital." b. "Everyone gets used to giving themselves injections. It really does not hurt." c. "Your disease will not be managed properly if you refuse to administer the shots." d. "Tell me what it is about the injections that are concerning you."`

ANS: D Devote as much teaching time as possible to insulin injection and blood glucose monitoring. Clients with newly diagnosed diabetes are often fearful of giving themselves injections. If the client is worried about giving the injections, it is best to try to find out what specifically is causing the concern, so it can be addressed. Giving the injections for the client does not promote self-care ability. Telling the client that others give themselves injections may cause the client to feel bad. Stating that you don't know another way to manage the disease is dismissive of the client's concerns.

After teaching a client who has diabetes mellitus and proliferative retinopathy, nephropathy, and peripheral neuropathy, the nurse assesses the client's understanding. Which statement made by the client indicates a correct understanding of the teaching? a. "I have so many complications; exercising is not recommended." b. "I will exercise more frequently because I have so many complications." c. "I used to run for exercise; I will start training for a marathon." d. "I should look into swimming or water aerobics to get my exercise."

ANS: D Exercise is not contraindicated for this client, although modifications based on existing pathology are necessary to prevent further injury. Swimming or water aerobics will give the client exercise without the worry of having the correct shoes or developing a foot injury. The client should not exercise too vigorously.

A nurse teaches a client who is prescribed an insulin pump. Which statement should the nurse include in this client's discharge education? a. "Test your urine daily for ketones." b. "Use only buffered insulin in your pump." c. "Store the insulin in the freezer until you need it." d. "Change the needle every 3 days."

ANS: D Having the same needle remain in place through the skin for longer than 3 days drastically increases the risk for infection in or through the delivery system. Having an insulin pump does not require the client to test for ketones in the urine. Insulin should not be frozen. Insulin is not buffered.

A nurse reviews laboratory results for a client with diabetes mellitus who presents with polyuria, lethargy, and a blood glucose of 560 mg/dL. Which laboratory result should the nurse correlate with the client's polyuria? a. Serum sodium: 163 mEq/L b. Serum creatinine: 1.6 mg/dL c. Presence of urine ketone bodies d. Serum osmolarity: 375 mOsm/kg

ANS: D Hyperglycemia causes hyperosmolarity of extracellular fluid. This leads to polyuria from an osmotic diuresis. The client's serum osmolarity is high. The client's sodium would be expected to be high owing to dehydration. Serum creatinine and urine ketone bodies are not related to the polyuria.

A nurse reviews the laboratory results of a client who is receiving intravenous insulin. Which should alert the nurse to intervene immediately? a. Serum chloride level of 98 mmol/L b. Serum calcium level of 8.8 mg/dL c. Serum sodium level of 132 mmol/L d. Serum potassium level of 2.5 mmol/L

ANS: D Insulin activates the sodium-potassium ATPase pump, increasing the movement of potassium from the extracellular fluid into the intracellular fluid, resulting in hypokalemia. In hyperglycemia, hypokalemia can also result from excessive urine loss of potassium. The chloride level is normal. The calcium and sodium levels are slightly low, but this would not be related to hyperglycemia and insulin administration.

A nurse cares for clients who have various skin infections. Which infection is paired with the correct pharmacologic treatment? a.Viral infection - Clindamycin (Cleocin) b.Bacterial infection - Acyclovir (Zovirax) c.Yeast infection - Linezolid (Zyvox) d.Fungal infection - Ketoconazole (Nizoral)

ANS: D Ketoconazole is an antifungal. Clindamycin and linezolid are antibiotics. Acyclovir is an antiviral drug.

A nurse teaches a client with type 1 diabetes mellitus. Which statement should the nurse include in this client's teaching to decrease the client's insulin needs? a. "Limit your fluid intake to 2 liters a day." b. "Animal organ meat is high in insulin." c. "Limit your carbohydrate intake to 80 grams a day." d. "Walk at a moderate pace for 1 mile daily."

ANS: D Moderate exercise such as walking helps regulate blood glucose levels on a daily basis and results in lowered insulin requirements for clients with type 1 diabetes mellitus. Restricting fluids and eating organ meats will not reduce insulin needs. People with diabetes need at least 130 grams of carbohydrates each day.

After teaching a client with type 2 diabetes mellitus who is prescribed nateglinide (Starlix), the nurse assesses the client's understanding. Which statement made by the client indicates a correct understanding of the prescribed therapy? a. "I'll take this medicine during each of my meals." b. "I must take this medicine in the morning when I wake." c. "I will take this medicine before I go to bed." d. "I will take this medicine immediately before I eat."

ANS: D Nateglinide is an insulin secretagogue that is designed to increase meal-related insulin secretion. It should be taken immediately before each meal. The medication should not be taken without eating as it will decrease the client's blood glucose levels. The medication should be taken before meals instead of during meals.

A nurse teaches a client who has very dry skin. Which statement should the nurse include in this client's education? a."Use lots of moisturizer several times a day to minimize dryness." b."Take a cold shower instead of soaking in the bathtub." c."Use antimicrobial soap to avoid infection of cracked skin." d."After you bathe, put lotion on before your skin is totally dry."

ANS: D The client should bathe in warm water for at least 20 minutes and then apply lotion immediately because this will keep the moisture in the skin. Just using moisturizer will not be as helpful because the moisturizer is not what rehydrates the skin; it is the water. Bathing in warm water will rehydrate skin more effectively than a cold shower, and antimicrobial soaps are actually more drying than other kinds of soap.

A client has returned to the nursing unit after an open Nissen fundoplication. The client has an indwelling urinary catheter, a nasogastric (NG) tube to low continuous suction, and two IVs. The nurse notes bright red blood in the NG tube. What action should the nurse take first? a. Document the findings in the chart. b. Notify the surgeon immediately. c. Reassess the drainage in 1 hour. d. Take a full set of vital signs.

ANS: D The drainage in the NG tube should initially be brown with old blood. The presence of bright red blood indicates bleeding. The nurse should take a set of vital signs to assess for shock and then notify the surgeon. Documentation should occur but is not the first thing the nurse should do. The nurse should not wait an additional hour to reassess.

A nurse cares for a client experiencing diabetic ketoacidosis who presents with Kussmaul respirations. Which action should the nurse take? a. Administration of oxygen via face mask b. Intravenous administration of 10% glucose c. Implementation of seizure precautions d. Administration of intravenous insulin

ANS: D The rapid, deep respiratory efforts of Kussmaul respirations are the body's attempt to reduce the acids produced by using fat rather than glucose for fuel. Only the administration of insulin will reduce this type of respiration by assisting glucose to move into cells and to be used for fuel instead of fat. The client who is in ketoacidosis may not experience any respiratory impairment and therefore does not need additional oxygen. Giving the client glucose would be contraindicated. The client does not require seizure precautions.

A nurse cares for a client who has type 1 diabetes mellitus. The client asks, "Is it okay for me to have an occasional glass of wine?" How should the nurse respond? a. "Drinking any wine or alcohol will increase your insulin requirements." b. "Because of poor kidney function, people with diabetes should avoid alcohol." c. "You should not drink alcohol because it will make you hungry and overeat." d. "One glass of wine is okay with a meal and is counted as two fat exchanges."

ANS: D Under normal circumstances, blood glucose levels will not be affected by moderate use of alcohol when diabetes is well controlled. Because alcohol can induce hypoglycemia, it should be ingested with or shortly after a meal. One alcoholic beverage is substituted for two fat exchanges when caloric intake is calculated. Kidney function is not impacted by alcohol intake. Alcohol is not associated with increased hunger or overeating.

A nurse assesses a client with a mechanical bowel obstruction who reports intermittent abdominal pain. An hour later the client reports constant abdominal pain. Which action should the nurse take next? a. Administer intravenous opioid medications. b. Position the client with knees to chest. c. Insert a nasogastric tube for decompression. d. Assess the client's bowel sounds.

ANS: D A change in the nature and timing of abdominal pain in a client with a bowel obstruction can signal peritonitis or perforation. The nurse should immediately check for rebound tenderness and the absence of bowel sounds. The nurse should not medicate the client until the provider has been notified of the change in his or her condition. The nurse may help the client to the knee-chest position for comfort, but this is not the priority action. The nurse need not insert a nasogastric tube for decompression.

A nurse assesses a client who is recovering from a diskectomy 6 hours ago. Which assessment finding should the nurse address first? a. Sleepy but arouses to voice b. Dry and cracked oral mucosa c. Pain present in lower back d. Bladder palpated above pubis

ANS: D A distended bladder may indicate damage to the sacral spinal nerves. The other findings require the nurse to provide care but are not the priority or a complication of the procedure.

The nurse is caring for four clients with traumatic brain injuries. Which client should the nurse assess first? a. Client with cerebral perfusion pressure of 72 mm Hg b. Client who has a Glasgow Coma Scale score of 12 c. Client with a PaCO2 of 36 mm Hg who is on a ventilator d. Client who has a temperature of 102° F (38.9° C)

ANS: D A fever is a poor prognostic indicator in clients with brain injuries. The nurse should see this client first. A Glasgow Coma Scale score of 12, a PaCO2 of 36, and cerebral perfusion pressure of 72 mm Hg are all desired outcomes.

A nurse assesses a client on the medical-surgical unit. Which statement made by the client should alert the nurse to the possibility of hypothyroidism? a. "My sister has thyroid problems." b. "I seem to feel the heat more than other people." c. "Food just doesn't taste good without a lot of salt." d. "I am always tired, even with 12 hours of sleep."

ANS: D Clients with hypothyroidism usually feel tired or weak despite getting many hours of sleep. Thyroid problems are not inherited. Heat intolerance is indicative of hyperthyroidism. Loss of taste is not a manifestation of hypothyroidism.

A nurse assesses clients at a community health center. Which client is at highest risk for the development of colorectal cancer? a. A 37-year-old who drinks eight cups of coffee daily b. A 44-year-old with irritable bowel syndrome (IBS) c. A 60-year-old lawyer who works 65 hours per week d. A 72-year-old who eats fast food frequently

ANS: D Colon cancer is rare before the age of 40, but its incidence increases rapidly with advancing age. Fast food tends to be high in fat and low in fiber, increasing the risk for colon cancer. Coffee intake, IBS, and a heavy workload do not increase the risk for colon cancer.

After teaching a young adult client who is newly diagnosed with type 1 diabetes mellitus, the nurse assesses the client's understanding. Which statement made by the client indicates a correct understanding of the need for eye examinations? a. "At my age, I should continue seeing the ophthalmologist as I usually do." b. "I will see the eye doctor when I have a vision problem and yearly after age 40." c. "My vision will change quickly. I should see the ophthalmologist twice a year." d. "Diabetes can cause blindness, so I should see the ophthalmologist yearly."

ANS: D Diabetic retinopathy is a leading cause of blindness in North America. All clients with diabetes, regardless of age, should be examined by an ophthalmologist (rather than an optometrist or optician) at diagnosis and at least yearly thereafter.

After teaching a client who is diagnosed with new-onset status epilepticus and prescribed phenytoin (Dilantin), the nurse assesses the client's understanding. Which statement by the client indicates a correct understanding of the teaching? a. "To prevent complications, I will drink at least 2 liters of water daily." b. "This medication will stop me from getting an aura before a seizure." c. "I will not drive a motor vehicle while taking this medication." d. "Even when my seizures stop, I will continue to take this drug."

ANS: D Discontinuing antiepileptic drugs can lead to the recurrence of seizures or status epilepticus. The client does not need to drink more water and can drive while taking this medication. The medication will not stop an aura before a seizure.

A client with cancer is admitted to a short-term rehabilitation facility. The nurse prepares to administer the client's oral chemotherapy medications. What action by the nurse is most appropriate? a. Crush the medications if the client cannot swallow them. b. Give one medication at a time with a full glass of water. c. No special precautions are needed for these medications. d. Wear personal protective equipment when handling the medications.

ANS: D During the administration of oral chemotherapy agents, nurses must take the same precautions that are used when administering IV chemotherapy. This includes using personal protective equipment. These medications cannot be crushed, split, or chewed. Giving one at a time is not needed.

A nurse cares for a client who is recovering from a parathyroidectomy. When taking the client's blood pressure, the nurse notes that the client's hand has gone into flexion contractions. Which laboratory result does the nurse correlate with this condition? a. Serum potassium: 2.9 mEq/L b. Serum magnesium: 1.7 mEq/L c. Serum sodium: 122 mEq/L d. Serum calcium: 6.9 mg/dL

ANS: D Hypocalcemia destabilizes excitable membranes and can lead to muscle twitches, spasms, and tetany. This effect of hypocalcemia is enhanced in the presence of tissue hypoxia. The flexion contractions (Trousseau's sign) that occur during blood pressure measurement are indicative of hypocalcemia, not the other electrolyte imbalances, which include hypokalemia, hyponatremia, and hypomagnesemia.

A nurse assesses a client who is prescribed levothyroxine (Synthroid) for hypothyroidism. Which assessment finding should alert the nurse that the medication therapy is effective? a. Thirst is recognized and fluid intake is appropriate. b. Weight has been the same for 3 weeks. c. Total white blood cell count is 6000 cells/mm3. d. Heart rate is 70 beats/min and regular.

ANS: D Hypothyroidism decreases body functioning and can result in effects such as bradycardia, confusion, and constipation. If a client's heart rate is bradycardic while on thyroid hormone replacement, this is an indicator that the replacement may not be adequate. Conversely, a heart rate above 100 beats/min may indicate that the client is receiving too much of the thyroid hormone. Thirst, fluid intake, weight, and white blood cell count do not represent a therapeutic response to this medication.

A nurse assesses a client with a neurologic disorder. Which assessment finding should the nurse identify as a late manifestation of amyotrophic lateral sclerosis (ALS)? a. Dysarthria b. Dysphagia c. Muscle weakness d. Impairment of respiratory muscles

ANS: D In ALS, progressive muscle atrophy occurs until a flaccid quadriplegia develops. Eventually, the respiratory muscles are involved, which leads to respiratory compromise. Dysarthria, dysphagia, and muscle weakness are early clinical manifestations of ALS.

A nurse assesses clients who are at risk for diabetes mellitus. Which client is at greatest risk? a. A 29-year-old Caucasian b. A 32-year-old African-American c. A 44-year-old Asian d. A 48-year-old American Indian

Diabetes is a particular problem among African Americans, Hispanics, and American Indians. The incidence of diabetes increases in all races and ethnic groups with age. Being both an American Indian and middle-aged places this client at highest risk.

A nurse assesses clients at a community health fair. Which client is at greatest risk for the development of hepatitis B? a. A 20-year-old college student who has had several sexual partners b. A 46-year-old woman who takes acetaminophen daily for headaches c. A 63-year-old businessman who travels frequently across the country d. An 82-year-old woman who recently ate raw shellfish for dinner

Hepatitis B can be spread through sexual contact, needle sharing, needle sticks, blood transfusions, hemodialysis, acupuncture, and the maternal-fetal route. A person with multiple sexual partners has more opportunities to contract the infection. Hepatitis B is not transmitted through medications, casual contact with other travelers, or raw shellfish. Although an overdose of acetaminophen can cause liver cirrhosis, this is not associated with hepatitis B. Hepatitis E is found most frequently in international travelers. Hepatitis A is spread through ingestion of contaminated shellfish.


Conjuntos de estudio relacionados

Chapter 25 the reproductive system

View Set

Chapter 2 biomechanics questions

View Set

Earth Science Exogenic Processes

View Set

PrepU: Chapter 15: Postpartum Adaptations

View Set

Genetic Disorders [Robbins and Cotran Review of Pathology (4E) CH5/Rubins Illustrated Pathology Review (2E) CH6]

View Set

Strategy week 5 (strategic innovation, industry context)

View Set